You are on page 1of 738

Orthopaedic Oncology

OITE Review 2012


8.29.2012

Ruth Delaney, MD

Special thanks to Matteos, Boulton,


Kujo, First Aid for the Boards
Those who have come before us
Introduction
• History: age, sex,
duration of sx, presence
and quality of pain,
history of trauma, weight
loss, smoking history and
history of prior
malignancy
– Red flags: pain that
extends beyond expected
duration, night pain, T-
spine pain
– Beware the history of
trauma
Radiographic Differential
Diagnosis
Bone Forming Tumors Cartilage Forming Tumors The Rest

Osteoid Osteoma Osteochondroma Infection


Osteoblastoma Chondromyxoid fibroma Metastases
Osteosarcoma Chondroblastoma Round cell tumors
Blastic Metastases Enchondroma Fibrous Dysplasia
Paget's Disease Chondrosarcoma Non-Ossifying Fibroma
(Fibrous Dysplasia) Simple Bone Cyst
Aneurysmal Bone Cyst
Histiocytosis
Giant cell tumor
Metabolic condition
PINK BLUE
Bone-Producing Tumors
• Osteoid Osteoma
• Osteoblastoma
• Enostosis (bone island)
• Osteosarcoma
Bone Forming Tumors
• Possibilities: fracture callus,
myositis ossificans, fibrous
dysplasia, osteosarcoma,
osteoblastoma, osteoid osteoma
• WOVEN bone & spindle cell
stroma
– Bone differentiates reactive
from neoplastic
– Stromal cells differentiate
benign from malignant
• Woven bone: reactive (OB
rimming) vs neoplastic (no OB
rimming)
• Malignant stroma: hypercellularity,
atypia, pleomorphism, high mitotic
rate, etc.
Osteoid Osteoma
• Most common first two decades
• Classic pattern of constant pain
relieved by ASA/NSAIDs
• Proximal femur is the most common
location followed by the tibia,
posterior elements of the spine, and
the humerus
• Osteoid Osteoma is found in the
diaphysis or the metaphysis of the
proximal end of the bone more
often than the distal end
• Small lytic nidus with target
appearance
• Four diagnostic features include (1)
a sharp round or oval lesion that is
(2) less than 2 cm in diameter, (3)
has a homogeneous dense center
and (4) a 1-2 mm peripheral • CT scan as preferred method of
radiolucent zone evaluation
Osteoid Osteoma
Osteoid Osteoma
• Histology: Osteoid
trabeculae in loose,
vascular, stromal
connective tissue
• Rx:
– NSAIDs x 2 yrs (50%
successful)
– CT-guided radiofrequency
ablation (95% successful)
Osteoid Osteoma
• Affects young (<30 yo old), lower
extremity, pain gets better with NSAIDS
• Imaging shows a central nidus with
reactive bone, hot on bone scan, always
<1-1.5cm.
• Treatment: RF ablation
• Histology: woven bone, sharp border
• *most common benign bone tumor of
carpal bones
Osteoblastoma
• ~80% of these tumors occur in
patients younger than 30 years
• ~40% located in the spine--
usually involve the posterior
elements, and 17% of spinal
osteoblastomas in the sacrum
• Osteoblastoma of the long tubular
bones (LE>>UE) is often
diaphyseal
• On x-ray--appear as a radio-
lucent defect with a central
density due to ossification. The
lesion is well circumscribed and
may have a surrounding sclerosis
• Classic: calcified lesion in
posterior elements of the spine
Osteoblastoma
• Histology:
Osteoblastic rimming
of trabeculae
• Rx: excision with at
least a Marginal
Margin because
recurrence rate 20%
(40%+ w/curettage)
Osteoid Osteoma vs. Osteoblastoma
• Common • Uncommon
• Usually < 1 cm • Usually > 2 cm
• Regular tissue pattern • Irregular tissue
• Pain pattern
constant/nocturnal
• Pain sporadic
• Pain often relieved
with NSAIDs • Pain not relieved with
• Axial involvement NSAIDs
uncommon • Axial
involvement
common
Bone Island or Enostosis
• Focus of mature cortical bone within the
cancellous bone
• Cold on bone scan
Osteosarcoma
• Osteosarcoma is very rare in young children

• Incidence increases steadily with age, increasing more


dramatically in adolescence, corresponding with the
growth spurt

• Typically occurs in first 3 decades with second peak


beyond sixth decade
Osteosarcoma
• most commonly occurs in the extremities of long bones near
metaphyseal growth plates
• most common sites are the femur (42%, 75% of which are distal
femur), tibia (19%, 80% of which are proximal tibia), and humerus
(10%, 90% of which are proximal humerus)
• other significant locations are the skull and jaw (8%) and pelvis (8%)
Osteosarcoma

• most commonly occurs in the


extremities of long bones near
metaphyseal growth plate
• OCCURS AT MOST
METABOLICALLY ACTIVE
SITES
• The most common sites are
the femur (41%, 75% of which
are distal femur), tibia (19%,
80% of which are proximal
tibia), and humerus (10%,
90% of which are proximal
humerus).
• Other significant locations are
the skull and jaw (8%) and Most in the Distal Femur!!!
pelvis (8%) Second Most in Proximal Tibia!!!
Osteosarcoma
• most common presenting
symptom is pain, particularly
pain with activity
• lesions can be purely osteolytic
(~30% of cases), purely
osteoblastic (~45% of cases), or
a mixture of both
• On xray, permeative
metaphyseal lesions with soft
tissue extension and new bone
formation
• Periosteal reaction is common
and frequently takes on a
“sunburst” appearance
• MRI of the primary lesion is the
best method to assess the
extent of intramedullary disease
as well as associated soft tissue
masses and skip lesions
Osteosarcoma
• Histology: spindle–
shaped tumor cells that
produce osteoid
• Rx:Neo chemorestage
resection
Maintenance chemo =
60-70% long term
survival with localized
OSA
• Surgery alone = 10-20%
pt survival
• Increased risk OSA:
missing Rb gene (Ch13),
Paget’s, prior XRT
Osteosarcoma Sub-types
• Conventional (~90%)-
usually begins in medullary
canal
• Parosteal (~5%)-low grade
“lobulated” attachment to
cortex/75% at posterior
aspect of distal femur
• Telengiectatic (~4%)-purely
lytic, “bag of blood”,
confused with ABC but has
spindle cell
• Periosteal (2%)-juxtacortical,
diaphyseal, “sunburst,”
creates “crater”
• Others: well-diff, IM, small
cell, multicentric
Osteosarcoma Sub-Types
• High- grade intramedullary
• Telangiectatic
• Parosteal
• Periosteal
High-grade intramedullary
• Most Common type of OS
• 75% present as IIB, and the other 25%
present as mets (lung)
• 50% Occur around the knee
• *image entire bone to look for skip mets
(equivalent to mets).
• Hot on Bone scan, Histology is pink osteoid.
• Tx-Chemo, surgery, chemo
• Percentage of tumor necrosis in response to
chemo is main prognostic factor.
Telangiectatic Osteosarcoma
• Looks similar to ABC – Watch out
• Lytic with no mineralization
• * if located on ulnar side of DR= OS
• * if located on radial side of DR = GCT
Parosteal Osteosarcoma
• Low grade surface lesion- “stuck on bone”
• 80% distal posterior femur, painless mass
• Often confused with Fibrous dysplasia by
pathologist
• Bland spindle cells around the bone
• Treatment- wide surgical excision- NO
CHEMO.
Periosteal Osteosarcoma
• Rare, diaphyseal location, second decade,
sunburst pattern , chondroblastic
• Treatment: chemo, surgery, chemo

• Prognosis is worse than parosteal but not


as bad as high-grade intramedullary.
Summary – Osteoblastoma &
Osteosarcoma
Osteoblastoma
• Big brother to osteoid osteoma, pain NOT
relieved by NSAIDS.
• 2-6cm in size.
• Posterior elements of spine most common
location.
• Histology: woven bone with distinct
borders.
Osteosarcoma
• Most common primary sarcoma of bone
• Affects young patients
• Associated with retinoblastoma
• Most commonly presents in the 2nd and 3rd
decade of life (also in older adults with
Pagets)
• Best predictor of survival is the stage of
the disease.
• Most common presentation is Stage IIB
Other Bone-Forming
Conditions
Other Bone-Forming
Conditions
• Blastic Metastases:
permeative lesion with
infrequent soft tissue
extension
– 30/60/90 rule
renal/breast/prostate
• Paget’s: early lyticlate
blastic (coarse
trabeculae and bony
enlargement—mosaic
pattern)
– OITE Pearls:
• Assoc w/Paramyxovirus
• increased cranial
diameter (hats don’t fit)
• deafness (2/2 nerve
compression)
Fibrous Dysplasia
• Peak age of diagnosis is 5 - 20
years
– Two thirds of patients with
polyostostic disease are
symptomatic before the age of 10
– With monostotic disease, patients as
old as 20 - 30 years are
asymptomatic
• ~70-80% are monostotic
– This form most frequently occurs in
the rib (28%), femur (23%), tibia or
craniofacial bones (10-25%),
humerus, and vertebrae, in
decreasing order of frequency
• ~20-30% of fibrous dysplasias
are polyostotic
– This form more frequently involves
the skull and facial bones, pelvis,
spine, and shoulder girdle
– The sites of involvement are the
femur (91%), tibia (81%), pelvis
(78%), ribs, skull and facial bones
(50%), upper extremities, lumbar
spine, clavicle, and cervical spine, in
decreasing order of frequency
Fibrous Dysplasia
• Xrays: lucent lesion in
the diaphysis or
metaphysis, with
endosteal scalloping
and with or without
bone expansion and the
absence of periosteal
reaction
– Usually, the matrix is
smooth and relatively
homogeneous;
classically, described as
“ground-glass”
appearance
– Classic “Shepherd’s
crook” deformity
Fibrous Dysplasia
• Histology: irregular foci of woven bone
arising from a cellular fibrous blue
background
– irregular trabeculae have been
described as “Chinese letters” or
“alphabet soup”

• Rx: Observation /benign/usually


disappears
– If >75% cortex or
painfulcurettage/grafting
– rigid, intramedullary fixation with the
strongest possible device (a steel or
titanium cephalomedullary nail) is
the best method for treatment of
proximal femoral lesions

• McCune-Albright Syndrome:
polyostotic, café au lait spots,
precocious puberty, hyperthyroid Mutation is alpha subunit of a G protein
Myositis Ossificans
• Juxtaposed to bone, and not connected to
cortex
• Can be confused with synovial cell
sarcoma
• Can be confused with parosteal OS
• MO ossifies from periphery inward
Bone-Forming Lesions
Histology Summary
• Reactive lesion: woven bone with osteoblastic
rimming and a benign spindle cell stroma (frx
callus, myositis ossificans)
• Benign or low-grade bone forming neoplasm:
woven bone bone with no osteoblastic rimming
and bland-appearing spindle cell sroma (osteoid
osteoma, osteoblastoma, fibrous dysplasia)
• Malignant bone tumor: woven bone with no
osteoblastic rimming associated with malignant
spindle cell stroma (osteosarcoma)
Bone-Forming Lesions
OITE questions
OITE QUESTION
85. A 15 yo cross country runner reports shin pain that
is present during running, at rest, and at night.
The pain is relieved by ibuprofen. Radiographs, a
bone scan, and a CT scan are shown in Figures
32a through 32c. What is the most likely
diagnosis?
1. Cortical desmoid
2. Osteoblastoma
3. Stress fracture
4. Enchondroma
5. Osteoid osteoma
PINK = BONE
OITE QUESTION
85. A 15 yo cross country runner reports shin pain that
is present during running, at rest, and at night.
The pain is relieved by ibuprofen. Radiographs, a
bone scan, and a CT scan are shown in Figures
32a through 32c. What is the most likely
diagnosis?
1. Cortical desmoid
2. Osteoblastoma
3. Stress fracture
4. Enchondroma
5. Osteoid osteoma
OITE QUESTION
Osteosarcoma most commonly develops in which of the following
locations?

1- Pelvis
2- Distal humerus
3- Proximal tibia
4- Proximal femur
5- Proximal humerus
OITE QUESTION

> > >

OCCURS AT MOST METABOLICALLY ACTIVE SITES


OITE QUESTION
Osteosarcoma most commonly develops in which of the following
locations?

1- Pelvis
2- Distal humerus
3- Proximal tibia
4- Proximal femur
5- Proximal humerus

Preferred Response: 3
Recommended Reading(s):
Menendez LR (ed): Orthopaedic Knowledge Update: Musculoskeletal Tumors. Rosemont,
IL, American Academy of Orthopaedic Surgeons, 2002, pp 175-186.
Mankin HJ, Hornicek FJ, Rosenberg AE, et al: Survival data for 648 patients with
osteosarcoma treated at one institution. Clin Orthop Relat Res 2004;429:286-291
OITE QUESTION

A 9-year-old boy is seen for bilateral thigh pain. He has a history of


precocious puberty. Examination reveals multiple café-au-lait spots. AP
pelvis and frog lateral hip radiographs are shown in Figures 21a and 21b.
His condition is linked to an abnormality in
1- G protein function.
2- osteoclastic function.
3- vitamin D metabolism.
4- sulfate transporter gene.
5- type I collagen formation.
OITE QUESTION

A 9-year-old boy is seen for bilateral thigh pain. He has a history of


precocious puberty. Examination reveals multiple café-au-lait spots. AP
pelvis and frog lateral hip radiographs are shown in Figures 21a and 21b.
His condition is linked to an abnormality in
1- G protein function.
2- osteoclastic function.
3- vitamin D metabolism.
4- sulfate transporter gene.
5- type I collagen formation. Recommended Reading(s):
DiCaprio MR, Enneking WF: Fibrous dysplasia:
Pathophysiology, evaluation, and
treatment. J Bone Joint Surg Am 2005;87:1848-1864.
Parekh SG, Donthineni-Rao R, Ricchetti E, et al: Fibrous
dysplasia. J Am Acad Orthop
Surg 2004;12:305-313.
OITE QUESTION
Which of the following diseases of bone (when
nonmetastatic at diagnosis) carries the worst prognosis
for 5-year survival)?

• 1- Lymphoma
• 2- Osteosarcoma
• 3- Ewing’s sarcoma
• 4- Paget’s sarcoma
• 5- Conventional chondrosarcoma
OITE QUESTION
Which of the following diseases of bone (when
nonmetastatic at diagnosis) carries the worst prognosis
for 5-year survival)?

• 1- Lymphoma
• 2- Osteosarcoma
• 3- Ewing’s sarcoma
• 4- Paget’s sarcoma
• 5- Conventional chondrosarcoma
Cartilage-forming Lesions
Radiographic Differential
Diagnosis
Bone Forming Tumors Cartilage Forming Tumors "Third List"

Osteoid Osteoma Osteochondroma Infection


Osteoblastoma Chondromyxoid fibroma Metastases
Osteosarcoma Chondroblastoma Round cell tumors
Blastic Metastases Enchondroma Fibrous Dysplasia
Paget's Disease Chondrosarcoma Non-Ossifying Fibroma
(Fibrous Dysplasia) Simple Bone Cyst
Aneurysmal Bone Cyst
Histiocytosis
Giant cell tumor
Metabolic condition
Cartilage-Producing Lesions
• Enchondroma
• Periosteal Chondroma
• Osteochondroma
• Multiple Hereditary Osteochondroma
• Chondromyxoid Fibroma
• Chondrosarcoma
• Clear Cell Chondrosarcoma
• Differentiated Chondrosarcoma
Cartilage Forming Tumors
• Normal Cartilage: sparsely
cellular, one cell per lacuna, one
pyknotic nucleus per cell, well
formed matrix
• 3 tumor patterns:
– Benign (enchondroma)
merging into low grade which
merges into intermediate then
high grade
– Chondroblastoma
• Cobblestone
chondroblasts and
intervening chicken wire
calcification
– Chondromyxoid fibroma
• Benign spindle cell lesion
with some areas of
immature cartilage
Enchondroma
• Benign, metaphyseal, 60% are in the hand
• Most common primary lesion in the hand
• Imaging- stippled rings and arches, “popcorn”
calcifications, “blue balls” of cartilage
• Compared to “smoke up the chimney” for
bone infarct
• Ollier’s disease- multiple lesions/ 30%
malignant transformation
• Mafucci disease- multiple bone lesions and
hemangiomas- 100% malignant
Enchondroma
• peak incidence 20-40 yo
(Ollier’s 0-10 yo)
• usually found in the short
tubular bones of the hands and
feet
– most common primary
tumor in the hand and is
normally found in the
diaphysis
• NOT common in axial skeleton
(if so, think chondrosarcoma)
• Xrays: lucent area in medullary
canal (may see endosteal
scalloping) with variable
mineralization (rings, stipples,
arcs)”popcorn”
Enchondroma
• Multiple enchondromas may occur in 3
distinct disorders:
– Ollier disease: nonhereditary disorder
characterized by multiple
enchondromas with a predilection for
unilateral distribution (30% develop low
grade chondrosarcoma)
– Maffucci syndrome: nonhereditary,
less common than Ollier disease
(100% develop chondrosarcoma)
• +multiple hemangiomas
• +other malignancies – follow with
PET scan
– Metachondromatosis: multiple
enchondromas and osteochondromas
• inherited by autosomal dominant
transmission
Enchondroma
Enchondroma
• Histology: mineralized
hyaline cartilage
– May see bone encased
lobules separated by
marrow
• Rx: solitary painless
enchondromas may be
observed
– Painful or worrisome lesions
should be treated with
biopsy followed by
intralesional
resection/curettage
– Large defects can be filled
with bone graft
– All specimens must be
analyzed carefully for
malignancy.
Periosteal Chondroma
• Rare
• Occurs in 10-20 year olds
• Presents under the periosteum
• 50% at the proximal humerus
Osteochondroma
• 35% of all benign lesions, very common
• Does not grow in adulthood
• Key feature is continuity with the medullary
canal and with the cortex
• Sessile, broad based and points away
from the joint
• Cartilage cap is less than 2cm, if bigger-
be concerned for chondrosarcoma
transformation 1%.
Osteochondroma
• Histology: endochondral
ossification on the basal
surface of hyaline
cartilage so it resembles
a normal growth plate
with rows of
chondrocytes
– cartilage more
disorganized than normal,
has binucleate
chondrocytes in lacunae,
and is covered with a thin
layer of periosteum
• Rx: Asymptomatic =
Observation
MHE- Multiple Hereditary Exostosis
• A subset of osteochondromas
• Most frequent lesion to have secondary
chondrosarcomas, especially if they grow
after skeletal maturity.
• Autosomal Dominant
• EXT1, EXT2, EXT3
Osteochondroma
• most common benign bone tumor
• most commonly present in the
second decade of life
• grow until skeletal maturity and then
stop once the growth plates fuse
• most commonly found around the
knee and the proximal humerus
• Can be pedunculated or sessile
• Hallmarks: Grow away from the
physis and have medullary
continuity
• Secondary sarcomatous
degeneration should be suspected if
grows after puberty or has cartilage
cap >3 cm in adulthood
• OITE pearl: Osteochondromatosis
(HMOCE - Hereditary Multiple
Osteo- Cartilagenous Exostoses)
– AD Ch8 (EXT1), Ch11 (EXT2)
– risk of malignant transformation
to chondrosarcoma may be 25-
30% compared to
approximately 1% for a solitary
osteochondromas
OITE QUESTION
Patients with multiple hereditary osteochondromas frequently have
loss of forearm rotation and ulnar shortening. What procedure is
most likely to improve forearm rotation in these
patients?
1- Tendon transfer
2- Ulnar lengthening
3- Radial head excision
4- Distal ulnar resection
5- Simple excision of the osteochondroma
OITE QUESTION
OITE QUESTION
Patients with multiple hereditary osteochondromas frequently have
loss of forearm rotation and ulnar shortening. What procedure is
most likely to improve forearm rotation in these
patients?
1- Tendon transfer
2- Ulnar lengthening
3- Radial head excision
4- Distal ulnar resection
5- Simple excision of the osteochondroma

Shin EK, Jones NF, Lawrence JF: Treatment of multiple hereditary osteochondromas of the
forearm in children: A study of surgical procedures. J Bone Joint Surg Br 2006;88:255-260.
Akita S, Murase T, Yonenobu K, et al: Long-term results of surgery for forearm deformities
in patients with multiple cartilaginous exostoses. J Bone Joint Surg Am 2007;89:1993-
1999.
Chondromyxoid Fibroma
• Occurs in 20-30 year olds in proximal tibia
or foot, metaphysis.
• Eccentric, well-demarcated lesion, 25%
recur
• Treatment: Curretage and bone graft
• Histology: stellate appearing cells, “soap
bubbles” or snot
Chondromyxoid fibroma
• Extremely rare
• Presents 2nd-3rd decades
• found most often in the
metaphysis around the knee in
the proximal tibia, proximal
fibula, or distal femur
• Xray: eccentrically placed Iytic
lesion with well defined margins
in the metaphysis of the lower
extremity
• Histology: chondrocytes in
myxoid matrix separated by
fibrous bands
• Rx: Extended curettage/Graft
~10% recurrence
– En bloc excision as
complete cure
Chondroblastoma
• Epiphyseal lesion of the young about the
knee, proximal humerus, and proximal femur
• Painful-often abut the joint, and can cross
the physis
• 2-5% metastasis to the lungs
• Calcaneus lesion = Codman’s tumor
• Histology: Chicken-wire calcifications;
cobblestone appearance
• Treatment: Intralesional curretage and bone
graft
Chondroblastoma
• ~90% occur in those aged 5-25
years
• Most common epiphyseal lesion
in children behind infection
• typically occurs in the epiphysis
of a long bone
• most common site is the lower
extremity (72% of cases), and
50% of tumors occur around the
knee
– The femur is involved in
33% of cases; the humerus,
in 20%; and the tibia, in
18%
• symptoms are nonspecific and
include joint pain, tenderness,
swelling
– Joint effusion occurs in
approximately 30% of
patients
– Get an xray before you tap
a joint!
Chondroblastoma
• Xray: radiolucent with well
defined margins
– fine calcifications, either
punctate or in rings, may be
visible
• Histology: densely packed
polygonal Blue chondroblasts
with “Fried Egg” appearance and
“chicken wire” calcification
• Rx: biopsy and curettage with +/-
adjuvant liquid nitrogen or phenol,
or a mechanical burr
– ~20% recurrence rate
• 2% mets to lungs
• Classic: painful lucent lesion in
BLUE = Cartilage the epiphysis of a child
Chondrosarcoma
• Treatment is surgery!
• Occurs in patients >50 yrs old
• Most common location is the pelvis, ribs, and
proximal femur
• Axial and proximal skeletal lesions are more
aggressive
• Histologic grade correlates with mets:
• Grade 1- 5%
• Grade 2-20%
• Grade 3- 60%
• Dedifferentiated- almost 0% survival
Chondrosarcoma
• 2nd most frequent primary
malignant tumor of bone,
representing approximately 25% of
all primary osseous neoplasms
• Usually occur in patients older than
40 years
• THINK axial skeleton: most
commonly involve the pelvic bones,
femur, humerus, ribs, scapula,
sternum, or spine
– In tubular bones, the
metaphysis is the most
common site of origin
• most common symptom at
presentation is pain, often present
for months and typically dull in
character
– may be worse at night
– average duration of symptoms
prior to presentation is 1-2
years
Chondrosarcoma
• Xrays: fusiform, lucent defect
with scalloping of the inner
cortex and periosteal reaction
– Extension into the soft tissue
may be present as well as
punctate or stippled calcification
of the cartilage matrix
• CT helpful in defining the
integrity of the cortex and
distribution of calcification
• MRI invaluable in surgical
planning as it demonstrates the
intraosseus and soft tissue
involvement of the tumor
– also helpful in evaluating
possible malignant
degeneration of
osteochondromas by allowing
accurate measurements of the
cartilage cap which should be
less than 2 cm thick
Dedifferentiated
Chondrosarcoma
• Most Malignant cartilage lesion
• 50% have a pathologic fracture due to
tumor’s aggressive and very destructive
nature.
• Treatment- aggressive- chemo, surgery,
chemo
• 13% 5 year survival rate
Chondrosarcoma
• Categorized by grade:
– Grade 1 (Low grade)-low
cellularity, abundant
ECM, small dark nuclei
(5% mets)
– Grade 2 (Low grade)-
increased cellularity [50%
chondrosarc] (20% mets)
– Grade 3 (High grade/
most aggressive)-high
cellularity, mitotic figures,
etc. (70% mets)
• 75% are Low grade
• Most classified as
conventional, but other
subgroups are clear cell,
myxoid, mesenchymal, and
dedifferentiated
Chondrosarcoma
• Centralin Medullary cavity
• Surface
– Seen in 20-40 yo group most often
in pelvis/prox femur
– “explosive” lesion of stippling that is
NOT circumscribed

• Dedifferentiated
– mix of low grade chondrosarcoma
and high grade spindle cell sarcoma
where the spindle cells are no
longer identifiable as having a
cartilage origin • Mesenchymal
– biphasic quality is on x-ray with predilection for the spine, ribs and jaw
areas of endosteal and it presents in the third decade
scalloping/cortical thickening rare variant with a bimorphic histologic
contrasted with areas of cortical picture of low grade cartilaginous cells
destruction and hypercellular small, uniform, and
– Most malignant—5 yr surv 10% undifferentiated cells that resemble
Ewing's sarcoma

• Clear cell
Chondrosarcoma
• Clear cell
– adult variant of
chondroblastoma
– found in the epiphysis of the
femur and humerus
• Classic—head of femur
– Histology: clear cells with
vacuolated cytoplasm
• matrix has significantly
calcified trabeculae and
giant cells
– rare, low-grade tumor with an
improved prognosis over other
chondrosarcomas
• OITE pearls: Epiphyseal lesions
= chondroblastoma (pedi), giant
cell and clear cell (adult)
Chondrosarcoma
• Treatment =Wide surgical
excision
• XRT and Chemo
resistant
• Biopsies must be planned
with future tumor excision
in mind
• Patients with adequately
resected low grade
chondrosarcomas have
an excellent survival rate
• The survival of patients
with high grade tumors
depends on the location,
size and stage of the
tumor
Cartilage-Forming Lesions
OITE Questions
OITE QUESTION

A 12-year-old patient has an epiphyseal lesion. What is the


most likely diagnosis?
1- Giant cell tumor
2- Aneurysmal bone cyst
3- Eosinophilic granuloma
4- Dysplasia epiphysealis hemimelica
5- Chondroblastoma
OITE QUESTION

A 12-year-old patient has an epiphyseal lesion. What is the


most likely diagnosis?
1- Giant cell tumor
2- Aneurysmal bone cyst
3- Eosinophilic granuloma
4- Dysplasia epiphysealis hemimelica
5- Chondroblastoma
OITE QUESTION
• Which of the following conditions shown in
these pictures has the highest rate of
malignant change?

1. Ollier’s disease
2. Enchondromatosis
3. Maffucci’s syndrome
4. Multiple exostoses
5. Solitary osteochondroma
OITE QUESTION
• Which of the following conditions shown in
these pictures A-D has the highest rate of
malignant change?

1. Ollier’s disease
2. Enchondromatosis
3. Maffucci’s syndrome
4. Multiple exostoses
5. Solitary osteochondroma
OITE QUESTION
23. An 80-year-old woman reports the sudden development
of pain in the left distal thigh. She denies any history of
trauma. Figures 9a through 9d show radiographs, a bone
scan, and a biopsy specimen. What is the likely
diagnosis?
1- Enchondroma
2- Osteosarcoma
3- Dedifferentiated chondrosarcoma
4- Metastatic breast carcinoma
5- Paget’s disease
OITE 2006 – Ortho Diseases
OITE QUESTION
23. An 80-year-old woman reports the sudden development
of pain in the left distal thigh. She denies any history of
trauma. Figures 9a through 9d show radiographs, a bone
scan, and a biopsy specimen. What is the likely
diagnosis?
1- Enchondroma
2- Osteosarcoma
3- Dedifferentiated chondrosarcoma
4- Metastatic breast carcinoma
5- Paget’s disease
Radiographic Differential
Diagnosis
Bone Forming Tumors Cartilage Forming Tumors "Third List"

Osteoid Osteoma Osteochondroma Infection


Osteoblastoma Chondromyxoid fibroma Metastases
Osteosarcoma Chondroblastoma Round cell tumors
Blastic Metastases Enchondroma Fibrous Dysplasia
Paget's Disease Chondrosarcoma Non-Ossifying Fibroma
(Fibrous Dysplasia) Simple Bone Cyst
Aneurysmal Bone Cyst
Histiocytosis
Giant cell tumor
Metabolic condition
“Third List” Tumors
• Fibrous Dysplasia (COVERED IN BONE PRODUCING LESIONS)
• Non-ossifying fibroma
• Osteofibrous dysplasia
• Aneurysmal Bone cyst
• Unicameral Bone cyst
• Myeloma
• Lymphoma
• Ewing’s Sarcoma
• Neuroblastoma
• Histiocytosis
• Giant Cell Tumor
Fibrous Dysplasia
• Developmental abnormality that can be
mono- or polyostotic
• Café au lait spots- Coast of Maine (more
rugged)
• Coast of California (smoother) —
neurofibromatosis
• Associated with Gs-alpha Protein
• Common location is proximal femur-
shepard’s crook.
• X-ray has ground glass appearance
Fibrous Dysplasia
• Peak age of diagnosis is 5 to 20
years
• sites of involvement are the
femur (91%), tibia (81%), pelvis
(78%), ribs, skull and facial
bones (50%), upper extremities,
lumbar spine, clavicle, and
cervical spine, in decreasing
order of frequency
• Xrays: lucent lesion in the
diaphysis or metaphysis, with
endosteal scalloping and with or
without bone expansion and the
absence of periosteal reaction
– “ground glass” appearance
• Rx: Observation
b/benign/usually disappears
– If >75% cortex or
painfulcurettage/grafting
LEARN FROM THE MISTAKES OF
OTHERS IN FIBROUS DYSPLASIA
• Do NOT use autograft
• Do NOT use cancellous
allograft
• Do NOT try to remove all of the
disease
• Do NOT allow the deformity to
be come severe
• Do NOT use plates and screws
if possible
• Do NOT expect a single
operation to solve the problem
Stanton, Robert P. J Bone & Mineral Res, 2006; 21:p105-109.
Osteofibrous Dysplasia
• A.k.a ossifying fibroma
• most common site in adults
is the mandible
• most common site in
children is the tibia, followed
by other long bones
• occurs during the first
decade of life and presents
clinically as a painless,
enlarging mass
• XRAY: multiple eccentric,
well-circumscribed lucent
lesions in anterior cortex of
tibia
• Histology: irregular spicules
of trabecular bone lined by
osteoblasts in fibroblastic
background
– stains positive for
cytokeratin
Osteofibrous Dysplasia
• Treatment
– Older patients :
• conservative management
• Curettage

– Pts w/open growth plates


• observe
• if bowing of the tibia becomes an issue, bracing is
recommended
Fibrous Dysplasia vs Osteofibrous
Dysplasia

Fibrous Dysplasia Osteofibrous Dysplasia –


less well circumscribed bone
surrounded by osteoblasts
Non-ossifying fibroma
(fibrous cortical defect)
• Xrays: eccentric, multi-
loculated sub-cortical lesions
with a central lucency and a
scalloped sclerotic margin
• Histology: highly cellular
(including giant cells &
lipophages) with storiform
fibrogenic areas
• Rx: observation (most regress
spontaneously)
– only definite indication to
treat is a pathologic
fracture
• *Pearl - Jaffe-Campanacci
syndrome: multiple NOF, cafe-
au-lait spots, mental
retardation, hypogonadism,
ocular/cardio abnormalities.
Non-ossifying fibroma
(fibrous cortical defect)
• found mostly in children with
75% occurring in the second
decade
• non-neoplastic process that
occurs in the juxta-epiphyseal
region of the long bones
– ~90% involve the tubular
long bones
– Common sites include the
femur (most commonly the
distal femoral metaphysis
38%), the proximal and
distal tibia (43%)
– most lesions occur around
the knee
• Classic scenario: child who
has a minor sports injury and a
plain xray shows lesion
Round cell tumors
myeloma, lymphoma, metastatic dz, small cell
osteosarcoma, Ewing’s sarcoma, infection

• <5 yo then think Neuroblastoma


• 10-40 yo think of the destructive group
(OSA, Ewing’s, infection)
• >40 yo think Metastatic Dz, Myeloma,
Lymphoma
– Note that infection can be seen in any age
group
Ewing’s Sarcoma
• Malignant round cell sarcoma affecting young
patients 2-25.
• Commonly Diaphyseal
• <5 yo, consider leukemia or metastatic
neuroblastoma.
• >30 yo, consider metastatic carcinoma or
lymphoma
• Third most common primary sarcoma of bone
1. osteosarcoma
2. Chondrosarcoma
3. Ewings
Ewing’s Sarcoma
• A type of primitive peripheral
neuroectodermal tumor (PNET)
• Most common in the 1st and
2nd decades
• Preferentially affects whites
more than blacks and Asians
• 3:2 male to female ratio
• Found in the lower extremity
more than the upper extremity,
but any long tubular bone may
be affected
– most common sites are the
metaphysis and diaphysis of
the femur followed by the
tibia and humerus
• Clinical presentation includes
pain and swelling of weeks or
months duration Osteomyelitis is often the initial
– Erythema and warmth of diagnosis based on fevers,
the local area are
sometimes seen leukocytosis, anemia and an
increased ESR
Ewing’s Sarcoma
• Xrays:
often a central lytic lesion
associated with a
lamellated or "onion
skin" periosteal reaction
(caused by and splitting
and thickening of the
cortex by tumor cells)

– "onion-skin"
appearance is often
followed with a "moth-
eaten" or mottled
appearance and
extension into soft
tissue
Ewing’s Sarcoma
Imaging:

• CT is helpful in defining bone destruction

• MRI is essential to elucidate the soft tissue involvement


– T1-weighted images low intensity compared to the normal
high intensity of bone marrow. On T2 -weighted images
the tumor is hyper intense compared to muscle

• Increased uptake on bone scan


Ewing’s Sarcoma
• Histology: densely packed uniform small
round blue cells in sheets (+CD99)
– Glycogen is present within the cells
causing (+) reaction to periodic acid-
schiff (PAS) stain
– Most are positive with HBA-71 or 0-
13 stain which is an antibody to the
product of myc
– distinguished from metastatic
neuroblastoma by reticulin stain and
urine vanillyl mandelic acid and
homovanillic acid
– Rhabdomyosarcoma is ruled out if
the specimen stains negatively with
desmin, myoglobin and actin stains
– neural origin is supported by
electron microscope findings of
pseudorosettes
– This is further supported by the
common finding in Ewing's sarcoma
and primitive neuroectodermal
tumors of choline acetyltransferase
and the translocation t(11:22)
Ewing’s Sarcoma
• Treatment
– includes surgery, radiation and multi-drug
chemotherapy
– XRT or chemo used preoperatively
– Adjuvant chemotherapy follows surgery and
decreases recurrences

• Poor prognostic signs:


– increased age
– increased ESR at presentation
– leukocytosis at presentation

• Distinguish from metastatic neuroblastoma by keratin


stain
Neuroblastoma
• The second most common solid
malignancy of childhood (after brain
tumors)
– responsible for ~ 10% of all pediatric
neoplasms (and up to 1/2 of
malignancies diagnosed in infancy)
• predominantly a tumor of early childhood
(2/3 dx’d when < 5 yo)
• often begins in the nerve tissue of the
adrenal glands
• Xrays: permeative lesion with medullary
bone replacement and variable lysis
• Histology: densely cellular sheets of small
cells with dark nuclei and scant
cytoplasm that have an overall primitive
appearance
– Rosettes (Homer Wright
pseudorosettes) are often found - in
them, tumor cells are arranged in
circles around a central mass of
neuropil
• The majority (about 90%) produce
catecholamines which are an important
diagnostic feature
– elevated blood levels of
catecholamines and elevated urine
levels of metabolites such as
vanillymandelic acid VMA and
homovanillic acid HVA are present
• Metastatic dz = dismal <5% 3 year
survival and need to be treated with
surgery, chemotherapy, and radiation
Osteomyelitis
• More common in young
• The most common organism in all
pts = Staph Remember Neonates-
group B strep/E coli, Sickle-
Salmonella, shoe Puncture-
Pseudomonas, after Surg-Staph
epi
• Hematogenous osteo occurs most
commonly in the distal femur,
proximal tibia, proximal femur and
proximal humerus
• May be caused by direct
innoculation secondary to trauma
or surgery
– 5% of Open frx and 5% of acute
OM become chronic OM
– Biopsy chronic cases—1%
chronic sinuses get SCC
(Marjolin’s ulcer)
Osteomyelitis
• Xrays: Radiolucent and
radiodense lesions usually in
metaphysis (may cross growth
plate into epiphysis)
– Chronicmottled bone,
periosteal rxn, bony deformation
• “Brodie’s Abscess” =
Subacute OM—well defined
lytic defect and mottled
sclerosis of adjacent
metaphysis
• Sequestrum (dead cortical
bone) and Involucrum (new
cortical bone b/c periosteal bld
supply external)
Osteomyelitis
• On MRI scan, T1 weighted images
demonstrate infection as a low
signal with ill defined margins. T2
images show infection as a bright
signal..
• Histology: 3 featuresmixed cell
population, new capillaries and
tissue separation (edema)
• Rx: Surgical sampling or needle
biopsy is necessary for diagnosis
– Infected hardware should be
removed if the bone is healed
and stable
– Acute osteomyelitis is treated
with irrigation and
debridement as
necessary, followed by four to
six weeks of antibiotics
– Chronic osteomyelitis is best
treated with thorough
debridement, antibiotics, and
local flap coverage if
necessary
Hematopoietic Bone Lesions
• Lymphoma
• Multiple myeloma
• Solitary plasmacytoma
• Osteosclerotic myeloma
Lymphoma
• Bony involvement of Hodgkin’s disease (HD) is
between 9.2%-30.3% of patients

• Primary Bone Involvement (primary lymphoma


of bone) <1% -- Osseous lesions are most
commonly due to hematogenous dissemination
in stage IV disease or due to direct invasion from
an adjacent lymph node leading to focal
sclerosis
Lymphoma
• Xrays: extensive lytic/blastic
changes with 25-50% pts
having CORITCAL
THICKENING (which is also
seen in Chondrosarcoma,
Pagets, and Chronic OM)
• presents in the 3rd or 4th
decades or later
• The sites of predilection are
long bones and primarily
diaphyseal affecting the
medulla of the bone
• Multifocal in older age group
• Bone scan is useful and MRI is
very sensitive in the
localization of subtle marrow
disease
– hypointense signal on T1
weighted spin echo
sequences and high signal
on T2 weighted gradient
echo sequences
Lymphoma
• Histology: Bone marrow taken
over by lymphocytes of all
shapes and sizes (usually
large non-cleaved B-cell)
• Often, a diagnosis of
“histologic exclusion”
– Given lytic lesion in pt
>40yo on histologic slide
see NO glands (mets), NO
plasma cells (MM), NO
cartilage (CS), and NO
storiform pattern (MFH)
• Rx: Pts w/ one or multiple
bone lesions usually respond
well to combined modality
treatment, including chemo
and local XRT
• Surgery should be considered
for lesions that present a risk
of pathological fracture.
Lymphoma
• Primary is usually Non-Hodgkins
lymphoma
• Mottled, blastic bone lesion
• VERY hot on bone scan
• Histology- mixed round cell inflitrate
• Treatment- radiation and chemotherapy
• Mets, MM, and Lymphoma usually do not
require surgery (except for structural
insufficiency)
Multiple Myeloma
• Plasma cell malignancy in patients >50yo.
• Plasma cells produce immunoglobulins
• Bone destruction by RANKL
• Bisphosphonates reduce the number of skeletal
events in patients with MM.
• Numerous Punched out lesions
• 30% of bone scans are cold: need to get skeletal
survey
• Serum Protein Electrophoresis (SPEP)- M spike-
Immunoglobulin G (50%). Immunoglobulin A (25%)
• Treatment is chemo and radiation
Multiple Myeloma
• Most common primary malignant
tumor
• Median age: 68 y for men/70 y for
women
• Involves spine, skull, ribs, sternum
and pelvis but may affect any
bone with hematopoietic red
marrow
• Normocytic, normochromic
anemia secondary to marrow
failure and an increased ESR
• SPEP—may see “goal post sign”
– Specifically, SPEP>75% sens in
dx while UPEP adds ~20%
• Single greatest prognostic factor is
Renal Function
• Median survival 2 yrs
Multiple Myeloma
• Xrays: diffuse osteopenia with
multiple “punched out”
radiolucent lesions w/no
periosteal reaction
• Bone scan can fail to have
increased uptake in 25% pts
suggesting a skeletal survey
should always be done
• Histology: Uniform population of
monoclonal plasma cells (>30%
BM)
• Rx: palliative chemo or BMT
– Only patients with complete
remission experience any bony
healing
– Bisphosphonates used to inhibit
bone resorption
– Surgery & XRT 2-3 wks later for
impending/path frx
Plasmacytoma
• A local disease of MM
• Treatment is radiation
• Better prognosis than MM.
Osteosclerotic Myeloma
• Poems syndrome
• Associated with neuropathy
Tumor Like Conditions of Bone
• Aneurysmal Bone Cyst
• Unicameral Bone Cyst (Simple Bone Cyst)
• Histiocytosis (Langerhans’ Cell
Histiocytosis
• Fibrous Dysplasia
Aneurysmal Bone Cyst
• Vertebrae and long bones are typical
location
• Lytic and expansile lesion in patients less
than 20 years old.
• Fluid-fluid lines on x-ray, better seen on
MRI
• Will see RBC in histology
• Treatment: Curretage and bone graft
• 25% recur
Aneurysmal Bone Cyst
• found most commonly during
the second decade
• most common location is the
metaphysis of the lower
extremity long bones (>than
the upper extremity)
– vertebral bodies also may
be involved
• 30% have identifiable pre-
existing lesion
• translocation involving the
16q22 and 17p13
chromosomes has been
identified in the solid variant
and extraosseous forms of
aneurysmal bone cyst
Aneurysmal Bone Cyst
• Present with PAIN (vs
UBC—fracture)
• Xrays: purely lytic, expansile,
eccentric metaphyseal lesion
that erodes cortex (“soap
bubble”)
– Fluid-fluid levels
• Histology: blood-filled
spaces with NO endothelial
lining
• Rx: biopsy to rule out
telengiectatic OSA then
extended curettage/grafting
– Recurrence statistically
related to young age and
open growth plates, and
may be less likely following
wide excision
Aneurysmal Bone Cyst
• Present with PAIN (vs
UBC—fracture)
• Xrays: purely lytic, expansile,
eccentric metaphyseal lesion
that erodes cortex
• Fluid-fluid levels on
CT/MR
• Histology: blood-filled
spaces with NO endothelial
lining
• Rx: biopsy to rule out
telengiectatic OSA then
extended curettage/grafting
– Recurrence statistically
related to young age and
open growth plates, and
may be less likely following
wide excision
– White space and blood
Aneurysmal Bone Cyst
• Common in second decade
• Most common location is the
metaphysis of the lower
extremity long bones (>than
the upper extremity)
– vertebral bodies also
may be involved
• 30% have identifiable pre-
existing lesion
• Translocation involving the
16q22 and 17p13
chromosomes has been
identified in the solid variant
and extraosseous forms of
aneurysmal bone cyst
• Telangiectatic
Osteosarcoma DDX 1/1000
Unicameral Bone Cyst
• Also known as Simple
Bone Cyst
• Most commonly found
in children between 5-
15 yo (85% pts < 20 yo)
• Most common site: prox
humerus (50%)
followed by prox femur
(25%)
• Xrays: radiolucent,
centered lesion with
thinned cortices and
metaphyseal expansion
to width of physis
– “fallen leaf” sign = fx
Unicameral Bone Cyst
• Usually asymptomatic and only
present when pathologic frx
occurs
• Histology: thin fibrous
membrane lines lumen
• Rx: aspiration confirmation
then steroid injection(s)
– If pathologic frxlet heal
first before injecting
– In lower ext, may consider
open curettage/grafting;
however, recurrence rates
reported up to 40%
• MRI all cysts to diagnose fluid
inside
How to tell UBC from ABC
• UBC • ABC
– Minimal pain unless frx – Pain even without frx
– No h/o trauma unless frx – May have had direct blow
– Fluid-filled cavity – Blood-filed cavity
– Slightly expansile – Very expansile
– Xray with few loculations – Xray with many loculations
– MRI with minimal fluid-fluid – MRI with prominent fluid-fluid
levels levels
– (+)membrane lining – No membrane lining
– White space (mesenchymal tissue lining)
on path – White space
& blood on path
Histiocytosis (Langerhans’ cell
Histiocytosis
• Continum- Eosinophilic granuloma—Hand-
Schuller-Christian Disease—Letterer-Siwe
disease.
Eosinophilic Granuloma
• Affects multiple bones and occuring in the
youth.
• Punched out lesion on x-ray
• Vertebra plana
• Treament low dose radiation, bone graft
with curretage, steroid injection, or
observation
Eosinophilic Granuloma
• part of a spectrum of Langerhan's
cell histiocytosis, formerly known as
histiocytosis X
• localized lesion in bone or lung and
occurs most commonly in children
aged 5 to 10
• makes up 60-80% of all cases of
Langerhan's cell histiocytosis
• uncommon in blacks
• found in the skull, mandible, spine
and long bones
• In spine, may cause vertebra plana
(Calve’s disease)
• male to female ratio is two to one
• Letterer-Siwe disease is a fulminant
systemic disease that comprises
10% of Langerhan's cell
histiocytosis, occurs in children
under 3 years old and is rapidly fatal
• Hand-Schuller-Christian disease
(HSC) is a chronic disseminated
form of Langerhan' s histiocytosis
– occurs in older patients
– triad of HSC is diabetes
insipidus, exopthalmos and
skull lesions
Eosinophilic Granuloma
• XRAY: well-circumscribed, central
radiolucent lesion in metaphysis or
diaphysis of long bone with
radiodense rim
– “hole within a hole” pattern
• Histology: sheets of Langerhan's cells,
multinucleated giant cells, and
eosinophils
– LC as Eosinophilic histiocytes with
coffee bean shape nuclei
– identifiable under EM: racket shaped
cytoplasmic inclusion
bodies called Birbeck's granules
– Confirmed by CD1a stain
• Rx: With localized disease, often a
biopsy alone is enough to incite
healing
– Other treatment modalities of EG
include curettage, excision,
steroid injection, radiation and
observation
– Chemotherapy is recommended
for systemic disease
McCune-Albright syndrome
• Precocious puberty
• Café au lait spots
• Poly-ostotic fibrous dysplasia
• Chinese letters histology
• Treatment- observe vs. internal fixation
• “Fibrous dysplasia heals with fibrous
dysplasia”
Malignant Fibrous Histiocytoma
• Most frequent soft tissue tumor in
adults
• Highest incidence in 5th decade
• Found in the extremities 75% of
the time (50% lower ext)
– In order: Distal femur, prox tibia,
prox femur, prox humerus
• #1 sarcoma to complicate a pre-
existing condition (Paget’s, bone
infarct, radiation, AVN, surgery,
NOF)
– 20% Secondary to another
condition
– These lesions are ususally more
aggressive (presenting as Stage
IIb sarcoma)
Malignant Fibrous Histiocytoma
• Clinically, p/w rapidly enlarging
mass
• XRAY: extensive, permeative
lesion, usually central in
metaphyseal or diaphyseal
region
– “mottled bone”
– Associated soft tissue mass
• CT scan is helpful in
determining cortical erosion
• MRI is a must: intermediate
T1/high T2
• Increased uptake on Bone
scan
Malignant Fibrous
Histiocytoma
• Histology: Malignant
multi-nucleated giant
cells with “storiform”
spindle cell fibrogenic
areas (create
“cartwheels”)
– Stains (+) for CD68 and
Lysosome
– 5 subtypes: pleomorphic-
storiform (most common
50-60%), myxoid, giant
cell, inflammatory,
angiomatoid
• Rx: wide resection and
chemo
Bone tumors of unknown origin
• Giant cell tumor of bone
• Ewing’s Sarcoma (COVERED IN “ROUND CELL TUMORS”)
• Adamantinoma
Giant Cell tumor of Bone
• Aggressive benign lesion in 20-40 yos,
especially distal radius and proximal tibia.
• If ulnar side of distal radius, this
osteosarcoma
• Imaging- well marginated lytic lesion,
subchondral, eccentric, closed physis.
• Histology: multi-nucleated giant cells
• Treatment- aggressive curretage with
bone graft, with a reccurence rate of 2-5%
Giant Cell Tumor
• XRAY: eccentric, expanding
zone of well-defined
radiolucency at ends of bones
(no reactive sclerosis)
• Histology: giant cells with nuclei
appearing similar to those of the
surrounding nuclei
• Rx: Intralesional excision by
"extended" curettage is the
treatment of choice
– Curettage alone is associated with a
high recurrence rate, and this can
be decreased with the addition of
chemical cautery using phenol,
multiple freeze-thaw cycles using
liquid nitrogen, and treating the walls
of the cavity with a high-speed
rotary burr
– Curettage alone is thought to lead to
recurrence in 50% of cases
– Recurrence after extended
curettage is ~10%
– The tumor cavity may be filled with
cement or bone graft, according to
the surgeon's preference
Giant Cell Tumor of Bone
• occur most often during the
third decade
• usually found in the long
bones, most often the distal
femur, proximal tibia, and
distal radius
• Whether tumor arises in the
epiphysis or distal metaphysis
is a matter of controversy, but
giant cell tumors only occur
after the epiphyseal plates
have closed
– A diagnosis of GCT in a
patient with open growth
plates should be questioned
• Pts p/w progressive pain
• 2% met to lungs (10-30% of
met pts die)
– Therefore, Chest CT for
GCT
Adamantinoma
• Tibia lesion in patients >20 years old, can
involve tibia in up to 50%.
• Precursor of Osteofibrous dysplasia
(Campanacci disease)
• Histology- fibrous tissue with epithelioid
tissue
• Treatment- wide en bloc excision, consider
intercalary tibia resection.
• 40% recur
• Lung mets 25%
Adamantinoma
• extremely rare, low-grade
malignant tumor of
epithelial origin
• 90% in the diaphysis of
the tibia
• usually occurs in the 2nd
to 5th decade of life
• 20% of cases have
metastases late in the
course of the disease
• May mimic osteofibrous
dysplasia on xray
Adamantinoma
• XRAY: multiple eccentric, well-
circumscribed lucent defects
with intervening sclerosis in
the tibia (“soap bubble”)
• Histology: biphasic--blue
epithelial nests of cells within
fibrous background
– Epithelial origin, +keratin
• Rx: wide surgical excision
– insensitive to radiation and
may metastasize to lungs,
lymph nodes and abdominal
organs by both hematogenous
and lymphatic routes
– Chemotherapy is not used
Chordoma
• rare malignant tumor that arises
from notochord remnants
• account for 1-4% of all bone
tumors
• occur in older adults with the
highest prevalence in the 5th to
7th decade
• the ratio of male to female is 2:1
• due to their origin in the
notochord, chordomas occur in
the mid-line of the axial skeleton
– 50% occur in the
sacrococcygeal region and
33% occur at the base of
the skull
– other rare sites include
transverse processes of
vertebrae and the paranasal
sinuses
• p/w low back pain,
bowel/bladder dysfunction,
headaches
Chordoma
• XRAY: solitary mid-line lesion with
bony destruction
– often have an accompanying
soft tissue mass and ~50%
have focal calcifications
• CT and MRI scans help
demonstrate the soft tissue
component, calcifications and
epidural extension
• Chordomas have reduced uptake
on bone scan
• Histology: lobules of “physaliferous”
cells in fibrous septa
– Stain (+) for cytokeratin, S-100,
vimentin, and epithelial
membrane antigen
• Rx: Wide surgical excision is
desirable but rarely feasible based
on the anatomic location of the
tumor
– sexual function and sphincter
control may be compromised
after surgery
– Radiation is used if complete
resection is impossible
– Chemotherapy can be used for
late stage disease
• Death from Local Complications
Testable facts s/p Sacrectomy
Bowel and Bladder Function After Sacral Resection (Todd et al., CORR 4/2002)

• Resection Spared Level Normal Bowel Normal Bladder


• Bilateral S2-S5 Both S1 0/10 (0%) 0/10 (0%)
• Bilateral S3-S5 Both S2 2/5 (40%) 3/12 (25%)
• Bilateral S4-S5 Both S3 4/4 (100%) 9/13 (69%)
• Variable Unilateral S3 2/3 (67%) 3/5 (60%)
• Unilateral S1-S5 Contralateral S1-S5 7/8 (87%) 8/9 (89%)
“Other” tumors/lesions
OITE Questions
OITE QUESTION
51. A 12 yo boy sustained a fracture in the distal ulna twice in the
past year. He denies any major fall or injury. A radiograph and a
biopsy speciman are shown in Figures 17a and 17b. What is the
most likely diagnosis?

1. Chondroblastoma
2. Giant Cell Tumor
3. Unicameral bone cyst
4. Fibrous dysplasia
5. Nonossifying fibroma
Empty White Space
OITE QUESTION
51. A 12 yo boy sustained a fracture in the distal ulna twice in the
past year. He denies any major fall or injury. A radiograph and a
biopsy speciman are shown in Figures 17a and 17b. What is the
most likely diagnosis?

1. Chondroblastoma
2. Giant Cell Tumor
3. Unicameral bone cyst
4. Fibrous dysplasia
5. Nonossifying fibroma
OITE QUESTION
A 35-year-old woman has increasing shoulder pain. A radiograph, sagittal
T2-weighted MRI scan, and CT scan are shown in Figures 236a through
236c. A biopsy specimen is shown in Figure 236d. What is the most likely
diagnosis?
1- Infection
2- Tuberculosis
3- Giant cell tumor
4- Aneurysmal bone cyst
5- Telangiectatic osteosarcoma
Blood and White Space
OITE QUESTION
A 35-year-old woman has increasing shoulder pain. A radiograph, sagittal
T2-weighted MRI scan, and CT scan are shown in Figures 236a through
236c. A biopsy specimen is shown in Figure 236d. What is the most likely
diagnosis?
1- Infection
2- Tuberculosis
3- Giant cell tumor
4- Aneurysmal bone cyst
5- Telangiectatic osteosarcoma

Preferred Response: 4
Recommended Reading(s):
Kaila R, Ropars M, Briggs TW, et al: Aneurysmal bone cyst of the pediatric shoulder girdle:
A case series and literature review. J Pediatr Orthop B 2007;16:429-436.
Schwartz HS (ed): Orthopaedic Knowledge Update: Musculoskeletal Tumors 2. Rosemont,
IL, American Academy of Orthopaedic Surgeons, 2007, pp 87-102
OITE QUESTION
An 8-year-old girl has had right elbow pain for the past 2 months.
Radiographs, MRI scans, and a biopsy specimen are seen in
Figures 250a through 250e. What is the most likely diagnosis?
1- Osteomyelitis
2- Chondroblastoma
3- Simple bone cyst
4- Eosinophilic granuloma
5- Aneurysmal bone cyst
Blood and White Space
OITE QUESTION
An 8-year-old girl has had right elbow pain for the past 2 months.
Radiographs, MRI scans, and a biopsy specimen are seen in
Figures 250a through 250e. What is the most likely diagnosis?
1- Osteomyelitis
2- Chondroblastoma
3- Simple bone cyst
4- Eosinophilic granuloma
5- Aneurysmal bone cyst
OITE QUESTION
242. A 55-year-old man has an isolated femoral
fracture following a motor vehicle accident. A
radiograph is shown in Figure 91. He is
hemodynamically stable and his pain is
adequately controlled. What is the next most
apporpriate step in management?

1 – antegrade intramedullary nailing of femur


2 – retrograde intramedullary nailing of the femur
3 – placement of a distal femoral traction pin
4 – wide resection
5 – biospy and staging
OITE QUESTION
242. A 55-year-old man has an isolated femoral
fracture following a motor vehicle accident. A
radiograph is shown in Figure 91. He is
hemodynamically stable and his pain is
adequately controlled. What is the next most
apporpriate step in management?

1 – antegrade intramedullary nailing of femur


2 – retrograde intramedullary nailing of the femur
3 – placement of a distal femoral traction pin
4 – wide resection
5 – biospy and staging
General Concepts review
Staging review
• Grade: Histology with aid of radiographic findings and clinical correlation
G 1: Low grade, uniform cell type without atypia, few mitoses
G 2: High grade, atypical nuclei, mitoses pronounced

• Site: T 1: Intracompartmental (Confined within limits of periosteum)


T 2: Extracompartmental (Breach in an adjacent joint cartilage, bone
cortex (or periosteum) fascia lata, quadriceps, and joint capsule)

• Metastasis: M 0: No identifiable skip lesions or distant metastases.


M 1: Any skip lesions, regional lymph nodes, or distant mets.

• Enneking's Staging System of Malignant Bone Tumors, CORR 1980

Ia Low grade, intracompartmental G1 T1 M0


Ib Low grade, extracompartmental G1 T2 M0
IIa High grade, intracompartmental G2 T1 M0
*IIb High grade, extracompartmental G2 T2 M0
IIIa Low or High grade, intracompart G 1-2 T1 M1
w/ metastases.
IIIb Low or High grade, extracompart. G 1-2 T2 M1
w/ metastases.
Osteosarcoma Most Commonly Diagnosed at IIB
Biopsy Pearls
• Do not do the biopsy unless you are the one
who is going to take care of the patient
– at least talk to the ortho oncologist at your
regional referral center before you biopsy
• Never use a transverse incision
• Hemostasis is vitally important
• 2-12 times greater complication if not done at
treating hospital
• FNA-70-75% accurate
• Core needle biopsy 85% accurate
• Open Biopsy- 96-98% accurate
Tumor Resection Definitions
• Intralesional: through the tumor
• Marginal: through the reactive zone
• Wide: En Blco removal ( Standard)
• Radical : Entire compartment removed
Pelvic Resections
• Internal hemipelvectomy – keep the limb, +/- reconstruct
with allograft hemipelvis (often need hemiarthroplasty in
addition, as native femoral head may not fit allograft
acetabulum, or native FH blood supply may be taken)
• External hemipelvectomy – take the limb

• Type I – ilium, no reconstruction needed


• Type II - periacetabular
• Type III – parasymphyseal region
• Type IV – involving SI joints/sacrum
Immunostains: Bone tumors
• S-100 = chordoma, EG
• Keratin= Mets, chordoma, adamantinoma
• CD1A = EG
• CD20 = Lymphoma
• CD99 = Ewing’s
• Cd138 = Myeloma
Ewing’s Sarcoma
• Chromosomal translocation T(11:22), found
in 90% of patients.
• Positive CD 99 reactivity
• Positive Vimentin stain (sarcoma stain)
• Histology: Monotonous sheets of small
smudgy blue cells, pseudorosettes.
• Lung mets are common
• Treatment- Chemo, resection vs radiation,
chemo again.
Immunostains: Soft Tissue
tumors
• Keratin = synovial cell, epitheliod sarcoma
• EMA = synovial cell
• Smooth muscle actin = leiomyosarcoma
• Desmin = Rhabdomyosarcoma
• Myoglobin = Rhabdomyosarcoma
• S100 = clear cell, nerve sheath tumor
• CD34 = Angiosarcoma
• Elastin = Elastofibroma
Common Chromosomal translocations

• T(11:22) = Ewing’s sarcoma


• T (X:18) = synovial sarcoma
• T (2:13) = alveolar rhabdomyosarcoma
• T (12:22) = clear cell sarcoma
• T(9:22) = chondrosarcoma
• T(12;16) = myxoid liposarcoma
Tumor Suppression Genes
• RB-1
– Regulates gene expression
– Both must be gone (recessive)
– 35% of osteosarcomas
• P53
- Prevent entrance into S-phase
- One bad copy and you have the tumor
- 20-65% of Osteosarcomas
Oncogenes
• Induce uncontrolled growth
• Resessive
• FAK, erb-2
• Chemotherapy for these tumors
Chemotherapy
• Induces programmed cell death
• Eliminated micromets in lungs
• Improves survival rate with >95% kill
• Reduces recurrence
• Bleomycin= concern for pulmonary fibrosis
• Adriamycin=concern for cardiomyopathy
Immunohistochemistry Review
• Keratins: seen in epithelial cells. Classic marker of
metastatic carcinomas. Also seen in sarcomas which
show some epithelial differentiation including
adamantinoma, epithelioid sarcoma (most common
sarcoma found in hand), and synovial sarcoma (most
common sarcoma found in foot).
Immunohistochemistry Review
• Vimentin: typical stain of all tumors of mesothelial origin.
(+) for almost all sarcomas. (-) in carcinomas.

• Desmin: typically found in muscle cells and tumors with


myo-differentiaition. These include rhabdomyosarcoma,
leiomyosarcoma, and leiomyomas. Occasionally, (+) for
desmoids and primitive neuroectodermal tumors.

• Actin: as with desmin, indicative of myogenous


differentiation.

• S-100: most commonly associated with neural,


chondroid, or melanocytic differentiation.

• Factor VIII related antigen: (+) in lesions with vascular


differentiation like hemangiomas.
Immunohistochemistry Review
• Metastatic cancer Keratin
• Lymphoma CD20 (B
cell)
• Ewing’s CD99
• Synovial Sarcoma vimentin,
EMA
• Nerve tumors S-100
• Langerhans cell histiocytosis (EG) S100,CD1A
• Chordoma Keratin, S100,
vimentin, EMA
• Myeloma CD138
• Adamantinoma Keratin
WARNING: TEST TOPICS
• CT Chest/Abd/Pelvis to Stage
• Osteosarcoma – Chemo/Surgery/Chemo
• Chondrosarcoma - Surgery
• Metastatic disease (renal cell bleeds, lung in
hand) – Mechanical
• Plasmcytoma / Multiple Myeloma -
Mechanical
• Fibrous Displasia - Mechanical
• Enchondroma - Mechanical
• Giant Cell Tumor of Bone – Locally
aggressive, Mechanical, Mets
OITE Questions
OITE 2008 – Basic Science / Onc
• 15 A 19-year-old man has the mass shown in
Figure 5a. MRI reveals a heterogeneous mass.
A biopsy specimen is shown in Figure 5b. What
is the most likely diagnosis?

• 1- Synovial sarcoma
• 2- Epithelioid sarcoma
• 3- Liposarcoma
• 4- Lipoma
• 5- Desmoid tumor
OITE 2008 – Basic Science / Onc
Q15. Figs 5a and 5b
OITE 2008 – Basic Science / Onc
• 15 A 19-year-old man has the mass shown in
Figure 5a. MRI reveals a heterogeneous mass.
A biopsy specimen is shown in Figure 5b. What
is the most likely diagnosis?

• 1- Synovial sarcoma
• 2- Epithelioid sarcoma
• 3- Liposarcoma
• 4- Lipoma
• 5- Desmoid tumor
OITE 2008 – Basic Science / Onc
Recognizing synovial sarcoma
Grossly -- well circumscribed, firm, gray-pink tumor
Radiographically -- focal calcifications
Microscopically – Many are histologically similar to synovial tissue and are
characterized by a biphasic growth pattern consisting of plump epithelioid cells that
form cords, nests, and glandular spaces, and areas of uniform spindle cells (in plump
fascicles with hyalinization and distinct lobulation accompanied by mast cells,
occasional osseous or cartilaginous metaplasia, and focal whorling) that resemble
fibrosarcomas. May be monophasic without epithelial cells.

Synovial sarcoma
• Most prevalent between 15-40 years of age.
• More common in males and tends to affect the extremities (85%-95%)
• The natural history of primary synovial sarcoma of the extremity is related to tumor
size (> 5 cm) and invasion of bone and neurovascular structures
• Treatment: surgical excision with a wide or radical margin and limb preservation when
possible; Perioperative radiation if unable to get wide margin

Lewis JJ, Antonescu CR, Leung DH, et al: Synovial sarcoma: A multivariate analysis of prognostic factors in 112
patients with primary localized tumors of the extremity. J Clin Oncol 2000;18:2087-2094.
Thompson RC Jr, Garg A, Goswitz J, et al: Synovial sarcoma: Large size predicts poor outcome. Clin Orthop Relat Res
2000;373:18-24.
OITE 2008 – Basic Science / Onc
• 31 A 34-year-old woman has a painful right
knee mass. A radiograph, MRI scans, and H &
E biopsy specimens (20X and 100X) are shown
in Figures 10a through 10d. What is the next
most appropriate step in management?

• 1- Chemotherapy alone
• 2- Arthrotomy with intralesional resection
• 3- Extra-articular wide resection
• 4- Above-knee amputation
• 5- CT of the chest and a bone scan
OITE 2008 – Basic Science / Onc
Q31. Figs 10a & 10b
OITE 2008 – Basic Science / Onc
• Q31. Figs 10c & 10d
OITE 2008 – Basic Science / Onc
• A 34-year-old woman has a painful right knee
mass. A radiograph, MRI scans, and H & E
biopsy specimens (20X and 100X) are shown in
Figures 10a through 10d. What is the next most
appropriate step in management?

• 1- Chemotherapy alone
• 2- Arthrotomy with intralesional resection
• 3- Extra-articular wide resection
• 4- Above-knee amputation
• 5- CT of the chest and a bone scan
OITE 2008 – Basic Science / Onc
• PVNS
– PVNS is a proliferation of synovial villi and nodules that may occur
locally within a joint or diffusely. Proliferation of synovial tissue can affect
the whole of the synovium (diffuse), or a localised area (nodular).

– Microscopically – subsynovial nodular proliferation of large round, polyhedral, or


spindle cells with prominent cytoplasm and pale nuclei; phagocytic histiocyte-like
cells, lipid-laden foam cells and multinucleated giant cells are interspersed with
hemosiderin-laden cells (look for iron deposition – pathopneumonic of PVNS)

– Most common presentation – Progressive and insidious onset of pain


and swelling of a joint
• Knee is most commonly affected, but it is also seen in the hip and
shoulder
• Typically in 3rd to 4th decade
– Treatment – Complete open or arthroscopic synovectomy
OITE 2008 – Basic Science / Onc
• A 10-year-old girl has had pain in the right hip for the past 2 weeks.
She denies any history of trauma or other diseases. Imaging
studies and biopsy specimens are shown in Figures 17a through
17e. What is the most appropriate treatment?

• 1- Preoperative chemotherapy, wide resection, and postoperative


chemotherapy
• 2- Hip arthrotomy, irrigation, and intravenous antibiotics for 6
weeks
• 3- In situ pinning with a 7.3-mm cannulated screw
• 4- Radiation therapy
• 5- Crutch walking and nonsteroidal anti-inflammatory drugs
OITE 2008 – Basic Science / Onc
Q46. Fig 17a
OITE 2008 – Basic Science / Onc
Q46. Fig 17 b & c
OITE 2008 – Basic Science / Onc
Q46. Fig 17 d & e
OITE 2008 – Basic Science / Onc
• A 10-year-old girl has had pain in the right hip for the past 2 weeks.
She denies any history of trauma or other diseases. Imaging
studies and biopsy specimens are shown in Figures 17a through
17e. What is the most appropriate treatment?

• 1- Preoperative chemotherapy, wide resection, and postoperative


chemotherapy
• 2- Hip arthrotomy, irrigation, and intravenous antibiotics for 6
weeks
• 3- In situ pinning with a 7.3-mm cannulated screw
• 4- Radiation therapy
• 5- Crutch walking and nonsteroidal anti-inflammatory drugs
OITE 2008 – Basic Science / Onc
• Recognizing Osteosarcoma
– Grossly: large tender soft tissue mass typically over
metaphyses of long bones
– Radiologically: destruction of the normal trabecular
bone pattern, indistinct margins, no endosteal bone
response, a mixture of radiodense and radiolucent
areas, with periosteal new bone formation, lifting of
the cortex, and Codman's triangle
– Microscopically: malignant sarcomatous stroma, w/
production of osteoid and bone
• 3 main types: osteoblastic, chondroblastic, fibroblastic
OITE 2008 – Basic Science / Onc
• 53 A 5-year-old girl has knee pain, a low grade fever,
and a limp. A radiograph and MRI scan are shown in
Figures 19a and 19b, and a biopsy specimen is shown in
Figure 19c. Molecular genetic analysis identifies an
EWS-FLI1 gene rearrangement. In addition to a CT
scan of the chest and a bone scan, what other study is
indicated for complete tumor staging?

• 1- Spinal tap
• 2- MRI of the brain
• 3- Bone marrow biopsies
• 4- Sentinel lymph node biopsy
• 5- Serum alkaline phosphatase
OITE 2008 – Basic Science / Onc
Q53. Figs 19 a – c.
OITE 2008 – Basic Science / Onc
• 53 A 5-year-old girl has knee pain, a low grade fever,
and a limp. A radiograph and MRI scan are shown in
Figures 19a and 19b, and a biopsy specimen is shown in
Figure 19c. Molecular genetic analysis identifies an
EWS-FLI1 gene rearrangement. In addition to a CT
scan of the chest and a bone scan, what other study is
indicated for complete tumor staging?

• 1- Spinal tap
• 2- MRI of the brain
• 3- Bone marrow biopsies
• 4- Sentinel lymph node biopsy
• 5- Serum alkaline phosphatase
OITE 2008 – Basic Science / Onc
• Recognizing Ewing’s Sarcoma
Grossly – rapid growing painful mass that tender on exam
Radiographically –highly aggressive appearance with permeative or
“moth-eaten” bone destruction occurring in the metadiaphysis of long
bones. Codman’s triangle often present. Periosteal reaction (classically:
onion skin appearance) and soft tissue mass without calcifications are
common.
Microscopically – Small blue cell tumor; can’t differentiate from other
small blue cell tumors therefore need BM Bx

• Simon MA, Springfield D (eds): Surgery for Bone and Soft Tissue Tumors. Philadelphia, PA,
Lippincott-Raven, 1998, pp 287-297.
• Carvajal R, Meyers P: Ewing's sarcoma and primitive neuroectodermal family of tumors. Hematol
Oncol Clin North Am 2005;19:501-525.
85. A 15 yo cross country runner reports shin pain that
is present during running, at rest, and at night.
The pain is relieved by ibuprofen. Radiographs, a
bone scan, and a CT scan are shown in Figures
32a through 32c. What is the most likely
diagnosis?
1. Cortical desmoid
2. Osteoblastoma
3. Stress fracture
4. Enchondroma
5. Osteoid osteoma
85. A 15 yo cross country runner reports shin pain that
is present during running, at rest, and at night.
The pain is relieved by ibuprofen. Radiographs, a
bone scan, and a CT scan are shown in Figures
32a through 32c. What is the most likely
diagnosis?
1. Cortical desmoid
2. Osteoblastoma
3. Stress fracture
4. Enchondroma
5. Osteoid osteoma
92. A 45 yo man has a long-standing history of an
ankle mass. He now reports that the mass
has enlarged and become quite painful over
the past several months. A radiograph and
axial MRI scans are shown in Figures 35a and
35b. What is the next most appropriate step
in management?
1. Below knee amputaton
2. Neoadjuvant chemotherapy
3. Radiation Therapy
4. Chest imaging and a bone scan
5. Limb salvage with an allograft arthrodesis
92. A 45 yo man has a long-standing history of an
ankle mass. He now reports that the mass
has enlarged and become quite painful over
the past several months. A radiograph and
axial MRI scans are shown in Figures 35a and
35b. What is the next most appropriate step
in management?
1. Below knee amputaton
2. Neoadjuvant chemotherapy
3. Radiation Therapy
4. Chest imaging and a bone scan
5. Limb salvage with an allograft arthrodesis
143. An 11 yo boy falls during a soccer game and is unable
to finish the match. The radiographs shown in figures
62a and 62b reveal a radiolucent lesion of the tibia but
no fracture. What is the most appropriate
management?

1. Repeat radiographs in 3 to 6 months


2. Needle biopsy and radiofrequency ablation under CT
direction
3. MRI, bone scane, chest CT, biopsy, and chemotherapy
4. Curettage and bone grafting
5. En bloc resection with autogenous tricortical bone
grafting
143. An 11 yo boy falls during a soccer game and is unable to
finish the match. The radiographs shown in figures 62a and
62b reveal a radiolucent lesion of the tibia but no fracture.
What is the most appropriate management?

1. Repeat radiographs in 3 to 6 months


2. Needle biopsy and radiofrequency ablation under CT
direction
3. MRI, bone scane, chest CT, biopsy, and chemotherapy
4. Curettage and bone grafting
5. En bloc resection with autogenous tricortical bone grafting
159. A 12 yo boy is seen in the ED at 11pm with
severe thigh pain. A radiograph is shown in
Figure 69. A decision is made to transfer his
care to the local orthopaedic oncologist in the
morning. What should be done prior to
transfer?

1. Splinting and pharmacologic pain control


2. Biopsy
3. Spanning external fixator
4. Femoral traction pin
5. Crutch training
159. A 12 yo boy is seen in the ED at 11pm with
sever thigh pain. A radiograph is shown in
Figure 69. A decision is made to transfer his
care to the local orthopaedic oncologist in the
morning. What should be done prior to
transfer?

1. Splinting and pharmacologic pain control


2. Biopsy
3. Spanning external fixator
4. Femoral traction pin
5. Crutch training
OITE 2008—Orthopedic Diseases
162. A 34-year-old man has had progressive back, rib, and right thigh
pain for the past 2 months. A bone scan is shown in figure 72.
Further work-up includes routine laboratory analysis, including
PSA; radiographs of the involved bones, and CT of the chest,
abdomen, and pelvis. A primary is not discovered. The prognosis
for this patient is most similar to patients with bone metastases
from which of the following primary sites?

1. Breast
2. Prostate
3. Kidney
4. Thyroid
5. Lung
OITE 2008—Orthopedic Diseases
Figure 72:
OITE 2008—Orthopedic Diseases
162. A 34-year-old man has had progressive back, rib, and right thigh
pain for the past 2 months. A bone scan is shown in figure 72.
Further work-up includes routine laboratory analysis, including
PSA; radiographs of the involved bones, and CT of the chest,
abdomen, and pelvis. A primary is not discovered. The prognosis
for this patient is most similar to patients with bone metastases
from which of the following primary sites?

1. Breast
2. Prostate
3. Kidney
4. Thyroid
5. Lung
OITE 2008—Orthopedic Diseases
Schwartz HS (ed): OKU: MSK Tumors 2. Rosemont, IL, American.
AAOS, 2007, pp 353-357.

“In 3-4% of patients with metastatic carcinoma, the primary site of origin
is unknown.”

Breast, lung, thyroid, kidney, prostate classically met to bones (“BLT w/


Kosher Pickle” if you recall USMLE Step I).

“If the lesion appears to primarily involve the cortex of a bone, or is


located distal to the knee or elbow, it is more likely to originate from
an occult primary carcinoma of the lung than from other sites.”
“In one study, radiographs of the chest identified the lung as the
primary tumor site in 43% of patients.”
OITE 2008—Orthopedic Diseases
167. A 54-year-old man with a history of alcoholism and prostate
carcinoma has mild knee pain after falling. Radiographs are shown in
Figures 74a nd 74b. What is the most likely radiographic diagnosis?

1. Chondrosarcoma
2. Bone infarct
3. Osteogenic sarcoma
4. Metastatic carcinoma of uncertain origin.
5. Metastatic prostate carcinoma.
OITE 2008—Orthopedic Diseases
Figure 74a: Figure 74b:
OITE 2008—Orthopedic Diseases
167. A 54-year-old man with a history of alcoholism and prostate
carcinoma has mild knee pain after falling. Radiographs are shown in
Figures 74a nd 74b. What is the most likely radiographic diagnosis?

1. Chondrosarcoma
2. Bone infarct
3. Osteogenic sarcoma
4. Metastatic carcinoma of uncertain origin.
5. Metastatic prostate carcinoma.
OITE 2008—Orthopedic Diseases
Bone Infarction vs. Enchondroma
• Linear or serpiginous pattern more suggestive of
infarction; popcorn or floculent pattern w/ arcs/swirls
matrix suggestive of chondroid matrix/enchondroma.
Bone Infarct Enchondroma
OITE 2008—Orthopedic Diseases
173. A 24-year-old man has a painful mass on his left humerus. A
radiograph, MRI scan, and a biopsy specimen are shown in Figures
77a through 77c. What is the most likely diagnosis?

1. Myositis ossificans
2. Osteochondroma
3. Periosteal osteosarcoma
4. Periosteal chondroma
5. Sarcoma
OITE 2008—Orthopedic Diseases
Figure 77a: Figure 77b:
OITE 2008—Orthopedic Diseases
Figure 77c:
OITE 2008—Orthopedic Diseases
173. A 24-year-old man has a painful mass on his left humerus. A
radiograph, MRI scan, and a biopsy specimen are shown in Figures
77a through 77c. What is the most likely diagnosis?

1. Myositis ossificans
2. Osteochondroma
3. Periosteal osteosarcoma
4. Periosteal chondroma
5. Sarcoma
OITE 2008—Orthopedic Diseases
Periosteal chondroma: atlasgeneticsoncology.org

Periosteal chondroma is a painful cartilaginous lesion that arises from surface


of cortex deep to the periosteum, producing broad based cartilaginous mass
that may extend into soft tissues; often develops after adolescence.
Characteristically erodes and induces sclerosis of the contiguous cortical
bone.

Pathology: It persists as mass of mature cartilage. Low power microscopy


shows well circumscribed lobulated hyaline masses. Cellularity can vary,
from hypo- to hyper-cellularity. The cartilage looks more active than
enchondroma and the lesion may be confused with chondrosarcoma.
OITE 2008—Orthopedic Diseases
174. A 29-year-old woman has a right shoulder mass. Figures 78a through
78c show the radiograph, CT scan, and H&E biopsy specimen. These
findings are associated with which of the following conditions?

1. Multiple hereditary exostoses


2. Hyperparathyroidism
3. Renal failure
4. Retinoblastoma
5. Hemangiomatosis
OITE 2008—Orthopedic Diseases
Figure 78a: Figures 78b:
OITE 2008—Orthopedic Diseases
Figure 78c:
OITE 2008—Orthopedic Diseases
174. A 29-year-old woman has a right shoulder mass. Figures 78a through
78c show the radiograph, CT scan, and H&E biopsy specimen. These
findings are associated with which of the following conditions?

1. Multiple hereditary exostoses


2. Hyperparathyroidism
3. Renal failure
4. Retinoblastoma
5. Hemangiomatosis
OITE 2008—Orthopedic Diseases
204. A 60-year-old man has left shoulder pain. A review of systems
reveals hematuria. A radiograph of the affected area is shown in
Figure 89a, and MRI scans are shown in Figures 89b and 89c.
Examination reveals a pulsatile mass. CT of the chest, abdomen, and
pelvis is likely to reveal which of the following findings?

1. Spiculated lung mass


2. Thickened colon
3. Renal mass
4. Hilar adenopathy
5. Prostate irregularity
OITE 2008—Orthopedic Diseases
Figure 89a: Figure 89b:
OITE 2008—Orthopedic Diseases
Figure 89c:
OITE 2008—Orthopedic Diseases
204. A 60-year-old man has left shoulder pain. A review of systems
reveals hematuria. A radiograph of the affected area is shown in
Figure 89a, and MRI scans are shown in Figures 89b and 89c.
Examination reveals a pulsatile mass. CT of the chest, abdomen, and
pelvis is likely to reveal which of the following findings?

1. Spiculated lung mass


2. Thickened colon
3. Renal mass
4. Hilar adenopathy
5. Prostate irregularity
OITE 2008—Orthopedic Diseases
Rougraff BT: Evaluation of the patient with carcinoma of unknown origin
metastatic to bone. Clin Orthop Relat Res 2003;415:S105-S109.

Breast, lung, thyroid, kidney, prostate classically met to bones (“BLT w/ Kosher
Pickle” if you recall USMLE Step I).

• This patient has hematuria, which immediately raises the suspicion of renal
cell carcinoma.
• Renal cell is also very vascular and therefore a large lesion can be pulsatile.
• Note that according to above reference, most mets to bone of unknown
origin are either lung (most common) or renal, possibly because these
organs are harder to examine on physical exam or tumors may grow larger
before becoming symptomatic.
OITE 2008 Orthopaedic Diseases
214. A 43-year-old woman reports a 2-month history of increasing
knee pain. She has no history of malignancy. Radiographs, MRI
scans, and a biopsy specimen are showin in Figures 92a through
92e. What is the most likely diagnosis?
1. Chordoma
2. Giant cell tumor
3. Metastatic carcinoma
4. Osteosarcoma
5. Aneurysmal bone cyst
OITE 2008 Orthopaedic Diseases
214. A 43-year-old woman reports a 2-month history of increasing
knee pain. She has no history of malignancy. Radiographs, MRI
scans, and a biopsy specimen are showin in Figures 92a through
92e. What is the most likely diagnosis?
1. Chordoma
2. Giant cell tumor (Correct location but no giant cells)
3. Metastatic carcinoma
4. Osteosarcoma
5. Aneurysmal bone cyst

1. Dorfman HD, Czerniak B: Bone Tumors. St Louis, MO, Mosby, 1998, pp 1009-1029.
2. Marcove RC, Arlen M: Atlas of Bone Pathology. Philadelphia, PA, JB Lippincott, 1992,
pp 522-530.
OITE 2008 Orthopaedic Diseases
1. Chordoma (Wrong Location, Usually in Sacrum, base of skull
and rare spine; Correcta age; malignant slow growing neoplasm of
notochord remnant; Physaliphorous Cells – Large cells with
central nuclei, numerous vacuoles in eosinophilic cytoplasm)
2. Giant cell tumor (Correct location but no giant cells)
3. Metastatic carcinoma – (Pathology shows clear cell variant of
Renal Carcinoma; Mets are the most common diagnosis; just have
to know it is not the other answers)
4. Osteosarcoma (Not the correct age; Usually does not cross the
physis; No osteoid on path)
5. Aneurysmal bone cyst (Common in spine and flat bones
although 15% occur in the femur; Large spaces marked by septa
filled with blood; Giant Cells may be present; Occasional
osteoclast lining of septa)

1. Dorfman HD, Czerniak B: Bone Tumors. St Louis, MO, Mosby, 1998, pp 1009-1029.
2. Marcove RC, Arlen M: Atlas of Bone Pathology. Philadelphia, PA, JB Lippincott, 1992,
pp 522-530.
OITE 2008 Orthopaedic Diseases
226. A 56-year-old man with a history of neurofibromatosis reports
burning pain in his foot. An MRI scan of the thigh reveals the
mass shown in Figure 96A. A biopsy is most likely to reveal the
histology shown in which of the following figures?
1. Figure 96b
2. Figure 96c
3. Figure 96d
4. Figure 96e
5. Figure 96f
1. Figure 96b

2. Figure 96c

3. Figure 96d

4. Figure 96e

5. Figure 96f
OITE 2008 Orthopaedic Diseases
226. A 56-year-old man with a history of neurofibromatosis reports
burning pain in his foot. An MRI scan of the thigh reveals the
mass shown in Figure 96A. A biopsy is most likely to reveal the
histology shown in which of the following figures?
1. Figure 96b
2. Figure 96c
3. Figure 96d
4. Figure 96e
5. Figure 96f

1. Weiss SW, Goldblum JR: Benign tumors of peripheral nerves, in Enzinger FM, Weiss SW
(eds): Soft Tissue Tumors. St Louis, MO, Mosby, 2001, pp 1111-1207.
2. Schwartz HS (ed): Orthopaedic Knowledge Update: Musculoskeletal Tumors 2. Rosemont,
IL, American Academy of Orthopaedic Surgeons, 2007, pp 273-275
1. Round blue cells
with myxoid features
can been seen in
types of
chondrosarcoma

2. Osteoid with
malginant spindle cell
stroma seen in
Osteosarcoma

3. Wavy Spindle Cells


are seen in
Neurofibromas

4. No stroma and low


cystoplasm to nuclear
ration can be seen in
Ewing’s Scarcoma

5. Vacuoles can be
seen in Chordomas
and Fatty Tumors
OITE 2008 Orthopaedic Diseases
236. A 28-year-old woman notes a growing mass in her hand for the
past several weeks. Examination reveals a nontender mass in her
palm with no skin changes or ipsilatearl adenopathy. An axial MRI
scan is shown in Figure 98a and a biopsy specimen is shown in
Figure 98b. Immunohistochemistry analysis is positive for
epithelial membrane antigen, cytokeratin and sporadic S-100.
What is the most likely diagnosis?
1. Glomus tumor
2. Squamous cell carcinoma
3. Ganglion
4. Giant cell tumor of the tendon sheath
5. Synovial sarcoma
OITE 2008 Orthopaedic Diseases
236. A 28-year-old woman notes a growing mass in her hand for the
past several weeks. Examination reveals a nontender mass in her
palm with no skin changes or ipsilatearl adenopathy. An axial MRI
scan is shown in Figure 98a and a biopsy specimen is shown in
Figure 98b. Immunohistochemistry analysis is positive for
epithelial membrane antigen, cytokeratin and sporadic S-100.
What is the most likely diagnosis?
1. Glomus tumor
2. Squamous cell carcinoma
3. Ganglion
4. Giant cell tumor of the tendon sheath
5. Synovial sarcoma

1. Weiss SW, Goldblum JR, et al: Malignant soft-tissue tumors of uncertain type, in Strauss M,
Gery L (eds): Enzinger and Weiss's Soft Tissue Tumors, ed 4. St Louis,
MO, Mosby, 2001, pp 1483-1509.
2. Pradhan A, Cheung YC, Grimer RJ, et al: Soft-tissue sarcomas of the hand: Oncological
outcome and prognostic factors. J Bone Joint Surg Br 2008;90:209-214.
OITE 2008 Orthopaedic Diseases
1. Glomus tumor – related to glomus bodies which help Giant Cell Tumor of
with temperature regulation, paroxysmal pain, worse tendon sheath
with cold, Dx with MRI
2. Squamous cell carcinoma – very rare in the hand,
path would likely have keratinized layer of skin, will not
be the OITE answer
3. Ganglion – lesion not associated with, MRI shows
homogenous fluid filled cystic structure www.bweems.com/gctten.html
4. Giant cell tumor of the tendon sheath – Benign lesion
associated with tendon sheath, giant cells are usally
present on path, must differenciat from syncovial cell Synovial Sarcoma
sarcoma on path, treatment is marginal excision, 30%
reoccur
5. Synovial sarcoma – S-100+, EMA and Cytokeratin +,
Spindle cells on path, Associate with tendon sheaths,
80% occur in the extremities, used to be called
tendosynovial sarcoma
OITE 2008 Orthopaedic Diseases
252. Figure 102 shows the radiograph of a 40-year-old man who
reports worsening knee pain and a limp. A bone scan reveals no
other sites of disease. What is the most likely diagnosis?
1. Chondroblastoma
2. Low-grad intramedularry osteogenic sarcoma
3. Giant cell tumor
4. Ewing’s sarcoma
5. Enchondroma
OITE 2008 Orthopaedic Diseases
252. Figure 102 shows the radiograph of a 40-year-old man who
reports worsening knee pain and a limp. A bone scan reveals no
other sites of disease. What is the most likely diagnosis?
1. Chondroblastoma (Patient is too old, diagnosed < 20 y/o)
2. Low-grade intramedularry osteogenic sarcoma (Does not cross
the physis)
3. Giant cell tumor (Correct age, crosses the physis)
4. Ewing’s sarcoma (Does not cross the physis)
5. Enchondroma (Does not cross the physis)

1. Turcotte RE: Giant cell tumor of bone. Orthop Clin North Am 2006;37:35-51.
2. Greenspan A, Jundt G, Remagen W (eds): Differential Diagnosis in Orthopaedic Oncology,
ed 2. Philadelphia, PA, Lippincott Williams & Wilkins, 2007, pp 387-457.
OITE 2008 Orthopaedic Diseases
273. A 28-year-old man with a history of osteogenic sarcoma 10 years
ago was treated with standard multi-agent chemotherapy that
included doxorubicin. Currently, he is disease free and needs to
undergo revision of a distal femur replacement. What test should
be performed as part of his preop work-up?
1. Liver ultrasound
2. Echocardiography
3. Bone densitometry
4. Bone marrow aspiration
5. Skin tuberculin testing
OITE 2008 Orthopaedic Diseases
273. A 28-year-old man with a history of osteogenic sarcoma 10 years
ago was treated with standard multi-agent chemotherapy that
included doxorubicin. Currently, he is disease free and needs to
undergo revision of a distal femur replacement. What test
shoulder be performed as part of his preop work-up?
1. Liver ultrasound (many are hepatotoxic, follow with LFT’s, liver
mets response to chemo can be followed with US)
2. Echocardiography (Doxorubicin is cardiotoxic)
3. Bone densitometry (Methotrexate)
4. Bone marrow aspiration (L-phenylalanine mustard (L-PAM) 
leukemia)
5. Skin tuberculin testing – can be offered before chemo so
prophylaxis can be started before they are immunocompromized
OITE 2008—Science
136. Which of the following is diagnosed by urine protein electrophoresis?
1- Monoclonal heavy chains of myeloma
2- Monoclonal light chains of myeloma
3- Diffuse B-cell lymphoma
4- Primary lymphoma of bone
5- Amyloidosis
OITE 2008—Science
136. Which of the following is diagnosed by urine protein electrophoresis?
1- Monoclonal heavy chains of myeloma
2- Monoclonal light chains of myeloma
3- Diffuse B-cell lymphoma
4- Primary lymphoma of bone
5- Amyloidosis

RECOMMENDED READINGS:
Dorfman HD, Czerniak B: Bone Tumors. St Louis, MO, Mosby, 1998, pp 664-728.
Explanation
• Myeloma is an overproduction of Plasma
cells ( B-cells) overproducing one kind of
antibody.
• These antibodies are composed of light-
chained and heavy-chained parts; the
light-chained parts can be detected in high
quantities in the urine, called Bence-Jones
proteins, and are diagnositic for myeloma.
OITE 2009 – Basic Science & Tumors
142. What predominant cell type is typically
seen on a biopsy specimen in a patient
with multiple myeloma?

1. Spindle cells
2. Giant cells
3. Osteoclasts
4. Lymphocytes
5. Plasma cells
OITE 2009 – Basic Science & Tumors
142. What predominant cell type is typically
seen on a biopsy specimen in a patient
with multiple myeloma?

1. Spindle cells
2. Giant cells
3. Osteoclasts
4. Lymphocytes
5. Plasma cells
OITE 2009 – Basic Science & Tumors
Reference: Fletcher C, Unni K, Mertens F (eds): Pathology and
Genetics of Tumors of Soft Tissue and Bone.
Multiple myeloma
-characteristic cell = plasma cell
-most common neoplasm of bone
-6th & 7th decades of life
-characterized by osteolytic lesions,
bone pain, hypercalcemia,
monoclonal gammopathy,
amyloid deposits (eg in kidney) Plasma cells in multiple myeloma

-targets = vertebrae, ribs, skull, pelvis, femur, clavicle, scapula


-diagnose w/ bone marrow aspirate; often cold on bone scan
-Poor prognosis (median survival 3 years)
OITE 2009 – Basic Science & Tumors
156. Cytogenetic analysis is a useful
approach for detecting which of the
following genetic abnormalities?

1. Point mutations
2. Frame-shift mutations
3. Microsatellite instability
4. Translocations
5. DNA cross-linking
OITE 2009 – Basic Science & Tumors
156. Cytogenetic analysis is a useful
approach for detecting which of the
following genetic abnormalities?

1. Point mutations
2. Frame-shift mutations
3. Microsatellite instability
4. Translocations
5. DNA cross-linking
51. A 12 yo boy sustained a fracture in the distal ulna twice in the
past year. He denies any major fall or injury. A radiograph and a
biopsy speciman are shown in Figures 17a and 17b. What is the
most likely diagnosis?

1. Chondroblastoma
2. Giant Cell Tumor
3. Unicameral bone cyst
4. Fibrous dysplasia
5. Nonossifying fibroma
51. A 12 yo boy sustained a fracture in the distal ulna twice in the
past year. He denies any major fall or injury. A radiograph and a
biopsy speciman are shown in Figures 17a and 17b. What is the
most likely diagnosis?

1. Chondroblastoma
2. Giant Cell Tumor
3. Unicameral bone cyst
4. Fibrous dysplasia
5. Nonossifying fibroma
UBCs

1. Occur in young pts


2. Present with pain/path fx after minor trauma
3. XRAY: Central, lytic, cortical thinning and
expansion but not usually beyond width of
physis, fallen leaf sign
4. PATH: Thin fibrous membrane, giant cells,
hemosiderin, lipophages, chronic inflammation
5. TREATMENT: Steroid injection vs. curettage &
packing if in high risk location (prox femur)
104. Synovial sarcoma is closely associated
with translocation of which of the following
genes?

1 – BCR-ABL
2 – EWS-FLI1
3 – PDGF-COL1A1
4 – SYT-SSX
5 – PAX3-FKHR
104. Synovial sarcoma is closely associated
with translocation of which of the following
genes?

1 – BCR-ABL = (CML)
2 – EWS-FLI1 = (Ewings)
3 – PDGF-COL1A1 = (Dermatofibrosarcoma
Protuberans)
4 – SYT-SSX
5 – PAX3-FKHR = (Alveolar
rhabdomyosarcoma)
251.Which of the following soft tissue
sarcomas is most common in children?

1 – epitheloid sarcoma
2 – rhabdomyosarcoma
3 – synovial sarcoma
4 – liposarcoma
5 – extraskeletal Ewing’s sarcoma
251.Which of the following soft tissue
sarcomas is most common in children?

1 – epitheloid sarcoma
2 – rhabdomyosarcoma
3 – synovial sarcoma
4 – liposarcoma
5 – extraskeletal Ewing’s sarcoma
• Five most common malignant bone tumors
in adults >50
– Metastases (PTBLK)
– Lymphoma
– Myeloma (most common primary bone tumor)
– Chondrosarcoma (most common sarcoma
age>50)
– Osteosarcoma (most common sarcoma)
• Ewings is most common malignant bone
tumor in children/adolescent
• Rhabdomyosarcoma is most common soft
tissue sarcoma in children
65. For which of the following skeletal tumors is
radiation therapy routinely used for definitive
local control?

1- Osteogenic sarcoma
2- Ewing’s sarcoma
3- Chondrosarcoma
4- Pleomorphic sarcoma
5- Paget’s sarcoma
65. For which of the following skeletal tumors is
radiation therapy routinely used for definitive
local control?

1- Osteogenic sarcoma
2- Ewing’s sarcoma
3- Chondrosarcoma
4- Pleomorphic sarcoma
5- Paget’s sarcoma
Question 232 -

What is the most current recommendation for definitive treatment of a 15 year-old boy
who has a high grade osteosarcoma of the distal femur?

1. Surgical resection only


2. Raditation therapy only
3. Radiation therapy and surgical resection
4. Chemotherapy only
5. Chemotherapy and surgical resection
Answer: 5

Chemotherapy and surgical resection combined has lead to a >80% 5 year survival rate.
Surgical restriction alone has lead to a high recurrence and metastatic rates and poor
(17.4%) survival rates. Radiation therapy alone did not control local recurrences and
pulmonary metastasis. radiation and surgery also had high local recurrences and
metastatic rates. Neoadjunctive chemotherapy has provided many improvements in
treatment including elimination of micro metastases, necrosis of the primary tumor,
reduction of tumor size and neovasculization and helps prevent local recurrences.
Complete surgical resection is the mainstay of treatment of osteogenic sarcoma, the
addition of neoadjunctive chemotherapy has proven to be the most effective.
Question 228 -

Which of the following terms best describe most osteosarcomas at the time of
diagnosis?

1. Metastatic
2. Low-grade, intracompartmental
3. Low-grade, extracompartmental
4. High-grade intracompartmental
5. High-grade extracompartmental
Answer: 5

DISCUSSION: Osteosarcoma is a difficult disease to cure because it is predominantly


high-grade and extracompartmental at the time of presentation, 90% stage IIB. The
common presentation is child or young adult with complaints of pain around the knee.
The proximal femur and humerus, as well as, the pelvis are not uncommon sites. Pre
operative multiagent chemotherapy is given for 8-12 weeks and then staging studies are
performed. Resection, if possible, is then performed and maintenance chemotherapy is
given for 6-12 months. Long term survival has increased from 10-20% to 60-70%, with
increased limb salvage.

REFERENCES: Enneking WF, Spanier SS, Goodman MA. Clin Orthop 1980;153:106-20.

This question deals with the surgical staging of musculoskeletal tumors:

STAGE GRADE SITE


IA LOW INTRACOMPARTMENTAL
IB LOW EXTRACOMPARTMENTAL
IIA HIGH INTRACOMPARTMENTAL
IIB HIGH EXTRACOMPARTMENTAL
III ANY METS

Upon diagnosis, most osteosarcomas are high grade, extracompartmental lesions, (stage IIB).
Question 186 -

A 55 year old man with metastatic prostate cancer has a painful lesion of the midshaft
of the humerus in which approx. 75% of the cortex is involved. Management should
consist of.

1. an incisional biopsy.
2. a humeral cuff and a sling
3. closed interlocking nailing
4. radiation therapy to the humerus
5. plate fixation with bone fixation
Answer: 3

Redmond reviewed 13 patients who had 16 pathological fractures of the shaft of the
humerus secondary to metastatic disease. All but one fracture was stabilized with
closed IM nailing. 14 extremities returned to nearly normal function in 3 weeks. The
conclusion stated that IM nailing of the humerus for pathological fractures provides
immediate stability and can be accomplished with a closed technique, brief operative
time, minimum morbidity, with a resultant early return of function to the extremity.
Question 89 -

A 14-year-old boy has a 4-month history of aching pain in the distal thigh.
Examination reveals a mass in the distal thigh. Figure 25a shows a plain radiograph,
Figures 25b and 25c show MRI images, Figure 25d shows a bone scan, and Figure 25e
shows a CT scan of the chest. The most likely diagnosis and Musculoskelatal Tumor
Society (Enneking) stage is

1. osteosarcoma, stage IIB


2. osteosarcoma, stage III
3. parosteal osteosarcoma, stage IIB
4. periosteal osteosarcoma, stage IIB
5. periosteal osteosarcoma, stage III
Answer: 2

Osteosarcoma commonly occurs about the knee in children and young adults. Patients present primarily with
pain. More than 90% of the intramedullary osteosarcomas are high grade and penetrate the cortex early to
form a soft tissue mass. About 10-20% of patients have pulmonary metastases at presentation. Plain
radiographs demonstrate a lesion in which there is both bone destruction and bone formation. MRI and CT
scans are useful for defining the anatomy of the lesion in regard to intramedullary extension, involvement of
neurovascular structures, and muscle invasion.
Parosteal Osteosarcoma commonly occurs at the posterior aspect of the distal femur, proximal tibia, and
proximal humerus. Patients present with a painless mass. The radiographic appearance is characteristic in
that it demonstrates a heavily ossified, often lobulated mass arising form the cortex.
Periosteal Osteosarcoma is a rare form of osteosarcoma and occurs most commonly in the diaphysis of long
bones (femur and tibia). The radiographic appearance is fairly constant; a sunburst type lesion rests on a
saucerized cortical depression.

Staging system of the Musculoskelatal Tumor Society (Enneking System)


Stage Description
I-A Low grade, intracompartmental, no metastases
I-B Low grade, extracompartmental, no metastases
II-A High grade, intracompartmental, no metastases
II-B High grade, extracompartmental, no metastases
III-A Any grade, intracompartmental, with metastases
III-B Any grade, extracompartmental, with metastases

Figure 25a demonstrates an intramedullary lesion, Figures 25b-c demonstrate a large lesion extending into
the soft tissue, and figure 25e demonstrates lung metastasis. With mets it must be a grade III and the
radiographic appearance is typical for osteosarcoma.
Question 71

A 12 year old girl has a Ewing’s sarcoma of the proximal fibula with no metastatic
disease or neurovascular involvment. Treatment should include

1. radiation therapy.
2. chemotherapy.
3. surgical resection.
4. radiation therapy and surgical resection.
5. chemotherapy and surgical resection.
Answer: 5

Ewings sarcoma is the second most common primary bone tumor in children.
It usually occurs in the first or second decades of life. Radiographically it has a highly
aggressive appearance with permeative bone destruction occurring in the diaphysis of
long bones. It may also occur, however, in flat bones. Periosteal reaction and soft tissue
mass without calcifications are common. The periosteal reaction may have a sunburst or
onion skin appearance. Radiographic differential diagnosis includes histiocytosis X,
osteomyelitis, osteosarcoma, and medullary neuroblastoma.
Historically, the 5 year survival rate of the this tumor was found to be about 5-
10%. In the past, the use of 5000-6000cGY was utilized to treat the entire bone that
was affected. This provided good local control of the disease, but it did not prevent
pulmonary metastasis. Also, the malice effects of radiation ie. skin burns, pathologic
fractures, and damages to growth plates were noted to occur. Currently, several large
series have shown that 5 year survival rates of 75-80% can be achieved with
preoperative chemotherapy to shrink the lesion followed by wide excision of the mass.
Occasionally, post-operative radiation may be performed, but the indications are still
unclear.
Thus, answers 1, 2, and 3 are incomplete, answer 4 is an old therapeutic
regimen, and answer 5 is the most correct answer for the question.
Question 42 -

Which of the following conditions has the highest rate of malignant change?

1. Ollier’s disease
2. Enchondromatosis
3. Maffucci’s syndrome
4. Multiple exostoses
5. Solitary osteochondroma
Answer: 3

Enchondromatosis is known as Ollier’s disease, which carries a 25% chance of


malignant transformation most commonly chondrosarcoma. A solitary exostosis carries
a less than 1% chance of malignant transformation while multiple can be up to 25%.
Maffucci’s syndrome is by definition multiple enchondromas with hemangiomas has a
near 100% chance of malignant transformation, therefore is the correct response.
OITE 1997
Question 1 -

A 55-year-old woman reports a spontaneous onset of severe pain in her ribs. AP and
lateral chest radiographs show severe osteopenia, two rib fractures, and three vertebral
compression fractures. Laboratory studies show a hemoglobin level of 9.0g/dL and a
monoclonal spike on serum protein electrophoresis. Which of the following imaging
studies would be most helpful in establishing the diagnosis?

1. Skeletal survey
2. Technical bone scan
3. Bone density determination
4. MRI scan of the thoracic spine
5. CT scan of the chest and abdomen
Answer: 1

The inital presentation of this patient suggests either mutiple myeloma (MM) or
metastatic CA (insidious pain onset, fractures and fracture locations). The laboratory shows an
anemia which can be consistent with both MM and metastatic CA (but is found in 62% of MM
patients). The monoclonal spike on serum electrophoresis according to RA Kyle ("Multiple
Myeloma: Review of 869 Cases"), is present in 76% of MM patients. Kyle also found 79% of
MM patients show skeletal radiographic abnormalities.
The characteristic radiographic lesions of MM are punched-out lytic areas without
osteoblastic changes, which may be sharply circumscribed. The vertebrae, skull, thoracic cage,
pelvis, and proximal portions of humerus and femur are the most common sites of involvement of
MM. The vertebral pedicles are rarely involved in myeloma, whereas they are frequently the site
of metastatic CA.
"In our [the author's] experience, diphosphonate bone scans are inferior to conventional
roentgenography (for MM)". "...in cases of carcinoma... the scan is superior". "Multiple osseous
lesions will be demonstrated on the radiographic skeletal survey in 90% of patients... and will
help determine [biopsy sites] (to diagnose MM vs. metastatic CA) " . "...radionuclide scans are
usually considerably more sensitive than conventional radiographs for the early detection of
skeletal metastases," according to Sim and Frassica. ("Metastatic Bone Disease and Myeloma" in
Evarts, CM: Surgery of the Musculoskeletal System, ed. 2 p. 5022).
Since the preliminary diagnosis favors MM, the better response is skeletal
survey to determine biopsy sites and not the bone scan.
Question 3-

What is the most common metastatic carcinoma to the hand?

1. Lung
2. Renal
3. Breast
4. Thyroid
5. Prostate
Answer: 1

Kann, SE, et al (in Instructional Course Lectures 46) note "40% of metastases
to the hand (55 of 129 in four series) arose from a primary lung tumor. The other site of
primary lesions are the breast, kidney, colon, thyroid, head and neck, and prostate."
Question 4

An otherwise healthy 45-year old man has an intraosseous low-grade chondrosarcoma


of the distal femur with no dedifferentiation or metastatic disease. Treatment should
consist of

1. surgical resection only.


2. radiation therapy only.
3. radiation therapy and surgical resection.
4. chemotherapy only.
5. chemotherapy and surgical resection.
Answer: 1

"The rate of DNA synthesis in even high-grade [chondrosarcomas] is very


low...Thus, radiation or chemotherapeutic agents that destroy primary tumors by
interfering with DNA synthesis would be predicted to have limited effect." (Mankin et
al "Chondrosarcoma of Bone" in Evarts, CM: Surgery of the Musculoskeletal System,
ed. 2)
Bauer, et al, (in Acta Orthop. Scand. 1995; 66(3): 283-288) analyzed 40 low
grade chondrosarcomas of the extremities. Their ten year local recurrence rate was 0.09
and no metastases were found in 23 patients treated with intralesional curettage alone.
The authors feel their results imply treatment with limited surgery would be beneficial
compared with the morbidity associated with en bloc resection and reconstruction.
OITE 2006--Ortho Diseases

Question 114
Which of the following tumors has the greatest potential
to metastasize to the lung?
• 1- Osteoblastoma
• 2- Enostosis
• 3- Desmoplastic fibroma
• 4- Giant cell tumor
• 5- Enchondroma
OITE 2006--Ortho Diseases

Question 114
Which of the following tumors has the greatest potential
to metastasize to the lung?
• 1- Osteoblastoma
• 2- Enostosis
• 3- Desmoplastic fibroma
• 4- Giant cell tumor
• 5- Enchondroma
42. Which of the following bone tumors contains
epithelial cells in addition to mesenchymal cells?

1. Osteosarcoma
2. Adamantinoma
3. Conventional chondrosarcoma
4. Osteoma
5. Giant cell tumor
42. Which of the following bone tumors contains
epithelial cells in addition to mesenchymal cells?

1. Osteosarcoma
2. Adamantinoma
3. Conventional chondrosarcoma
4. Osteoma
5. Giant cell tumor
Adamantinoma
• Pathology: Basic histology
consists of nests of epithelial or
epitheliod cells surrounded by a
fibrous stroma (see image). The
epithelial component can take
several different patterns (nests,
islands, cords, or tubules). The
most common pattern is a
basaloid pattern – nests of
epithelial cells with peripheral
palisading. The epithelial cells
are positive for cytokeratin, while
the stromal cells are postitive for
vimentin.
Desai SS, Jambhekar N, Agarwal M, et al: Adamantinoma of
tibia: A study of 12 cases. J Surg Oncol 2006;93:429-433.
89) Increased telemorase activity in
chondrosarcoma, as determined by
immunohistochemistry, has been shown
to directly correlate with
1) Decreased tumor grade
2) The rate of recurrence
3) Technetium activity on bone scan
4) Gadolinium uptake on MRI
5) Tumor senescence.
1) Decreased tumor grade
2) The rate of recurrence
3) Technetium activity on bone scan
4) Gadolinium uptake on MRI
5) Tumor senescence
Martin JA, DeYoung BR, Gitelis S, et al. Telemorase
reverse transcriptase subunit expression is associated
with chondrosarcoma malignancy. Clin Orthop Rleat Res
2004; 426: 117-124.

-cartilage neoplasias include malignant chondrosarcomas


and benign enchondromas. Aggressive
chondrosarcomas are often difficult to distinguish from
less aggressive ones, or benign enchondromas. They all
express aggrecan rich type II collagen reinforced ECM
that resembles normal cartilage architecture. Therefore
the need to develop malignancy specific markers for
these tumors.
57. A 58 yo man has had increasing stiffness and swelling
in the right knee for the past 9 months. Radiographs and
a biopsy specimen are shown in figures 19A-C. What is
the most likely diagnosis?

1. Tumoral Calcinosis
2. Synovial Chondromatosis
3. Synovial Hemangioma
4. Synovial Sarcoma
5. Periosteal Osetosarcoma
57. The answer is B. Synovial Chondromatosis.

The XR shows a knee with significant nodular


cartilage and bone formation.

The biopsy clearly shows balls of chondrocytes


coated in synovial lining cells.

Patients usually present as this patient did with a


stiff joint with swelling, usually with mechanical
symptoms.

Tx: Synovectomy + removal of loose bodies.


63. An 11 yo girl has knee pain and a limp. A radiograph
and an MRI are shown in figures 20A-B respectively. A
biopsy specimen is shown in Figure 20C. Routine
laboratory testing will most likely show an elevation in which
of the following?
1 – Alkaline
Phosphatase
2 – Total Bilirubin
3 – Hematocrit
4 – Lymphocytes
5 – Alpha Fetal Protein
63. The answer is 1 - alkaline phosphatase.
The image shows an osteogenic sarcoma. Due to
significant bony turnover, ALK PHOS will be
elevated.

* Total Bilirubin – elevated with involvement of


hepatobiliary system.

•Hematocrit – can be elevated during hematologic


cancers.
•Lymphocytes – elevated with infection/hemeonc
•AFP – elevated with germ cell tumors
121. Figure 37 shows the radiograph of a 15 yo
boy who has had knee pain for the past 3
months. What is the next most appropriate step
in management?
1 – Close observation with a follow up
radiograph in 6 weeks
2 – Biopsy
3 – CT scan of the chest, a bone scan, and a
MRI of the femur
4 – Cultures and IV antibiotics
5 – Radiofrequency ablation
121. The answer is C. CT chest, bone scan,
and MRI of the femur.

This boy has a lesion concerning for malignancy based on size


of tumor, symptoms of continued pain, and effect of lesion on
bone.
•Standard staging w/u includes:
Plain XR of limb
CT/MRI entire bone: eval compartments/bone
CXR/CT Chest: Eval for pulmonary mets
Bone Scans: Eval for other bony lesions
Lab tests: tailored for eval of other system involvement
including LFTs, CBC/diff, ESR/CRP, Ca/Phos,
Serum
& Urine Electrophoresis, UA, peripheral bloodsmear,
possible thyroid tests/mammagrams, poss CT
abdomen/renal US, PSA when searching for primary
OITE 2007 #186
• A 36-year-old woman has had knee pain for the
past 4 months. Radiographs, MRI scans, and a
biopsy specimen are shown in Figures 71a
through 71e. What is the most likely diagnosis?
– 1 Giant cell tumor of bone
– 2 Pigmented villonodular synovitis
– 3 Chondroblastoma
– 4 Rheumatoid Nodule
– 5 Osteomyelitis
OITE 2007 #186
• Figures 71a through 71e
OITE 2007 #186
• A 36-year-old woman has had knee pain for the
past 4 months. Radiographs, MRI scans, and a
biopsy specimen are shown in Figures 71a
through 71e. What is the most likely diagnosis?
– 1 Giant cell tumor of bone
– 2 Pigmented villonodular synovitis
– 3 Chondroblastoma
– 4 Rheumatoid Nodule
– 5 Osteomyelitis
OITE 2007 #197
• Figure 74a and 74b show the radiograph and MRI scans
of a 27-year-old woman who has had a 2-moth history of
progressive left knee pain. An open biopsy yields the
specimen shown in Figures 74c. What is the most
appropriate treatment?
– 1 Chemotherapy followed by proximal tibia resection with
prosthetic reconstruction
– 2 Proximal tibia resection with prosthetic reconstruction
– 3 Observation
– 4 External beam radiation therapy
– 5 Intralesional curettage with local adjuvant
OITE 2007 #197
• Figure 74a, 74b & 74c
OITE 2007 #197
• Figure 74a and 74b show the radiograph and MRI scans
of a 27-year-old woman who has had a 2-moth history of
progressive left knee pain. An open biopsy yields the
specimen shown in Figures 74c. What is the most
appropriate treatment?
– 1 Chemotherapy followed by proximal tibia resection with
prosthetic reconstruction
– 2 Proximal tibia resection with prosthetic reconstruction
– 3 Observation
– 4 External beam radiation therapy
– 5 Intralesional curettage with local adjuvant
OITE 2007 #203
• A 60-year-old woman has an enlarging painless mass in
the upper thigh. A radiograph and T1 and T2 weight
MRI scans are shown in Figures 75a – c. A biopsy
specimen is in 75d. Staging studies do no reveal any
evidence of metastatic disease. What is the next most
appropriate step in management?
– 1 Chemotherapy
– 2 Radiation Therapy
– 3 Mammography
– 4 Embolization
– 5 Surgery
OITE 2007 #203
• Figures 75a – d
OITE 2007 #203
• A 60-year-old woman has an enlarging painless mass in
the upper thigh. A radiograph and T1 and T2 weight
MRI scans are shown in Figures 75a – c. A biopsy
specimen is in 75d. Staging studies do no reveal any
evidence of metastatic disease. What is the next most
appropriate step in management?
– 1 Chemotherapy
– 2 Radiation Therapy
– 3 Mammography
– 4 Embolization
– 5 Surgery
OITE 2007 #203
• Chondrosarcoma is the second most common
primary malignant bone tumor
• Pathology large atypical chondrocytes
• “Although patients who have a high-grade lesion are
considered to be at high risk for metastasis,
chondrosarcomas generally are treated with
operative resection. Adjuvant radiation and
chemotherapy have been reserved for patients who
have a mesenchymal chondrosarcoma or a
dedifferentiated chondrosarcoma or for those who
have had inadequate operative treatment.”

• Lee FY. Mankin HJ. Fondren G. Gebhardt MC. Springfield DS. Rosenberg AE. Jennings LC.
Chondrosarcoma of bone: an assessment of outcome. JBJS AM. 81(3):326-38, 1999 Mar.
• Rex A. W. Marco, Steven Gitelis, Gregory T. Brebach, and John H. Healey. Cartilage Tumors:
Evaluation and Treatment
JAAOS, September/October 2000; 8: 292 - 304.
OITE 2007 #208
• A 62-year-old man with a history of nonmetastatic thyroid
carcinoma has right hip pain. Radiographs and a CT
scan are shown in figure 76a – c. He undergoes
placement of a locked IM rod to prevent pathologic
fracture. A biopsy specimen is shown in Figure 76d.
Staging reveals that this is an isolated lesion. Definitive
management should definitely include which of the
following?
– 1 Iodine 121
– 2 Radiation therapy to the right femur
– 3 Physical therapy for ROM and WBAT
– 4 Brachytherapy to the right proximal femur
– 5 Wide Resection
OITE 2007 #208
• 76a – d
OITE 2007 #208
• A 62-year-old man with a history of nonmetastatic thyroid
carcinoma has right hip pain. Radiographs and a CT
scan are shown in figure 76a – c. He undergoes
placement of a locked IM rod to prevent pathologic
fracture. A biopsy specimen is shown in Figure 76d.
Staging reveals that this is an isolated lesion. Definitive
management should definitely include which of the
following?
– 1 Iodine 121
– 2 Radiation therapy to the right femur
– 3 Physical therapy for ROM and WBAT
– 4 Brachytherapy to the right proximal femur
– 5 Wide Resection
231.A 15-year-old boy reports pain in his posterior
thigh. Radiographs and a CT scan are shown
in Figures 87a through 87c. A biopsy
specimen is shown in Figure 87d. What is the
most likely diagnosis?

1 – osteomyelitis
2 – aneurysmal bone cyst
3 – fibrous dysplasia
4 – nonossifying fibroma
5 – malignant fibrous histiocytoma
231.A 15-year-old boy reports pain in his posterior
thigh. Radiographs and a CT scan are shown
in Figures 87a through 87c. A biopsy
specimen is shown in Figure 87d. What is the
most likely diagnosis?

1 – osteomyelitis
2 – aneurysmal bone cyst
3 – fibrous dysplasia
4 – nonossifying fibroma
5 – malignant fibrous histiocytoma
242. A 55-year-old man has an isolated femoral
fracture following a motor vehicle accident. A
radiograph is shown in Figure 91. He is
hemodynamically stable and his pain is
adequately controlled. What is the next most
apporpriate step in management?

1 – antegrade intramedullary nailing of femur


2 – retrograde intramedullary nailing of the femur
3 – placement of a distal femoral traction pin
4 – wide resection
5 – biospy and staging
242. A 55-year-old man has an isolated femoral
fracture following a motor vehicle accident. A
radiograph is shown in Figure 91. He is
hemodynamically stable and his pain is
adequately controlled. What is the next most
apporpriate step in management?

1 – antegrade intramedullary nailing of femur


2 – retrograde intramedullary nailing of the femur
3 – placement of a distal femoral traction pin
4 – wide resection
5 – biospy and staging
254.A 48-year-old man reports a mass in his
thigh. He denies any history of
significant trauma. MRI scans are shown
in Figures 96a through 96c. What is the
most likely diagnosis?

1 – round cell liposarcoma


2 – dedifferentiated liposarcoma
3 – synovial sarcoma
4 – intramuscular lipoma
5 – hematoma
254.A 48-year-old man reports a mass in his
thigh. He denies any history of
significant trauma. MRI scans are shown
in Figures 96a through 96c. What is the
most likely diagnosis?

1 – round cell liposarcoma


2 – dedifferentiated liposarcoma
3 – synovial sarcoma
4 – intramuscular lipoma
5 – hematoma
273.Chemotherapy has been shown to
improve survival for all of the following
malignancies EXCEPT

1 – osteosarcoma
2 – Ewing’s sarcoma
3 – chondrosarcoma
4 – lymphoma of bone
5 – rhabdomyosarcoma
273.Chemotherapy has been shown to
improve survival for all of the following
malignancies EXCEPT

1 – osteosarcoma
2 – Ewing’s sarcoma
3 – chondrosarcoma
4 – lymphoma of bone
5 – rhabdomyosarcoma
OITE 2006 – Ortho Diseases

7. A 12-year-old patient has an epiphyseal lesion. What is


the most likely diagnosis?
1- Giant cell tumor
2- Aneurysmal bone cyst
3- Eosinophilic granuloma
4- Dysplasia epiphysealis hemimelica
5- Chondroblastoma
OITE 2006 – Ortho Diseases

7. A 12-year-old patient has an epiphyseal lesion. What is


the most likely diagnosis?
1- Giant cell tumor
2- Aneurysmal bone cyst
3- Eosinophilic granuloma
4- Dysplasia epiphysealis hemimelica
5- Chondroblastoma
OITE 2006 – Ortho Diseases

23. An 80-year-old woman reports the sudden development


of pain in the left distal thigh. She denies any history of
trauma. Figures 9a through 9d show radiographs, a bone
scan, and a biopsy specimen. What is the likely
diagnosis?
1- Enchondroma
2- Osteosarcoma
3- Dedifferentiated chondrosarcoma
4- Metastatic breast carcinoma
5- Paget’s disease
OITE 2006 – Ortho Diseases
OITE 2006 – Ortho Diseases
OITE 2006 – Ortho Diseases

23. An 80-year-old woman reports the sudden development


of pain in the left distal thigh. She denies any history of
trauma. Figures 9a through 9d show radiographs, a bone
scan, and a biopsy specimen. What is the likely
diagnosis?
1- Enchondroma
2- Osteosarcoma
3- Dedifferentiated chondrosarcoma
4- Metastatic breast carcinoma
5- Paget’s disease
OITE 2006 – Ortho Diseases

23. An 80-year-old woman reports the sudden development


of pain in the left distal thigh. She denies any history of
trauma. Figures 9a through 9d show radiographs, a bone
scan, and a biopsy specimen. What is the likely
diagnosis?
1- Enchondroma
2- Osteosarcoma
3- Dedifferentiated chondrosarcoma
4- Metastatic breast carcinoma
5- Paget’s disease
OITE 2006 Orthopedic Diseases

78. A 14 year old girl reports right thigh discomfort with activities.
An AP radiograph is shown in figure 24a, and coronal T1-weighted
and axial T2-weighted MRI scans are shown if figures 24b and
24c, respectively. A biopsy specimen is shown in Figure 24d. What
is the treatment of choice for this tumor?

1- Surgery only
2- Surgery and chemotherapy
3- Surgery and radiation therapy
4- Chemotherapy only
5- Chemotherapy and radiation therapy
OITE 2006 Orthopedic Diseases
OITE 2006 Orthopedic Diseases

78. A 14 year old girl reports right thigh discomfort with activities. An
AP radiograph is shown in figure 24a, and coronal T1-weighted
and axial T2-weighted MRI scans are shown if figures 24b and
24c, respectively. A biopsy specimen is shown in Figure 24d. What
is the treatment of choice for this tumor?

1- Surgery only
2- Surgery and chemotherapy
3- Surgery and radiation therapy
4- Chemotherapy only
5- Chemotherapy and radiation therapy
OITE 2006 Orthopedic Diseases

79. A 25 year old man reports progressive wrist pain and


swelling. An AP radiograph and biopsy specimen are
shown in Figures 25a and 25b. Management should
consist of:

1- below-elbow amputation
2- bisphosphonate treatment
3- nonsteroidal anti-inflammatory drugs
4- chemotherapy followed by wide surgical excision
5- intralesional resection and reconstruction
OITE 2006 Orthopedic Diseases
OITE 2006 Orthopedic Diseases

79. A 25 year old man reports progressive wrist pain and


swelling. An AP radiograph and biopsy specimen are
shown in Figures 25a and 25b. Management should
consist of:

1- below-elbow amputation
2- bisphosphonate treatment
3- nonsteroidal anti-inflammatory drugs
4- chemotherapy followed by wide surgical excision
5- intralesional resection and reconstruction
OITE 2006 Orthopedic Diseases

84. A 57 year old man was diagnosed with localized prostate


carcinoma 3 years ago, with negative margins and negative lymph
nodes. He now reports a 3-week history of severe right hip pain
that is worse with weight-bearing. Radiographs are shown in
Figures 26a and 26b. CT scans of the chest, abdomen, and
pelvis, as well as a whole body bone scan, are negative for other
lesions. What is the next most appropriate step for treatment?

1- Right hip hemiarthroplasty


2- Locked IM rod placement in right femur
3- Biopsy of the right femur
4- Hip disarticulation
5- Curettage, cementation, and plate fixation
OITE 2006 Orthopedic Diseases
OITE 2006 Orthopedic Diseases

84. A 57 year old man was diagnosed with localized prostate


carcinoma 3 years ago, with negative margins and negative lymph
nodes. He now reports a 3-week history of severe right hip pain
that is worse with weight-bearing. Radiographs are shown in
Figures 26a and 26b. CT scans of the chest, abdomen, and
pelvis, as well as a whole body bone scan, are negative for other
lesions. What is the next most appropriate step for treatment?

1- Right hip hemiarthroplasty


2- Locked IM rod placement in right femur
3- Biopsy of the right femur
4- Hip disarticulation
5- Curettage, cementation, and plate fixation
OITE 2006 Orthopedic Diseases

96. A 12 year old sustained a knee injury in a fall off his


bike. The patient reports that his knee pain improved
the day after injury. Radiographs shown in Figures
30a and 30b reveal a lesion in the distal femur.
Management should consist of

1- open biopsy and intralesional excision


2- CT-guided needle biopsy and anti-microbial therapy
3- curettage and autogenous iliac crest bone grafting
4- anti-inflammatory drugs as needed
5- observation and follow up
OITE 2006 Orthopedic Diseases
OITE 2006 Orthopedic Diseases

96. A 12 year old sustained a knee injury in a fall off his


bike. The patient reports that his knee pain improved
the day after injury. Radiographs shown in Figures
30a and 30b reveal a lesion in the distal femur.
Management should consist of

1- open biopsy and intralesional excision


2- CT-guided needle biopsy and anti-microbial therapy
3- curettage and autogenous iliac crest bone grafting
4- anti-inflammatory drugs as needed
5- observation and follow up
OITE 2006 Orthopedic Diseases

109. A 72 year old man with a history of smoking 40 packs of


cigarettes per year underwent a successful left total hip
replacement 10 years ago. He now reports a 2 month history of
progressive right hip pain. An AP pelvic radiograph and CT scan
are shown in Figures 35a and 35b. What is the next most
appropriate step in management?

1- Immediate cemented right total hip arthroplasty


2- Open reduction and internal fixation of the acetabular fracture
3- Activity modification, IV bisphosphonates, and follow up in 6
weeks
4- Technetium Tc 99m scan and CT of the chest, abdomen, and
pelvis
5- Radiation therapy
OITE 2006 Orthopedic Diseases
OITE 2006 Orthopedic Diseases

109. A 72 year old man with a history of smoking 40 packs of


cigarettes per year underwent a successful left total hip
replacement 10 years ago. He now reports a 2 month history of
progressive right hip pain. An AP pelvic radiograph and CT scan
are shown in Figures 35a and 35b. What is the next most
appropriate step in management?

1- Immediate cemented right total hip arthroplasty


2- Open reduction and internal fixation of the acetabular fracture
3- Activity modification, IV bisphosphonates, and follow up in 6
weeks
4- Technetium Tc 99m scan and CT of the chest, abdomen,
and pelvis
5- Radiation therapy
OITE 2006--Ortho Diseases
Question 119:
A 25 year-old man has had intermittent swelling in the
left thenar eminence for the past few months. He
describes the onset of a dull ache associated with
swelling. An AP radiograph is shown in Figure 39. What
is the most likely diagnosis?
• 1- Lipoma
• 2- Hemangioma
• 3- Synovial sarcoma
• 4- Ewing’s family of tumor
• 5- Giant cell tumor of the tendon sheath
OITE 2006--Ortho Diseases

Question 119: Figure 39


OITE 2006--Ortho Diseases
Question 119:
A 25 year-old man has had intermittent swelling in the
left thenar eminence for the past few months. He
describes the onset of a dull ache associated with
swelling. An AP radiograph is shown in Figure 39. What
is the most likely diagnosis?
• 1- Lipoma
• 2- Hemangioma
• 3- Synovial sarcoma
• 4- Ewing’s family of tumor
• 5- Giant cell tumor of the tendon sheath
OITE 2006--Ortho Diseases
Question 137
Which of the following diseases of bone (when
nonmetastatic at diagnosis) carries the worst prognosis
for 5-year survival)?
• 1- Lymphoma
• 2- Osteosarcoma
• 3- Ewing’s sarcoma
• 4- Paget’s sarcoma
• 5- Conventional chondrosarcoma
OITE 2006--Ortho Diseases
Question 137
Which of the following diseases of bone (when
nonmetastatic at diagnosis) carries the worst prognosis
for 5-year survival)?
• 1- Lymphoma
• 2- Osteosarcoma
• 3- Ewing’s sarcoma
• 4- Paget’s sarcoma
• 5- Conventional chondrosarcoma
OITE 2006--Ortho Diseases
Question 147:
A 2-year-old boy has a limp and anterior bowing of the
leg. A lateral radiograph is shown in Figure 52. What is
the most likely diagnosis?
• 1- Unicameral bone cyst
• 2- Osteofibrous dysplasia
• 3- Adamantinoma
• 4- Ewing’s sarcoma
• 5- Metastatic neuroblastoma
OITE 2006--Ortho Diseases
Question 147: Figure 52
OITE 2006--Ortho Diseases
Question 147:
A 2-year-old boy has a limp and anterior bowing of the
leg. A lateral radiograph is shown in Figure 52. What is
the most likely diagnosis?
• 1- Unicameral bone cyst
• 2- Osteofibrous dysplasia
• 3- Adamantinoma
• 4- Ewing’s sarcoma
• 5- Metastatic neuroblastoma
OITE 2006--Ortho Diseases
Question 161:
A 16 year-old girl has had vague right hip pain for the past 2
months. An AP radiograph of the pelvis and a T1 fat suppression,
gadolinium-enhanced MRI are shown in Figures 62a and 62b. A
biopsy specimen is shown in Figure 62c. What characteristic
genetic translocation is associated with this disease?

• 1- t(X;18)
• 2- t(2;13)
• 3- t(9;22)
• 4- t(11;22)
• 5- t(12;16)
OITE 2006--Ortho Diseases
Question 161: Figure 62a
OITE 2006--Ortho Diseases
Question 161: Figure 62b
OITE 2006--Ortho Diseases
Question 161: Figure 62c
OITE 2006--Ortho Diseases
Question 161:
A 16 year-old girl has had vague right hip pain for the past 2
months. An AP radiograph of the pelvis and a T1 fat suppression,
gadolinium-enhanced MRI are shown in Figures 62a and 62b. A
biopsy specimen is shown in Figure 62c. What characteristic
genetic translocation is associated with this disease?

• 1- t(X;18)
• 2- t(2;13)
• 3- t(9;22)
• 4- t(11;22)
• 5- t(12;16)
Question 16

Figure 4a shows a pigmented lesion on the right side of the neck of a 41-year-old man.
The patient's history reveals that he had multiple bone lesions during childhood and
juvenile-onset diabetes mellitus. Figure 4b and 4c show radiographs of his knee and
leg. What is the most likely diagnosis?

1. Ollier’s disease
2. Neurofibromatosis
3. McCune-Albright syndrome
4. Multiple hereditary exostoses
5. Multiple nonossifying fibromas
Answer: 3

The classic triad seen with McCune-Albright Syndrome is precocious puberty,


polyostotic fibrous dysplasia (seen in fig 4B and 4C) and café au lait spots (seen in fig
4A). The fibrous dysplasia is metaphyseal or diaphyseal and associated bowing is
common. Ollier's disease is associated with multiple enchondromatoses, but does not
include café au lait spots. Neurofibromatosis is associated with café au lait spots, but
they are generally on the trunk, pelvis and flexor creases of elbows and knees. Multiple
cutaneous fleshy tumors are also frequently seen by late childhood.
Source:Campbell's Operative Orthopedics
Atlas of Orthopedic Pathology
Merck Manual
Question 23 -

A 10-year old boy has had intermittent pain in his right groin and proximal thigh for the
past 6 months. Fingures 6a and 6b show plain radographs of the hip. Figure 6c shows
an axial proton density MRI scan through the lesion, and Figure 6d shows
representative tissue biopsy specimens at low power. What is the most likely diagnosis?

1. Chondroblastoma
2. Ewing’s sarcoma
3. Giant cell tumor
4. Simple bone cyst
5. Aneurysmal bone cyst
Answer: 5

Clinical Symptoms- pain is short in duration

B. Radiographic Features- ballooned or cystic expansion of the bone


no significant matrix the lesion affects the metaphysis of long bones
no significant mineralization
MRI- well defined lesion often with lobulated contour and internal septation with
multiple fluid levels.
C. Histology- hemosiderin laden macrophages, multinucleated giant cells, fibrous
stroma, and woven bone.

Kransdorf MS., and Berquist TH: Musculoskeletal Neoplasms; MRI of The


Musculoskeletal System 1996. pp 822-836.

Springfield DS: Bone and Soft Tissue Tumors; Lovell and Winter’s Pediatric
Orthpaedics 1996. pp50-51.

Wold LE., McLeod RA., Sim FH., and Unni KK: Atlas Of Orthopedic Pathology 1990.
pp232-237
Question 67

A 12-year-old girl has had painful, unilateral toe walking for the past 12 months.
Examination shows that her foot is fixed in equinus, and she has exquisite point
tenderness over the proximal and medial aspect of the medial gastrocnemius muscle. A
lateral radiograph of the knee is shown in figure 16a, and a T2-weighted axial MRI
scan of the proximal leg is shown in figure 16b. A photomicrograph of biopsy
material is shown in Figure 16c. What is the most likely diagnosis?

1. Rhabdomyosarcoma
2. Nodular Fascitis
3. Heterotopic Ossification
4. Soft-tissue hemangioma
5. Soft-tissue Ewing’s sarcoma
Answer: 4

Soft tissue hemangioma are common and many types are present at birth. Can occur deep in
skeletal muscle and other soft tissue of the extremities. Tend to be bluish in color. Cavernous
types frequently are visible on plain x-rays because of areas of calcifications that can be
described as smoke rings. These are felt to be pathognomonic (Enneking). Biopsy shows
vascular components that may difficult to tell from normal vascular tissues. MRI shows a
heterogeneous lesion with numerous small blood vessels and fatty infiltrate. Nonoperative
measures are used if can control discomfort. Operative treatment is wide resection, however
local recurrence is high (Miller). Rhabdomyosarcoma is a malignant tumor of skeletal muscle,
and is more common in children and adolescents. Also involved in high rate of lymph node
metastasis. Treatment includes wide resection, radiotherapy and prolonged systemic
polychemotherapy. Nodular fascitis consists of proliferating fibroblasts in myoid stroma with
predominant vascular pattern. Occurs most frequently in forearm of young adults with deep
fascia involved. Heterotopic ossification is usually from repetitive trauma-most commonly over
diaphyseal long bones. Usually not attached to underlying bone. There is a zonal pattern with
trabecular bone at peripherally and immature tissue at center. Ewing’s is rapid growing painful
mass that tender on exam. Usually begins in central portion of bone and rapidly extends
proximal and distal along canal and through vascular perforations. Bone is rapidly resorbed and
as characteristic periosteal reaction (Codman’s triangle). (Enneking, Miller)

Enzinger F, Weiss S (eds): Soft Tissue Tumors, ed 3. St Louis, MO, CV Mosby, 1995, pp605-
609.
Question 80

Figures 22a and 22b show plain xrays of a 33-year-old man who has had progressive
pain in his nondominant left shoulder for the past 5 months. A proton density MRI is
shown in Fig. 22c, and histiologic materials from the solid portion of the lesion are
shown in Figures 22d and 22e. What is the most likely diagnosis?

1. Enchondroma
2. Giant cell tumor
3. Chondroblastoma
4. Chondromyxoid fibroma
5. Clear cell chondrosarcoma
Answer: 5

Ref.: Unni KK; Dahlin’s Bone Tumors 1996, pp. 71-108

-males 70%, females 30%


-peak age; third decade
-most common location: the proximal femur accounts for approx. 50%. nearly always
involves the epiphyseal region of a long bone.
-clinical symptoms: pain is the most common at presentation
-X-ray fractures: lesion has epiphyseal location in prox. femur or humerus
Question 165 -

Figures 44a and 44b show the plain radiographs of a 12-year-old boy who has had left
medial knee pain for the past 4 months. Figure 44c shows representative histologic
material. What is the most likely diagnosis?

1. enchondroma
2. osteoblastoma
3. giant cell tumor
4. chondroblastoma
5. osteochondritis dessicans
Answer: 4

Chondroblastoma is a lytic lesion commonly located eccentrically in the


epiphysis of long bones, most often in skeletally immature individuals. The most
common sites of origin are the proximal humerus, proximal and distal femur, and
proximal tibia. The radiographic differential includes giant cell tumor crossing into the
epiphysis. In this case however, the characteristic highly cellular chondral oval
mononuclear cells seen on the histologic material confirm the diagnosis.

Turcotte RE, Kurt AM, Sim FH, et al. Chondroblastoma. Human Pathology 1993;
24: 944-949.
Question 219 -

A 35-year old man has had increasing pain in the knee for the past 4 months. An AP
radiograph of the knee is shown in Figure 58a, and low- and high-power
photomicrographs of the biopsy specimen are shown in Figures 58b and 58c. What is
the most likely diagnosis?

1. Osteosarcoma
2. Chondroblastoma
3. Giant Cell tumor
4. Aneurysmal bone cyst
5. Desmoplastic fibroma
Answer: 3

50% of Giant cell tumors usually occur about the knee, with 28% involving distal
femur. It is a lytic lesion with periosteal reaction. Histologically giant cells scattered
uniformly, and the nuclei of the mononuclear and giant cells are similar in appearance.
Most common age is 3rd decade. In this case the radiographic finding are similar to
osteosarcoma and Desmoplastic fibroma as well as Giant cell tumor, however,
histologic study singles out Giant cell tumor.

(Atlas of orthopedic surgery)


Question 222 -

Figures 60a and 60b show the radiographs of the ankle and distal leg of an 11-year-old
girl after she twisted her ankle while playing soccer. She has no history of ankle or leg
pain. Examination reveals localized swelling and tenderness over the lateral ankle, and
the tibia is not tender. The bone lesion identified in the tibia most likely is

1. osteoblastoma
2. osteoid osteoma
3. ossifying fibroma
4. fibrous dysplasia
5. nonossifying fibroma
Answer: 5

DISCUSSION: Nonossifying fibroma almost always develops in patients under the


age of twenty and is usually discovered when a x-ray is taken for an injury.
Histologically, there is a triphasic cytological appearance, featuring oval round cells,
spindle cells, and giant cells. It usually is a single well-delineated cortical defect with
circular or oval lesions and smooth, lobulated edges. Adjacent body is sclerotic without
periosteal reaction. The lower extremity long tubular bones are the most common sites
of occurrence. Treatment is usually not necessary, but occasionally a large lesion will
require intralesional excision and bone grafting.

REFERENCES: Marks KE, Bauer TW: Fibrous Tumors of Bone. Orthop Clin North
Am 1989; 20(3):377-93.
Question 227

An 80 year-old man has had increasing hip pain and difficulty ambulating for the past 6
months. An oblique radiograph pf the hip is shown in figure 64a, and a technetium bone
scan is shown in Figure 64b. Low-and high power photomicrographs are shown in
Figure 64c and 64d. What is the most likely diagnosis?

1. Pagets sarcoma
2. Pagets disease
3. Fibrous dysplasia
4. Hyperparathyroidism
5. Metastatic carcinoma
Answer: 2

Paget's disease is generally discovered in the patient 50 years of age and older. This
disease in more common in those of eastern and western European decent and rare in
Asians, Scandinavians, and blacks. 20% of patients are asymptomatic and the diagnosis
is made while being worked-up for an unrelated complaint. Symptomatic patients
complain of bone pain, skeletal deformities, changes in skin temperature, pathologic
fractures and symptoms related to nerve compression. Radiographic appearance is that
of focal bone resorption and formation in a disordered trabecular pattern, with enlarged
cortices and overall bone size. Paget's disease appears hot on technetium bone scans.
Treatment is indicated for those with increasing pain and/or deformity. Calcitonin and
diphosphonates are the pharmacologic agents available. Pathologic fractures, disabling
arthritis, and severe malalignment are indications for surgical intervention.

REFERENCES: Merkow RL, Lane JM: Paget's Disease of Bone. Orthop Clinics North
Am 1990;21(1):171-89.
OITE 2008 – Basic Science / Onc
• 8 Figure 3a shows the radiograph of a 30-year-old
woman who reports worsening left hip pain. Bone and CT
scans do not reveal any other sites of disease. A biopsy
specimen is shown in Figure 3b. The immunostaining
reactivity for smooth muscle actin is shown in Figure 3c.
What is the most likely diagnosis?

• 1- Ostegenic sarcoma
• 2- Ewing's sarcoma
• 3- Metastatic carcinoma
• 4- Malignant fibrous histiocytoma of bone
• 5- Leiomyosarcoma of bone
OITE 1997
Question 1 -

A 55-year-old woman reports a spontaneous onset of severe pain in her ribs. AP and
lateral chest radiographs show severe osteopenia, two rib fractures, and three vertebral
compression fractures. Laboratory studies show a hemoglobin level of 9.0g/dL and a
monoclonal spike on serum protein electrophoresis. Which of the following imaging
studies would be most helpful in establishing the diagnosis?

1. Skeletal survey
2. Technical bone scan
3. Bone density determination
4. MRI scan of the thoracic spine
5. CT scan of the chest and abdomen
Answer: 1

The inital presentation of this patient suggests either mutiple myeloma (MM) or
metastatic CA (insidious pain onset, fractures and fracture locations). The laboratory shows an
anemia which can be consistent with both MM and metastatic CA (but is found in 62% of MM
patients). The monoclonal spike on serum electrophoresis according to RA Kyle ("Multiple
Myeloma: Review of 869 Cases"), is present in 76% of MM patients. Kyle also found 79% of
MM patients show skeletal radiographic abnormalities.
The characteristic radiographic lesions of MM are punched-out lytic areas without
osteoblastic changes, which may be sharply circumscribed. The vertebrae, skull, thoracic cage,
pelvis, and proximal portions of humerus and femur are the most common sites of involvement of
MM. The vertebral pedicles are rarely involved in myeloma, whereas they are frequently the site
of metastatic CA.
"In our [the author's] experience, diphosphonate bone scans are inferior to conventional
roentgenography (for MM)". "...in cases of carcinoma... the scan is superior". "Multiple osseous
lesions will be demonstrated on the radiographic skeletal survey in 90% of patients... and will
help determine [biopsy sites] (to diagnose MM vs. metastatic CA) " . "...radionuclide scans are
usually considerably more sensitive than conventional radiographs for the early detection of
skeletal metastases," according to Sim and Frassica. ("Metastatic Bone Disease and Myeloma" in
Evarts, CM: Surgery of the Musculoskeletal System, ed. 2 p. 5022).
Since the preliminary diagnosis favors MM, the better response is skeletal
survey to determine biopsy sites and not the bone scan.
Question 3-

What is the most common metastatic carcinoma to the hand?

1. Lung
2. Renal
3. Breast
4. Thyroid
5. Prostate
Answer: 1

Kann, SE, et al (in Instructional Course Lectures 46) note "40% of metastases
to the hand (55 of 129 in four series) arose from a primary lung tumor. The other site of
primary lesions are the breast, kidney, colon, thyroid, head and neck, and prostate."
Question 4

An otherwise healthy 45-year old man has an intraosseous low-grade chondrosarcoma


of the distal femur with no dedifferentiation or metastatic disease. Treatment should
consist of

1. surgical resection only.


2. radiation therapy only.
3. radiation therapy and surgical resection.
4. chemotherapy only.
5. chemotherapy and surgical resection.
Answer: 1

"The rate of DNA synthesis in even high-grade [chondrosarcomas] is very


low...Thus, radiation or chemotherapeutic agents that destroy primary tumors by
interfering with DNA synthesis would be predicted to have limited effect." (Mankin et
al "Chondrosarcoma of Bone" in Evarts, CM: Surgery of the Musculoskeletal System,
ed. 2)
Bauer, et al, (in Acta Orthop. Scand. 1995; 66(3): 283-288) analyzed 40 low
grade chondrosarcomas of the extremities. Their ten year local recurrence rate was 0.09
and no metastases were found in 23 patients treated with intralesional curettage alone.
The authors feel their results imply treatment with limited surgery would be beneficial
compared with the morbidity associated with en bloc resection and reconstruction.
Question 16

Figure 4a shows a pigmented lesion on the right side of the neck of a 41-year-old man.
The patient's history reveals that he had multiple bone lesions during childhood and
juvenile-onset diabetes mellitus. Figure 4b and 4c show radiographs of his knee and
leg. What is the most likely diagnosis?

1. Ollier’s disease
2. Neurofibromatosis
3. McCune-Albright syndrome
4. Multiple hereditary exostoses
5. Multiple nonossifying fibromas
Answer: 3

The classic triad seen with McCune-Albright Syndrome is precocious puberty,


polyostotic fibrous dysplasia (seen in fig 4B and 4C) and café au lait spots (seen in fig
4A). The fibrous dysplasia is metaphyseal or diaphyseal and associated bowing is
common. Ollier's disease is associated with multiple enchondromatoses, but does not
include café au lait spots. Neurofibromatosis is associated with café au lait spots, but
they are generally on the trunk, pelvis and flexor creases of elbows and knees. Multiple
cutaneous fleshy tumors are also frequently seen by late childhood.
Source:Campbell's Operative Orthopedics
Atlas of Orthopedic Pathology
Merck Manual
Question 20

A 42-year-old health care professional has had knee pain for the past 2 months. An
MRI scan of the knee reveals a large effusion with loculations and synovial thickening,
and results of an open biopsy and culture are consistent with tuberculosis. Sensitivity
tests show no resistance to antibiotics. Following debridement and synovectomy,
appropriate antibiotic therapy should include

1. rifampin and pyridoxine.


2. rifampin and ethambutol hydrochloride.
3. isoniazid.
4. isoniazid and pyridoxine.
5. isoniazid, rifampin, pyrazinamide, and pyridoxine.
Answer: 5

Watts and Lifeso, JBJS 1996;78A:288-298.

*Current Concepts Review discusses tuberculosis of bone and joints. The case
presented shows classical findings on MRI. Successful medical treatment of
tuberculosis requires the prolonged administration of a minimum of three drugs to
which the organisms are susceptible, and at least one of these drugs must be
bactericidal.
Isoniazid is the most potent bactericidal drug available and is particularly effective
against actively growing organisms. Rifampin and pyrazinamide are the most effective
sterilizing drugs, and they are specifically effective against bacilli that are dormant and
undergo periodic bursts of activity. Ethambutol is bacteriostatic. Streptomycin is
bactericidal but must be given parenterally. Other drugs useful for multiple-drug-
resistant organisms include ethionamide, cycloserine, kanamycin, capreomycin, and
paraaminosalicylic acid.
The current recommendation for treatment of adults with musculoskeletal tuberculosis
is 300mg of isoniazid a day, 600mg of rifampin a day, and 20-30mg/kg of pyrazinamide
a day. 10mg of pyridoxine a day is given as prophylaxis against possible isoniazid-
induced neuropathy.
Question 23 -

A 10-year old boy has had intermittent pain in his right groin and proximal thigh for the
past 6 months. Fingures 6a and 6b show plain radographs of the hip. Figure 6c shows
an axial proton density MRI scan through the lesion, and Figure 6d shows
representative tissue biopsy specimens at low power. What is the most likely diagnosis?

1. Chondroblastoma
2. Ewing’s sarcoma
3. Giant cell tumor
4. Simple bone cyst
5. Aneurysmal bone cyst
Answer: 5

Clinical Symptoms- pain is short in duration

B. Radiographic Features- ballooned or cystic expansion of the bone


no significant matrix the lesion affects the metaphysis of long bones
no significant mineralization
MRI- well defined lesion often with lobulated contour and internal septation with
multiple fluid levels.
C. Histology- hemosiderin laden macrophages, multinucleated giant cells, fibrous
stroma, and woven bone.

Kransdorf MS., and Berquist TH: Musculoskeletal Neoplasms; MRI of The


Musculoskeletal System 1996. pp 822-836.

Springfield DS: Bone and Soft Tissue Tumors; Lovell and Winter’s Pediatric
Orthpaedics 1996. pp50-51.

Wold LE., McLeod RA., Sim FH., and Unni KK: Atlas Of Orthopedic Pathology 1990.
pp232-237
Question 42 -

Which of the following conditions has the highest rate of malignant change?

1. Ollier’s disease
2. Enchondromatosis
3. Maffucci’s syndrome
4. Multiple exostoses
5. Solitary osteochondroma
Answer: 3

Enchondromatosis is known as Ollier’s disease, which carries a 25% chance of


malignant transformation most commonly chondrosarcoma. A solitary exostosis carries
a less than 1% chance of malignant transformation while multiple can be up to 25%.
Maffucci’s syndrome is by definition multiple enchondromas with hemangiomas has a
near 100% chance of malignant transformation, therefore is the correct response.
Question 50 -

A 21 year old man has had increasing hip pain for the last 3 months that occurs
primarily during weightlifting exercises. AP and oblique radiographs of his hip are
shown in figures 13a and 13b. A CT scan of his hip is shown in figure 13c, and a T2-
weighted coronal MRI scan is shown in fig.13d. Low and high power
photomicrographs of the biopsy material are shown in 13e and 13f. What is the most
likely diagnosis?

1. chondroblastoma
2. giant cell tumor
3. unicameral bone cyst
4. aneurysmal bone cyst
5. hyperparathyroidism
Answer: 4

Physical exam - pain and swelling are the important features, and they vary in duration
from weeks to a few years. Lesion tends to increase in size until therapy initiated.
X-rays - a zone of rarefaction, which is usually well circumscribed and eccentric, is
associated with an obvious soft-tissue extension. In the classic case, this soft tissue
extension is produced by the bulging of periosteum and a resultant layer of
radiographically visible new bone that delimits the periphery of the tumor. The lesional
area tends to show trabeculation. Fusiform expansion may be produced when small
bones such as ribs or the fibula are affected. They may grow rapidly and mimic a
malignancy, especially telangiectatic osteosarcoma. Gross path - the cyst contains
anastomosing cavernomatous spaces that ordinarily comprise the bulk of the lesion. The
spaces are usually filled with unclotted blood; the blood may well up into, but not spurt
from, the tumor when it is unroofed. The eggshell-thick layer of subperiosteal new
bone, which delimits the lesion, is ordinarily discernible. With magnification, cavernous
spaces that may be filled with blood are identified. The walls of the spaces contain
spindled fibroblastic cells, multinucleated giant cells, and thin strands of bone.
Question 67

A 12-year-old girl has had painful, unilateral toe walking for the past 12 months.
Examination shows that her foot is fixed in equinus, and she has exquisite point
tenderness over the proximal and medial aspect of the medial gastrocnemius muscle. A
lateral radiograph of the knee is shown in figure 16a, and a T2-weighted axial MRI
scan of the proximal leg is shown in figure 16b. A photomicrograph of biopsy
material is shown in Figure 16c. What is the most likely diagnosis?

1. Rhabdomyosarcoma
2. Nodular Fascitis
3. Heterotopic Ossification
4. Soft-tissue hemangioma
5. Soft-tissue Ewing’s sarcoma
Answer: 4

Soft tissue hemangioma are common and many types are present at birth. Can occur deep in
skeletal muscle and other soft tissue of the extremities. Tend to be bluish in color. Cavernous
types frequently are visible on plain x-rays because of areas of calcifications that can be
described as smoke rings. These are felt to be pathognomonic (Enneking). Biopsy shows
vascular components that may difficult to tell from normal vascular tissues. MRI shows a
heterogeneous lesion with numerous small blood vessels and fatty infiltrate. Nonoperative
measures are used if can control discomfort. Operative treatment is wide resection, however
local recurrence is high (Miller). Rhabdomyosarcoma is a malignant tumor of skeletal muscle,
and is more common in children and adolescents. Also involved in high rate of lymph node
metastasis. Treatment includes wide resection, radiotherapy and prolonged systemic
polychemotherapy. Nodular fascitis consists of proliferating fibroblasts in myoid stroma with
predominant vascular pattern. Occurs most frequently in forearm of young adults with deep
fascia involved. Heterotopic ossification is usually from repetitive trauma-most commonly over
diaphyseal long bones. Usually not attached to underlying bone. There is a zonal pattern with
trabecular bone at peripherally and immature tissue at center. Ewing’s is rapid growing painful
mass that tender on exam. Usually begins in central portion of bone and rapidly extends
proximal and distal along canal and through vascular perforations. Bone is rapidly resorbed and
as characteristic periosteal reaction (Codman’s triangle). (Enneking, Miller)

Enzinger F, Weiss S (eds): Soft Tissue Tumors, ed 3. St Louis, MO, CV Mosby, 1995, pp605-
609.
Question 71

A 12 year old girl has a Ewing’s sarcoma of the proximal fibula with no metastatic
disease or neurovascular involvment. Treatment should include

1. radiation therapy.
2. chemotherapy.
3. surgical resection.
4. radiation therapy and surgical resection.
5. chemotherapy and surgical resection.
Answer: 5

Ewings sarcoma is the second most common primary bone tumor in children.
It usually occurs in the first or second decades of life. Radiographically it has a highly
aggressive appearance with permeative bone destruction occurring in the diaphysis of
long bones. It may also occur, however, in flat bones. Periosteal reaction and soft tissue
mass without calcifications are common. The periosteal reaction may have a sunburst or
onion skin appearance. Radiographic differential diagnosis includes histiocytosis X,
osteomyelitis, osteosarcoma, and medullary neuroblastoma.
Historically, the 5 year survival rate of the this tumor was found to be about 5-
10%. In the past, the use of 5000-6000cGY was utilized to treat the entire bone that
was affected. This provided good local control of the disease, but it did not prevent
pulmonary metastasis. Also, the malice effects of radiation ie. skin burns, pathologic
fractures, and damages to growth plates were noted to occur. Currently, several large
series have shown that 5 year survival rates of 75-80% can be achieved with
preoperative chemotherapy to shrink the lesion followed by wide excision of the mass.
Occasionally, post-operative radiation may be performed, but the indications are still
unclear.
Thus, answers 1, 2, and 3 are incomplete, answer 4 is an old therapeutic
regimen, and answer 5 is the most correct answer for the question.
Question 80

Figures 22a and 22b show plain xrays of a 33-year-old man who has had progressive
pain in his nondominant left shoulder for the past 5 months. A proton density MRI is
shown in Fig. 22c, and histiologic materials from the solid portion of the lesion are
shown in Figures 22d and 22e. What is the most likely diagnosis?

1. Enchondroma
2. Giant cell tumor
3. Chondroblastoma
4. Chondromyxoid fibroma
5. Clear cell chondrosarcoma
Answer: 5

Ref.: Unni KK; Dahlin’s Bone Tumors 1996, pp. 71-108

-males 70%, females 30%


-peak age; third decade
-most common location: the proximal femur accounts for approx. 50%. nearly always
involves the epiphyseal region of a long bone.
-clinical symptoms: pain is the most common at presentation
-X-ray fractures: lesion has epiphyseal location in prox. femur or humerus
Question 89 -

A 14-year-old boy has a 4-month history of aching pain in the distal thigh.
Examination reveals a mass in the distal thigh. Figure 25a shows a plain radiograph,
Figures 25b and 25c show MRI images, Figure 25d shows a bone scan, and Figure 25e
shows a CT scan of the chest. The most likely diagnosis and Musculoskelatal Tumor
Society (Enneking) stage is

1. osteosarcoma, stage IIB


2. osteosarcoma, stage III
3. parosteal osteosarcoma, stage IIB
4. periosteal osteosarcoma, stage IIB
5. periosteal osteosarcoma, stage III
Answer: 2

Osteosarcoma commonly occurs about the knee in children and young adults. Patients present primarily with
pain. More than 90% of the intramedullary osteosarcomas are high grade and penetrate the cortex early to
form a soft tissue mass. About 10-20% of patients have pulmonary metastases at presentation. Plain
radiographs demonstrate a lesion in which there is both bone destruction and bone formation. MRI and CT
scans are useful for defining the anatomy of the lesion in regard to intramedullary extension, involvement of
neurovascular structures, and muscle invasion.
Parosteal Osteosarcoma commonly occurs at the posterior aspect of the distal femur, proximal tibia, and
proximal humerus. Patients present with a painless mass. The radiographic appearance is characteristic in
that it demonstrates a heavily ossified, often lobulated mass arising form the cortex.
Periosteal Osteosarcoma is a rare form of osteosarcoma and occurs most commonly in the diaphysis of long
bones (femur and tibia). The radiographic appearance is fairly constant; a sunburst type lesion rests on a
saucerized cortical depression.

Staging system of the Musculoskelatal Tumor Society (Enneking System)


Stage Description
I-A Low grade, intracompartmental, no metastases
I-B Low grade, extracompartmental, no metastases
II-A High grade, intracompartmental, no metastases
II-B High grade, extracompartmental, no metastases
III-A Any grade, intracompartmental, with metastases
III-B Any grade, extracompartmental, with metastases

Figure 25a demonstrates an intramedullary lesion, Figures 25b-c demonstrate a large lesion extending into
the soft tissue, and figure 25e demonstrates lung metastasis. With mets it must be a grade III and the
radiographic appearance is typical for osteosarcoma.
Question 100

A 56 year old man who has a two year history of a progressive peripherial neuropathy
has symmetric motor and sensory deficits in the lower extremities that are worse
distally. Plain radiographs of the spine and pelvis show mulitple small sclerotic lesions
in the pubic rami, left and right ilia, and the lumbosacral spine. Serum electrophoresis
shows a monoclonal spike. What is the most likely diagnosis.

1. Metastatic lung cancer


2. Metastatic prostate cancer
3. Osteosclerotic myeloma
4. Non-Hodgkin’s lymphoma
5. Primary hyperparathyroidism
Answer: 3

This is a classic presentation of osteosclerotic myeloma otherwise know as multiple


myeloma. The major features of this disease are 97% have a homogenous monoclonal
spike on serum and urine electrophoresis usually IgG. Sensorimotor polyneuropathy
accompanies leading to severe weakness, atrophy, ataxia. Flat bones of the body
typically show multiple sclerotic punch out lesions. Malignant plasma cells produce
osteoclast activating factor that stimulate osteoclast proliferation and thus rapid bony
destruction.

Harrisons 10th Edition Principles of Internal Medicine


Question 112

Which of the following terms best describe most chondrosarcomas at intitial


presentation?

1. Metastatic
2. Low-grade, intracompartmental
3. Low-grade, extracompartmental
4. High-grade, intracompartmental
5. High-grade, extracompartmental
Answer: 2

In a study by Memorial Sloan-Kettering Cancer Center most patients presenting with


chindrosarcoma complain of pain 76% of the time as their primary symptom. Histologic
grading is divided into Grade I-27%, Grade II-42% (Low grade) and Grade III-31%
(high grade). Radiographically the lesion is centrally located 3/3 od the time.
Question 123

A 36 year-old woman who has had intermittent pain in her knee for the past 8 months
reports that over the past two months the pain has increased in frequency and intensity.
Laboratory studies show that the CBC and ESR are within normal limits. AP and lateral
radiographs are shown in Figures 33c and 33d. What is the most likely diagnosis?

1. Lymphoma
2. Osteomyelitis
3. Unicameral bone cyst
4. Aneurismal bone cyst
5. Eosinophilic Granuloma
Answer: 2

Although CBC and ESR were normal, this can happen with chronic osteo, the
histological slides show osteo and rule out the other choices above. There is a fairly
normal bone matrix (ruling out ABC or UBC) invaded with many small WBC’s in 33c.
Slide 33d shows many PMN’s, not lymphocytes, or cosinophils. Making the only
logical choice to be answer 4. The radiographs offer little help in differentiation, there is
periosteal thickeneing and evidence of an involucrum with lytic lesions although.
Ref- Dahlin DC, UnniKK: General aspects and data on 8452 cases, ed 4. Springfield,
IL, Charles Thomas, 1986, pp. 448-452.
Question 134 -

A 14-year-old boy undergoes excisional biopsy of a 3-cm mass over the lateral aspect
of the proximal forearm. No imaging studies were obtained prior to the biopsy. A
photomicrograph of the biopsy specimen is shown in Figure 36. What is the most likely
diagnosis?

1. Desmoid tumor
2. Rhabdomyosarcoma
3. Synovial sarcoma
4. Nodular fasciitis
5. Proliferative fasciitis
Answer: 3

In figure 36 a biphasic pattern of spindle cells and epithelial cells should help
to recognize a synovial sarcoma. Synovial sarcoma is most prevalent in adolescents
and young adults between 15-40 years of age. It is more common in males and tends to
affect the extremities (85%-95%).

Enzinger F, Weiss S (eds): Soft Tissue Tumors, ed 3. St. Louis, MO, CV Mosby,
1995, pp 757-786.
Question 139

A 56 year old man has had a slowly enlarging soft tissue mass in his left thigh for the
past six months. Plain radiographs show only a soft tissue shadow with no
mineralization or obvious bony involvement. The proton density MRI scans shown in
figures 39A and 39B show a coronal view and axial view, respectively, of the thigh. At
this time, management should include

1. excisional biopsy
2. incisional biopsy
3. resection with a wide margin
4. a repeat MRI scan in 3 months
5. a repeat clinical examination in 3 months
Answer: 2

The MRI study shows a large heterogenous soft tissue mass confined to the lateral compartment
of the thigh without evidence of bone involvement. Biopsy of this suspicious lesion is required
for both diagnosis and staging. Soft tissue biopsies can be performed either as a needle biopsy,
incisional biopsy, or excisional biopsy. Excisional biopsy is appropriate for small masses which
are believed to be benign. If there is any question of malignancy, incisional biopsy should be
performed through a longitudinal incision placed over the compartment involved in such a way
that the biopsy tract can be removed at the time of the definitive procedure. The clinical history
of a progressively enlarging mass with the MRI findings in figures 39a and b is highly suspicious
and warrants a biopsy. Repeat MRI in 3 months or clinical observation would only delay
treatment.

Ref. Enneking WF, Spanier SS, Goodman MA: A system for the surgical staging of
musculoskeletal Sarcoma. Clin Orthop 1980; 153:106-120.

General principles of tumors, in Crenshaw AH, (ed) Campbell’s Operative Orthopaedics, eighth
Edition, 1992, pp228-230.

Nelson TE, Enneking WF: Staging of bone and soft tissuesarcomas revisited, in Stauffer RN (ed):
Advances in Operative Orthopaedics. St. Louis, MO, Mosby Year Book, 1994.
Question 140 -

A 10 year old boy of Mediterranean ancestry whose height is in the 25th percentile
sustains a fracture of the distal femur following a mild fall. Radiographs reveal an
impacted fracture of the distal femur as well as osteopenia in both femora and the
pelvis. Laboratory studies show a hemoglobin level of 7 mg/dl. A complete
hematologic evaluation is likely to reveal

1. hemoglobin S and C
2. hemoglobin S chains only
3. no hematologic abnormalities
4. increased total iron binding capacity
5. absence of or severely deficient beta globulin
Answer: 5

The child described here most likely has thalassemia. This genetically determined hemoglobinopathy
seen in patients of Mediterranean descent is characterized by a primary deficiency in the production of
the beta chains of hemoglobin. The resulting abnormal hemoglobin leads to severe hemolytic anemia.
The anemia and secondary hemochromatosis then affect most organ systems including the skeleton.
The skeletal changes produced by thalessemia reflect compensatory hyperactivity and hypertrophy of
the bone marrow, manifested by osteoporosis, widened medullary spaces, and thinned cortices with
coarse reticulations. The clinical skeletal manifestations include arthralgias, delayed skeletal
maturation, premature fusion of the epiphyses in the long bones resulting in short stature, and an
increased incidence of fractures. The fractures heal more slowly than normal, and recurrent fractures at
the same site have been reported.
Hemaglobin S chains are indicative of sickle cell disease (HgbS-S) which occurs in 0.3-1.3% of
North American Blacks. Sickle cell disease can lead to fractures with minor trauma due to cortical
thinning secondary to marrow hyperplasia.
The presence of hemoglobin C and S indicates sickle cell- hemoglobin C disease. These patients
have musculoskeletal pain, frequently localized to the joints and have a slightly higher risk of
osteonecrosis of the femoral and humeral head. An increased TIBC suggests iron deficiency anemia.

Ref. Dines DM, Canale VC, Arnold WD: Fractures in thalassemia. J Bone Joint urg 1976; 58A:662-
666.
Resnick D, (ed) Diagnosis of Bone and Joint Disorders, WB Saunders Co, 1995 pp.2107-
2137.
Question 160 -

What is the most common clinical indicator of reflex sympathetic dystrophy of the
knee?

1. Effusion
2. Muscle atrophy
3. Atrophic hair changes
4. Disproportionate pain
5. Decreased range of motion
Answer: 4

Disproportionate pain.

O’Brien SJ, Ngeow J, Gibney MA, et al: Reflex sympathetic dystrophy of the knee:
Causes, diagnosis, and treatment. American Journal of Sports Medicine 1995;23. pp.
655-659.
Cooper DE, DeLee JC, Ramamurthy S: Reflex sympathetic dystrophy of the knee:
Treatment using continuous epidural anaesthesia. Journal of Bone Joint Surgery
1989;71A:365-369.
Question 165 -

Figures 44a and 44b show the plain radiographs of a 12-year-old boy who has had left
medial knee pain for the past 4 months. Figure 44c shows representative histologic
material. What is the most likely diagnosis?

1. enchondroma
2. osteoblastoma
3. giant cell tumor
4. chondroblastoma
5. osteochondritis dessicans
Answer: 4

Chondroblastoma is a lytic lesion commonly located eccentrically in the


epiphysis of long bones, most often in skeletally immature individuals. The most
common sites of origin are the proximal humerus, proximal and distal femur, and
proximal tibia. The radiographic differential includes giant cell tumor crossing into the
epiphysis. In this case however, the characteristic highly cellular chondral oval
mononuclear cells seen on the histologic material confirm the diagnosis.

Turcotte RE, Kurt AM, Sim FH, et al. Chondroblastoma. Human Pathology 1993;
24: 944-949.
Question 177

A 45-year-old woman who has had increasing foot pain for the past 9 months has
tenderness over the region of the cuboid. Oblique and lateral radiographs are shown in
Figures 49a and 49b. Low-and high-power photomicrographs are shown in Figures 49c
and 49d. What is the most likely diagnosis?

1. Chondroblastoma
2. Giant cell tumor
3. Unicameral bone cyst
4. Aneurysmal bone cyst
5. Metastatic carcinoma
Answer: 1

Chondroblastoma. Refer above for explanation. Note that there are islands of chondroid
matrix. This is an unusal location for this tumor. Note that chicken wire calcification on
the low power radiograph. This tumor usually occurs in patients less than 20 years old.
ABC usually occurs in patients less than 20 years of age. It is eccentric , lytic, expansile
area of bone destruction in the metaphysis. The eseential histo feature is cavernous
blood filled without an endothelial lining. UBC most commonly occurs over the
proximal humerus. The bone is often expanded. The lesion often appears trabeculated.
Tratment can include methyprednisolone injection or curretage and bone grafting
Bullough et al Atlas of orthopaedic Pathology.
Question 186 -

A 55 year old man with metastatic prostate cancer has a painful lesion of the midshaft
of the humerus in which approx. 75% of the cortex is involved. Management should
consist of.

1. an incisional biopsy.
2. a humeral cuff and a sling
3. closed interlocking nailing
4. radiation therapy to the humerus
5. plate fixation with bone fixation
Answer: 3

Redmond reviewed 13 patients who had 16 pathological fractures of the shaft of the
humerus secondary to metastatic disease. All but one fracture was stabilized with
closed IM nailing. 14 extremities returned to nearly normal function in 3 weeks. The
conclusion stated that IM nailing of the humerus for pathological fractures provides
immediate stability and can be accomplished with a closed technique, brief operative
time, minimum morbidity, with a resultant early return of function to the extremity.
Question 190 -

A 55-year old woman who has severe pain in her arm for the past 4 months reports that
she felt a sudden snap in her arm after trying to open a tight jar lid. An AP radiograph
of the humerus is shown in Figure 53a. A hight power photomictograph of the biopsy
specimen is shown in Figure 53b. What is the most likely diagnosis?

1. Lymphoma
2. Multiple Myeloma
3. Hyperparathyroidism
4. Metatstatic bone disease
5. Mesenchymal chondrosarcoma
Answer: 4

Dahlin DC, Unni KK: gineral aspects and data on 8452 cases, ed. 4. Springfield, IL,
Charles Thomas, 1985, pp 408-413.
Reference not found. In this age group (40-80 yrs.), the history alone suggests
metastatic disease. The X-ray shows an aggressive, permeative, destructive, mixed
blastic and lytic lesion with periosteal involvement and soft tissue extension in the
proximal diaphyseal humerus associated with fracture. This is uncharacteristic for the
other choices. Lymphoma and multiple myeloma are lytic. The high power H&E stain
doesn’t have chondroid matrix so mesenchymal chondrosarcoma is unlikely.
Hyperparathyroidism typically has osteoclasts and characteristic scalloping seen on
microscopy. Plus no labs are given to support the diagnosis. Most of this came from
various locations in Miller MD, Review of Otrthopaedics 2nd Ed.;WB Saunders, 1996.
Question 205

Which of the following margins is achieved in a hip disarticulation performed as


surgical treatment of a Musculoskeletal Tumor Society (Enneking) type IIA distal
femoral osteogenic sarcoma?

1. Wide
2. Radical
3. Marginal
4. Intralesional
5. Wide-contained
Answer: 2

Enneking WF: principles of Musculoskeletal Onocologic Surgery, in Evarts CM (ed):


Surgery of the Musculoskeletal System, ed 2, New York, NY. Churchill Livingstone,
1990, pp 4647-4669

Radical margins are aceived when the entire tumor and its compartment (all
surrounding muscles, ligaments and connective tissues) are removed. Marginal line of
resection goes through the reactive zone of tumor, the reactive zone contains
inflammatory cells, edema, fibrous tissue, and satellites of tumor cells and have 25 to 50
percent local recurrence. Wide margins are accomplished when the entire tumor is
removed with a cuff of normal tissue and have less than 10 percent local recurrence.
Intralesional margins go directly through the tumor and result in 100 percent local
recurrence. I don't know what wide contaminated means.
A radical margin is definitive for stage II lesions, but is no better than a wide-
margin for stage I lesions. If it can be achieved with a logical procedure, radical
local resection is just as effective as radial disarticulation for stage IIA lesions.
Stage IIB lesions in most circumstances require disarticulation to achieve a
radical margin.
Question 219 -

A 35-Year Old man has had increasing pain in the knee for the past 4 months. An AP
Radiograph of the knee is shown in Figure 58a, and low- and high-power
photomicrographs of the biopsy specimen are shown in Figures 58b and 58c. What is
the most likely diagnosis?

1. Osteosarcoma
2. Chondroblastoma
3. Giant Cell tumor
4. Aneurysmal bone cyst
5. Desmoplastic fibroma
Answer: 3

50% of Giant cell tumors usually occur about the knee, with 28% involving distal
femur. It is a lytic lesion with periosteal reaction. Histologically giant cells scattered
uniformly, and the nuclei of the mononuclear and giant cells are similar in appearance.
Most common age is 3rd decade. In this case the radiographic finding are similar to
osteosarcoma and Desmoplastic fibroma as well as Giant cell tumor, however,
histologic study singles out Giant cell tumor.

(Atlas of orthopedic surgery)


Question 222 -

Figures 60a and 60b show the radiographs of the ankle and distal leg of an 11-year-old
girl after she twisted her ankle while playing soccer. She has no history of ankle or leg
pain. Examination reveals localized swelling and tenderness over the lateral ankle, and
the tibia is not tender. The bone lesion identified in the tibia most likely is

1. osteoblastoma
2. osteoid osteoma
3. ossifying fibroma
4. fibrous dysplasia
5. nonossifying fibroma
Answer: 5

DISCUSSION: Nonossifying fibroma almost always develops in patients under the


age of twenty and is usually discovered when a x-ray is taken for an injury.
Histologically, there is a triphasic cytological appearance, featuring oval round cells,
spindle cells, and giant cells. It usually is a single well-delineated cortical defect with
circular or oval lesions and smooth, lobulated edges. Adjacent body is sclerotic without
periosteal reaction. The lower extremity long tubular bones are the most common sites
of occurrence. Treatment is usually not necessary, but occasionally a large lesion will
require intralesional excision and bone grafting.

REFERENCES: Marks KE, Bauer TW: Fibrous Tumors of Bone. Orthop Clin North
Am 1989; 20(3):377-93.
Question 227

An 80 year-old man has had increasing hip pain and difficulty ambulating for the past 6
months. An oblique radiograph pf the hip is shown in figure 64a, and a technetium bone
scan is shown in Figure 64b. Low-and high power photomicrographs are shown in
Figure 64c and 64d. What is the most likely diagnosis?

1. Pagets sarcoma
2. Pagets disease
3. Fibrous dysplasia
4. Hyperparathyroidism
5. Metastatic carcinoma
Answer: 2

Paget's disease is generally discovered in the patient 50 years of age and older. This
disease in more common in those of eastern and western European decent and rare in
Asians, Scandinavians, and blacks. 20% of patients are asymptomatic and the diagnosis
is made while being worked-up for an unrelated complaint. Symptomatic patients
complain of bone pain, skeletal deformities, changes in skin temperature, pathologic
fractures and symptoms related to nerve compression. Radiographic appearance is that
of focal bone resorption and formation in a disordered trabecular pattern, with enlarged
cortices and overall bone size. Paget's disease appears hot on technetium bone scans.
Treatment is indicated for those with increasing pain and/or deformity. Calcitonin and
diphosphonates are the pharmacologic agents available. Pathologic fractures, disabling
arthritis, and severe malalignment are indications for surgical intervention.

REFERENCES: Merkow RL, Lane JM: Paget's Disease of Bone. Orthop Clinics North
Am 1990;21(1):171-89.
Question 228 -

Which of the following terms best describe most osteosarcomas at the time of
diagnosis?

1. Metastatic
2. Low-grade, intracompartmental
3. Low-grade, extracompartmental
4. High-grade intracompartmental
5. High-grade extracompartmental
Answer: 5

DISCUSSION: Osteosarcoma is a difficult disease to cure because it is predominantly


high-grade and extracompartmental at the time of presentation, 90% stage IIB. The
common presentation is child or young adult with complaints of pain around the knee.
The proximal femur and humerus, as well as, the pelvis are not uncommon sites. Pre
operative multiagent chemotherapy is given for 8-12 weeks and then staging studies are
performed. Resection, if possible, is then performed and maintenance chemotherapy is
given for 6-12 months. Long term survival has increased from 10-20% to 60-70%, with
increased limb salvage.

REFERENCES: Enneking WF, Spanier SS, Goodman MA. Clin Orthop 1980;153:106-20.

This question deals with the surgical staging of musculoskeletal tumors:

STAGE GRADE SITE


IA LOW INTRACOMPARTMENTAL
IB LOW EXTRACOMPARTMENTAL
IIA HIGH INTRACOMPARTMENTAL
IIB HIGH EXTRACOMPARTMENTAL
III ANY METS

Upon diagnosis, most osteosarcomas are high grade, extracompartmental lesions, (stage IIB).
Question 231

What is the most likely reason that blood for a homologous transfusion that tested
negative for the HIV-antibody can carry a low but definite risk of HIV-transmission to
recipients?

1. There are many mutations of the HIV virus


2. The test for HIV-antibody is not very accurate
3. The virus may hide in the wall of red blood cells
4. The virus may hide in the wall of white blood cells
5. There is a delay between infection with HIV and the development
of a detectable antibody
Answer: 5

There is a delay between infection with HIV and the development of a detectable
antibody. Blood donated in the “window period” may ultimately lead to HIV infection
because no antibody formation has occurred thus detection is not possible. HIV
mutations may occur but antibodies should develop unless an immunodeficient
condition exists. HIV testing is very sensitive and very accurate with modern
techniques. HIV does not “hide” in the walls of wbc’s and rbc’s. HIV binds to the cell
surface, the viral capsid is released into the cell cytoplasm and ultimately viral DNA is
integrated into the nucleus and host DNA. (AIDS, 3rd Ed. V. DeVita Jr.)
Question 232 -

What is the most current recommendation for definitive treatment of a 15 year-old boy
who has a high grade osteosarcoma of the distal femur?

1. Surgical resection only


2. Raditation therapy only
3. Radiation therapy and surgical resection
4. Chemotherapy only
5. Chemotherapy and surgical resection
Answer: 5

Chemotherapy and surgical resection combined has lead to a >80% 5 year survival rate.
Surgical restriction alone has lead to a high recurrence and metastatic rates and poor
(17.4%) survival rates. Radiation therapy alone did not control local recurrences and
pulmonary metastasis. radiation and surgery also had high local recurrences and
metastatic rates. Neoadjunctive chemotherapy has provided many improvements in
treatment including elimination of micro metastases, necrosis of the primary tumor,
reduction of tumor size and neovasculization and helps prevent local recurrences.
Complete surgical resection is the mainstay of treatment of osteogenic sarcoma, the
addition of neoadjunctive chemotherapy has proven to be the most effective.
Question 243 -

The radiograph shown in Figure 67 most likely represents which of the following
disease processes?

1. Sickle Cell anemia


2. Rheumatoid arthritis
3. Ankylosing spondylitis
4. Degenerative disk disease
5. Diffuse idiopathic skeletal hyperostosis
Answer: 3

Notice the vertebral squaring and syndesmophytes that are characteristic of AS. There
is also symmetric blurring of the SI joints. Sickle cell and RA do not have these type of
changes in the axial skeleton. DDD has disc space narrowing and this x-ray has
maintained height. DISH can give similar findings, but does not have the SI changes
and apophyseal ankylosing.
Question 264 -

Figure 76 shows the radiographs of a 5-year-old girl who has pain in her left shoulder as
a result of a fall from a swing. Management should now include…

1. a biopsy.
2. a CT scan.
3. an MRI scan.
4. a sling and swathe.
5. curretage and bone grafting.
Answer: 4

The patient has a pathologic fracture through a unicameral bone cyst of her
proximal humerus. As this is a non-weight bearing bone, and a relatively benign lesion
you treat the fracture. Healing of the fracture will occasionally stimulate healing of the
cyst. If not then the cyst can be treated with curretage and bone grafting once the
fracture has healed.

(Green, Swiontkowski, Pathologic Fractures in Children – Skeletal Trauma in Children,


1st Edition, 1992.)
OITE 1998

Question 11

Which of the following techniques is most commonly used to classify tissue type in
soft-tissue tumors?

1. karyotyping
2. flow cytometry
3. transmission electron microscopy
4. immunohistochemistry staining
5. scanning electron microscopy
Answer: 4

Microscopy allows for visualization of the cells; however, special staining is required
for identification and classification of specific cells. Karyotyping refers only to the
number and structure of chromosomes in a cell. Flow cytometry is used to determine
the amount of DNA in a cell, and are predictive for the degree of malignancy. Staining
methods are currently the most commonly used method to classify tissue type.

Enzinger FM, Weiss SW (eds): Soft Tissue Tumors, ed 2. St Louis, MO, CV Mosby,
1983, pp 5-9, 83-101.
Question 15 -

A 55 year-old man has multiple lytic lesions in the humeri, clavicles, and scapulae.
Which of the following diagnostic studies best confirms a diagnosis of multiple
myeloma?

1. CT scan of the chest


2. Bone marrow biopsy
3. Complete blood cell count
4. Lateral radiograph of the skull
5. Erythrocyte sedimentation rate
Answer: 2

Myeloma is a malignant bone tumor derived from plasma cells. It is associated with
abnormalities of protein synthesis. It is the most common primary malignant tumor of
bone. The disease is most common between the ages of 50 and 80. Patients most
commonly present with bone pain, usually in the spine and ribs. Biopsy is necessary to
establish the diagnosis in a solitary lesion and is still the gold standard for diagnosis of
any lesion. Definitive diagnosis of multiple myeloma is established by bone marrow
aspiration. However, multiple myeloma may also be diagnosed with considerable
confidence based on radiographs and lab data. Bence Jones proteins may be found in
the urine. Serum protein abnormalities may cause formation of a rouleaux in the
peripheral blood smear. Serum and urine protein electrophoresis are usually abnormal.
Answers #3, #4, and #5 may provide some information but the definitive diagnosis is
based on bone marrow aspiration. Answer #1 provides little information.

Campbell's Operative Orthopaedics. Ninth edition. Volume 1, pp 726-727.


Question 17 -

Analysis of which of the following proteins is used to establish the diagnosis of Becker
muscular dystrophy?

1. Myosin
2. Troponin
3. Tropomyosin
4. Fibrillin
5. Dystrophin
Answer: 5

Becker muscular dystrophy is an X-linked inherited disorder present in approximately 1


in 30,000 live male births. The responsible gene is located on the xp21 region of the X
chromosome including sixty-five exons that encode for the protein dystrophin.
Duchenne muscular dystrophy is also related to a mutation of the dystrophin gene.
Muscle biopsies for dystrophin have been extremely successful for identifying these
dystrophies and distinguishing them from other clinically similar autosomal recessive
myopathies. (Ref: Shapiro, Hoffman)
Question 20 -

Which of the following radiographic findings would be characteristic of the knee joints
of a patient with neuropathic osteoarthropathy of the knee?

1. Fragmentation and subluxation of the normal joint articulation


2. Varus deformity with medial subchondral sclerosis
3. Preferential narrowing of the medial tibiofemoral compartment
4. Narrowing of the medial, lateral, and patellofemoral compartments
5. Bone proliferation at the patellar tendon and ligament insertions
Answer: 1

Neuropathic osteoarthropathy (a.k.a. Charcot joint) develops most often in weight-


bearing joints. The most likely cause is diabetes mellitus, but it is also associated with
syphilis, leprosy, yaws, congenital insensitivity to pain, spina bifida,
myelomeningocele, syringomyelia, aerodystrophic neuropathy, amyloid neuropathy,
peripheral neuropathy of alcoholism, spinal cord injury, peripheral nerve injury, post-
transplant neuropathy, and intraarticular steroid injections. The loss of sensation to the
joint is followed by severe degenerative changes, osteophyte formation, articular and
subchondral fractures, and often calcification of surrounding soft tissues. In the knee,
this is a tricompartmental disease and will not selectively affect one compartment over
another.
Question 45 -

A skeletal survey is more accurate than a bone scan for detecting skeletal involvement
in which of the following neoplastic diseases?

1. Ewing’s sarcoma
2. Osteogenic sarcoma
3. Multiple myeloma
4. Metastatic prostate carcinoma
5. Metastatic breast carcinoma
Answer: 3

Multiple Myeloma lesions are cold on bone scan and because of this a skeletal survey is
more useful.

Miller, Review or Orthopaedics; page 292


Question 59 -

A 60-year-old man with no history of cancer has a destructive lesion in the proximal
femur. He has a long history of tobacco use, but stopped smoking 5 years ago. A needle
biopsy specimen of the lesion shows adenocarcinoma. Which of the following studies
will most likely pinpoint the source of the primary tumor?

1. CT scan of the chest


2. technetium bone scan
3. bone marrow aspiration
4. serum protein electrophoresis
5. lateral skull radiograph
Answer: 1

Skeletal metastasis of unknown origin - ...plain radiographs of the chest established the
diagnosis of carcinoma of the lung in seventeen patients (43%).......
Unlike skeletal metastasis of known origin - most often breast or prostate - a metastasis
of unknown origin usually originates in the lung or kidney. In the present series, the
most common occult primary site was the lung (63%) and the second most common
was the kidney (10%).

Rougraff BT, Kneisl JS, Simon MA: Skeletal metastasis of unknown origin: A
prospective study of a diagnostic strategy. LBLS 1993; 75-A:1276-1281.
Question 60 -

What factor is most commonly associated with malignant transformation of a giant cell
tumor?

1. high-grade histology of the initial tumor


2. multiple local recurrences after curettage
3. previous treatment of the tumor with cryotherapy
4. previous treatment of the tumor with radiation therapy
5. extraosseous extension into two or more adjacent compartments
Answer: 4

Most authors have agreed that radiation therapy should be avoided in the treatment of
giant cell tumor, as there is a high prevalence of sarcomatous degeneration.

Gitelis S, Wilkins R, Conrad EU II: Benign bone tumors, in Pritchard DJ (ed):


Instructional Course Lectures 45. Rosemont, IL, American Academy of Orthopaedic
Surgeons, 1996, pp425-446. (p440)
Question 69 -

Crush fractures of the vertebral body are a particularly common problem in type 1
(postmenopausal) osteoporosis because

1. Trabecular bone is preferentially resorbed in this high bone turnover state


2. Loss of water content in the disk increases impact load to the vetrebral bodies
3. Stress is imposed by the relative stiffness of the arthrtic facet joints
4. Increased energy demands are imposed by decreased circulation to the vertebral
body
5. The thick cortical bone found in the vertebral body resorbs rapidly following
estrogen withdrawal
Answer: 1

The spine is composed primarily of trabecular bone, compared with cortical


bone, it has a high surface-to-volume ratio. Because metabolic activity (remodeling)
occurs on bone surfaces, trabecular bones in general and the vertebral bodies in
particular are resorbed preferentially in times of skeletal loss. Osteoporosis is
characterized by trabeculae of decreased size and number. It has also been
demonstrated in osteoporosis that there is a thinning of the cortex as well as a change in
the shape of the trabecular bone from plates to narrow bars. In the phases of bone loss,
vertebral body density declines before a similar loss is detected in cortical areas. The
body accommodates bone loss by redistribution. A 10 percent shift of bone mass
outward from the epicenter through an enlargement of the bone diameter will
compensate for 30 percent decrease in the bone mass against applied bending and
torque stresses but not against axial loading. This differential resorption explains the
timing and patterns of the fracture syndromes seen in osteoporosis. The incidence of
vertebral crush fractures rises immediately after menopause, secondary to high
trabecular content and estrogen deficiency. (precise mechanism of effects of estrogen
withdrawal unclear)

Bernstein J, Lane JM: Metabolic bone disorders of the spine, in Rothman RH, Simeone
FA (eds): The Spine, ed 3. Philadelphia, PA, WB Saunders, 1992, pp. 1381-1427.
Question 77 -

Which of the following types of sarcoma of the bone is most sensitive to external beam
radiation?

1. Ewings tumor
2. Parosteal osteosarcoma
3. Dedifferentiated chondrosarcoma
4. Low grade intramedullary chondrosarcoma
5. High grade intramedullary osteosarcoma
Answer: 1

Parosteal osteosarcoma occurs on the surface of the metaphyseal regions of the distal
femur or the proximal humerus most commonly. The treatment is wide surgical
resection versus limb salvage. Dedifferentiated chondrosarcoma has a moth eaten
appearance and may occur as a transformation of chondrosarcoma. Treatment is
resection and prognosis is poor. Low-grade intramedullary chondrosarcoma is also
treated with surgical resection. High-grade intramedullary osteosarcoma is usually
treated with pre-op chemo and resection. The only tumor listed where radiation is an
option is Ewings tumor.
Question 102 -

Flow cytometry of tumors measures the

1. Size of cells
2. Amount of DNA in cells
3. Nucleus-cytoplasm ratio
4. Specific DNA sequences
5. Specific messenger RNA sequences
Answer: 2

Flow cytometry is a method of quantitating components or structural features of cells


primarily by optical means. Ploidy and cell cycle analysis of cancers is the major
diagnostic use. Cells are passed single file through a laser beam by continuous flow and
several parameters are measured including Cell Diameter, proportional quantity of
granular (DNA) within the cell, and using fluorescent probes the total DNA or a
specific DNA/mRNA sequence can be counted. In examining tumors the amount of
DNA in each cell is important for determining neoplasia.

Ref. www.bio.umass.edu/mcfacs/intro.htm, the U. of Massachusetts web site on flow


cytometry.
Question 111 -

A 47-year-old woman who reports mild, aching pain in her knee has no history of
trauma. Examination of the knee is normal. Figure 23a shows the AP radiograph. A
bone scan shows increased uptake at this site only. Figure 23b shows the CT scan, and
Figure 23c shows the histology from the CT scan-guided needle biopsy. Treatment
should include

1. extended curettage and polymethylmethacrylate cementation


2. extra-articular resection of the knee and an allograft arthrodesis
3. wide resection of the proximal tibia and custom prosthetic replacement
4. prophylactic internal fixation and postoperative irradiation
5. excision of the lateral condyle and reconstruction with a hemicondylar allograft
Answer: 1

Patients with giant cell tumors typically present with local pain, swelling and
tenderness. Radiographs usually reveal a lesion destructive of both medullary and
cortical bone. The lesion is characterized by an expanding zone of radiolucency that is
located eccentrically in the end of a long bone in an adult.
The modern technique for the removal of a giant cell tumor involves wide decortication
of all the bone overlying the area of the tumor. The cavity is filled with
methylmethacrylate bone cement and covered with demineralized bone matrix to
stimulate the restoration of strong cortical boundaries. The other procedures are much
more invasive and not necessary to treat a low-grade neo-plastic lesion such as a giant
cell tumor. Fewer complications and better functional results have been found after
intralesional excision and insertion of methylmethacrylate than other techniques.

Gitelis S, Wilkins R, Conard EU II: Benign bone tumors. In Pritchard DJ: Instructional
Course Lectures 45,1996.
Question 123

Which of the following types of osteosarcoma is associated with the best prognosis for
long-term survival?

1. Parosteal
2. Periosteal
3. High-grade intramedullary
4. Osteosarcoma occurring in Paget’s disease
5. Osteosarcoma occurring in irradiated bone
Answer: 1

p. 194 (parosteal): Early adequate treatment [of parosteal osteosarcoma]


should lead to cure in most patients. A long-term survival rate of 80%-90% is to be
expected for patients who have parosteal osteosarcomas without dedifferentiation.”
p. 163 (in Paget’s disease): “Although long-term survival is rare for patients
with this type of sarcoma, four patients have survived more than 10 years.”
p. 164 (in irradiated bone): “The location of these tumors in unresectable
locations such as the skull, clavicle, scapula, and spine explains the traditionally poor
prognosis.”

Unni, Dahlin’s Bone Tumors, 5th ed., 1996.


Question 135 -

Which of the following metastatic tumors to bone carries the greatest risk of
complications from intraoperative bleeding?

1. Breast
2. Prostate
3. Gastrointestinal
4. Kidney
5. Multiple myeloma
Answer: 4

Just remember that renal cell CA is extremely vascular, and that an preop
embolization should be performed prior to the procedure in order to minimize
the blood loss.
Question 142 -

A form of renal osteodystrophy that is characterized by pure osteomalacia is caused by

1. Secondary hyperparathyroidism
2. Phosphate retention secondary to uremia
3. Insufficient renal synthesis of 1, 25 dihydroxy vitamin D
4. Aluminum deposition in bone from oral phosphate binders
5. Persistent acidosis aggravating the negative calcium balance
Answer: 4

There are many causes of rickets and osteomalacia. Renal osteodystrophy is a common
complication of chronic renal failure and is one of the most common causes of
osteomalacia. Pure osteomalacia is caused by the aluminum in phosphate binders used
to treat hyperphosphatemia in renal failure. Desferoxamine is an effective chelator of
aluminum in patients with biopsy documented aluminum-associated osteomalacia. Pure
osteomalacia also can be caused by hypophosphatemia. The other choices are part of
the mechanism of bone changes in renal osteodystrophy.

(Simon: Orthopedic Basic Science, 1994 pp168-172)


Question 146

Figure 30 shows the current radiograph of a 32-year-old woman who had a giant cell
tumor of the distal radius that was treated with curettage/burring and packed with
polymethylmethacrylate 2 years ago. What is the most likely diagnosis?

1. Osteomyelitis
2. Malignant degeneration
3. Stress fracture
4. Local recurrence of the giant cell tumor
5. Bone resorption due to methylmethacrylate
Answer: 4

Number four is the correct answer because recurrence is the most likely cause of the
lytic zone. Since cement resists invasion by the tumor, lysis of the surrounding bone is
inevitably produced in a recurrence. A 1-2 mm lytic zone can be found normally,
greater than 5 mm lytic zone is positive for recurrence and the 3 – 5 mm lytic zone is
very suspicious and should be followed by MRI and image guided needle biopsy. No
other sign of osteomyelitis or stress fractures are noted.
Question 153

Lymphatic metastasis is a common feature of which of the following lesions?

1. Liposarcoma
2. Nodular fasciitis
3. Rabdomyosarcoma
4. Malignant fibrous histiocytoma
5. Extra-abdominal desmoid tumor
Answer: 3

Rhabdomyosarcoma is a high grade malignancy with a rapid growth pattern. Local


recurrence and distant metastasis after inadequate excision occurs in almost all
instances and is uniformly fatal. The primary site of metastasis is the lung. Lymph node
metastasis occurs in about > 10% of the patients, (synovial cell sarcoma = 25%).
Whereas M.F.H. only occasionally metastasizes to regional lymph nodes, and
liposarcoma is slow growing and recurrences or local metastasis is < 10%.

Reference: Enneking, W.F. et al. Clinical Musculoskeletal Pathology , University of


Florida, pp. 16-1-16-46.
Question 170

Which of the following surgical options after resection of a sarcoma about the knee
would require a patient to expend the greatest amount of energy while walking?

1. Arthrodesis
2. Rotationplasty
3. Above-knee amputation
4. Osteoarticular allograft
5. Endoprosthesis (custom arthroplasty)
Answer: 3

The answer to this question is based on a study by Harris which tested the effeciency ,
rate, and percent of max rate in ambulation, in amputees, those with arthrodeses and
arthroplasties. They found that Above knee amputees expended the most energy,
followed by arthrodesis patients, and then arthroplasty patients.

Reference: Harris IE, Leff AR, Gitelis S. et al: Function after amputation, arthrodesis,
arthroplasty for tumors about the knee. JBJS 1990; 72A:1477-1485.
Question 186 -

Which of the following metastatic carcinomas has the worst long-term prognosis?

1. Lung
2. Breast
3. Prostate
4. Thyroid
5. Renal
Answer: 1

The answer is the lung, which has a dismal long-term prognosis. However, not included in the
question, but reviewed in the cited reference and worthy of noting is Gastric CA. This has the
worst prognosis with a 15% survival rate of 6 months after spinal metastasis is detected.
Recommendations are palliative care rather than surgical intervention.
Pulmonary has 6-month & 1-year survival rates of 15% & 22% respectively. Surgical
intervention will be based on the overall condition of the patient and when indicated the extent of
surgical intervention should be as small as possible.
Breast & prostate CA have better survival rates than the others, and are often hormonally
sensitive tumors. Making endocrine therapy initially crucial with the good long term survival of
prostate CA patients. When intervention is indicate, it should be combined with other modalities
(Chemo, Etc.) and a more extensive surgery including decompression and fixation, but also
resection of the metastatic lesion and reconstruction.
Breast CA rarely progresses with symptoms due to spinal metastasis, likely due to symptoms
being controlled with hormone and radiation therapy.
Renal CA has a poor 6 months survival rate of 51% which is close to lung CA, but a sharp
decline in deaths from 6-36 mos results in a high (40%) 3-year survival rate. These should be
categorized into rapid growing & slow growing with surgical intervention based on this
classification.

Reference: Tatsui H, et al. Survival rates of patients with metastatic spinal cancer after
scintigraphic detection of abnormal radioactive accumulation. Spine 1996;21:2143-48.
Question 187 -

The Magnetic resonance imaging signal characteristics of a high-grade soft-tissue


sarcoma are best described as

1. T1-low, T2-low.
2. T1-low, T2-high.
3. T1-moderate, T2-low.
4. T1-high, T2-low.
5. T1-high, T2-high.
Answer: 2

After plain radiographs of the affected area have been obtained, magnetic resonance imaging
modality is the best imaging modality for detecting and characterizing the lesion, regarding
definition of normal muscle, fascial boundaries, and the tumor mass. Although MR imaging is
not specific in determining whether lesions are benign or malignant, it can be useful in
evaluation other characteristics, such as size, pattern of growth, integrity of natural boundaries,
and homogeneity.
Intravenous contrast agents are not necessary to evaluate neurovascular structures. Both the T1
& T2 weighted images are essential to detect and characterize soft tissue lesions.
Most Tumors have long T1 and T2 relaxation times, therefore, in most instances signal intensity
alone is of limited benefit. Exceptions are lipoma, hematoma, intra-lesional hemorrhage.
Hemorrhage may occur in some soft tissue lesions, especially sarcomas, leading to the low T1
and high T2 intensity sound on MR. Miller, indicates that Water, CSF, acute hemorrhage and soft
tissue tumors appear dark on T1 and light on T2.

Reference: Soft-Tissue Tumors: Diagnosis, Evaluation, and Management. JAAOS 1994;2:202-


211.

Sundaram, M, The Role of MRI in evaluation of Muskuloskeletal Tumors, Radiology: Diagnosis-


Imaging_Intervention Vol 5: 104; 6-10.
Question 210 -

The radiograph shown in Figure 50a and the CT scan shown in Figure 50b reveal a
lesion in the left femoral neck of a 12-year-old boy who has pain in the left hip. The
most likely cause of the osteopenia of the left proximal femur is

1. disuse osteopenia
2. paraendocrine effect of the tumor
3. abnormally increased density on the right side
4. side effect of the treatment of the lesion
5. extensive tumor involvement of the left hip
Answer: 1

Figure 50A: AP pelvis with a 1 cm lucency in the calcar region of the femoral neck and
diffuse osteopenia of the proximal femur. Figure 50B: CT scan of the same patient
showing a well demarcated lcm lesion in the femoral neck with an obvious nidus.
Careful evaluation of the radiographs is critical in this question. First the obvious
osteoid osteoma must be recognized. From there answers 2 and 3 can be eliminated.
Neither radiograph reveals any sign that treatment has occurred, and the CT scan shows
no signs of extensive involvement, thus answer 5 should also be eliminated. The real
key is believing that such diffuse osteoporosis could occur in such a young child, as
Jones described in his article from 1969.

Jones G: Radiological appearances of disuse osteoporosis. Clin Radiol


1969;20:345-353.
Question 224 -

The diagnostic distinction between a benign enchondroma and a low-grade


intramedually chondrosarcoma is based primarily on the

1. clinical history and radiographic findings.


2. technetium bone scan
3. flow cytometry pattern of extracted chondrocytes
4. immunohistochemical staining patterns of a biopsy specimen
5. histologic features of a biopsy specimen stained with hematoxylin-cosin
Answer: 1

"The diagnostic strategies for benign bone tumors center on the initial radiographic
presentation." … "For example calcified, lytic phylangeal lesion
strongly suggests the presence of cartilage that is consistent with enchondroma."
Question 246 -

Figures 66a and 66b show the radiographs of an 8-year-old girl who has a firm,
immobile mass of her middle finger at the proximal interphalangeal joint. Figure 66c
shows the histopathology of the biopsy specimen. What is the most likely diagnosis?

1. Fracture healing
2. Chondrosarcoma
3. Periosteal chondroma
4. Periosteal osteosarcoma
5. Dysplasia epiphysealis hemimelica
Answer: 3

Periosteal chondroma is a benign chondroid tumor that arises under or in the


periosteum on the surface of cortical bone. Has a scalloping of the cortex with a well
defined margin between the tumor and bone. The tumor bed shows a variable amount of
sclerosis. Also a variable amount of calcification exists. Histologically, as in this case,
there is chondroid matrix. The predilection for the proximal end of long bones,
particularly the humerus, and the bones of the hands, has been noted. This tumor must
be differentiated from periosteal chondrosarcoma, which penetrates the cortical bone
and is generally larger with a soft tissue mass. Histology will also show more
polymorphism and hyperchromatic nuclei of chondrosarcoma.

Boriani. JBJS 1983;65A:205-212.


Question 262

A 45-year-old woman has a deep soft-tissue mass in the thigh. The MRI scan Shows a
12-cm heterogenous mass that is deep to the fascia, and the CT scan Shows three small
(5 to 10 mm) peripheral, noncalcified pulmonary nodules. What is the stage of disease
according to the staging system of the Musculoskeletal Tumor Society?

1. 1
2. 2
3. I
4. II
5. III
Answer: 5

The system adopted by the Musculoskeletal Tumor Society is based on three factors:
Grade (G), Site (T), and metastases (M). Each of these factors is stratified by
components that influence both prognosis and response to treatment. Grade is an
assessment of biologic aggressiveness of the legion. There are three stratifications of
Grade: G0 = benign, G1 = low grade malignant (few mitoses, moderate differentiation,
and distinct matrix), G2 = high grade malignant (frequent mitoses, poorly differentiated
cells, and sparse immature matrix). Site is the anatomic setting of the legion,
determined
by radiographic imaging. There are three strata T0 = intracapsular (does not extend
beyond the compartmental boarders of its origin and remains completely within its
capsule) T1 = extracapsular, intracompartmental, (extends beyond its capsule but
remains within its compartment), T2 = extracapsular, extracompartmental, (extends
beyond its compartmental boundaries). Metastasis has two strata M0 = no evidence of
regional or distant mets and M1 = regional or distant mets. Stages for benign tumors
are listed as 1,2 and 3 whereas stages for malignant tumors are I, II, and III.
Stage
Grade Site Mets

Benign tumors
Stage 1, latent G0
T0 M0
Stage 2, active G0
T0 M0
Stage 3, aggressive G0
T0 M0
Malignant Tumors
Stage I, low grade
A. Intracompartmental G1
T1 M0
B. Extracompartmental G1
T2 M0
Stage II, high grade
A. intracompartmental G2
T1 M0
B. extacompartmental G2
T2 M0
Stage III, metastatic
A. intracompartmental
distant mets G1-2
T1 M1
B. extracompartmental
distant mets
G1-2 T2 M1
By the nature of the radiographic appearance of this person’s tumor and
assuming we are dealing with a malignant tumor with metastases, this has to be a stage III tumor.
Question 269 -

A patient who has a malignant bone tumor of the hip undergoes resection and hip
arthrodesis. While walking, mean oxygen consumption would be approximately what
percent of that observed in someone who has a normal hip?

1. 50%
2. 100%
3. 130%
4. 200%
5. 300%
Answer: 3

“Unilateral arthrodesis has been shown to increase O2 consumption in normal gait by


32%.”
OITE 1999

Question 8

Figure 2 shows the plain AP radiograph of a 20-year-old man who has had a
painful swelling in his left wrist for the past 4 months. A bone scan shows a
solitary lesion. A radiograph and CT scan of the chest are normal. Prior to
biopsy, what is the most likely diagnosis?

1. Telangiectatic osteosarcoma
2. Chondromyxoid fibroma
3. aneurysmal bone cyst
4. giant cell tumor
5. chondroblastoma
Answer: 4

Bones involved are the distal femur, proximal tibia, distal radius. other long
bones. On x-ray a lytic lesion in the epiphysis without surrounding sclerosis or
periosteal bone reaction. Often has a "soap bubble" pattern. Treatment is en
bloc excision, if possible, but usual treatment is currettage.
Question 17 -

Which of the following is considered the most common primary carcinoma to


metastasize to bones distal to the elbow and knee?

1. Breast
2. Prostate
3. Kidney
4. Lung
5. Thyroid
Answer: 4

Axial skeletal involvement is more common than appendicular involvement, and


metastatic lesions below the elbow and knee are relatively uncommon.
Autopsy studies on patients with metastatic disease below the knee and elbow
revealed that lung was the most common primary site. The tibia was the most
common bone affected. Pain, local soft-tissue extension and pathologic
fractures were the most common presenting symptoms.

Reference: Leeson MC, Makley JT, Carter JR: Metastatic skeletal disease
distal to the elbow and knee. Clin Orthop 1986;206:94-99.
Question 25

Which of the following signal sequences describes the MRI scan characteristics
of normal tendons?

1. Low on T1-weighted images and low on T2-weighted images


2. Low on T1-weighted images and high on T2-weighted images
3. High on T1-weighted images and low on T2-weighted images
4. High on T1-weighted images and moderate on T2-weighted images
5. High on T1-weighted images and high on T2-weighted images
Answer: 1

T1 sequenced images are weighted toward fat and T2 sequenced


images are weighted toward water. Consequently, normal tendons, ligaments,
and cortical bone all appear low signal on all pulse sequences.

Beaty JH (ed): OKU 6. Rosemant IL, AAOS, 1999, 9981-87.


Question 36 -

Which of the following conditions will most likely eventually develop in a


newborn girl with achondroplasia?

1. Degenerative disease of the hips


2. Spinal stenosis
3. Atlantoaxial instability
4. Cardiomyopathy
5. Patellar subluxation
Answer: 2

Achondroplasia is the most common type of short-limb disproportionate


dwarfism. The primary defect found in achondroplastic dwarfs is abnormal
enchondral bone formation. Periosteal and intramembranous ossification is
normal. Spinal deformities are the most common and potentially disabling
problem for the achondroplastic dwarf. The spinal canal is developmentally
narrowed, particularly in the lower lumbar segments. Stenosis of the spinal
canal and intervertebral foramen is secondary to short, thickened pedicles,
interpedicular narrowing, thickened laminae, and inferior facets. Additional
factors, such as intervertebral disc herniation, degenerative spondylolysis,
excessive lumbar lordosis, or anterior wedging of the vertebral bodies from a
thoracolumbar kyphosis, narrow the canal further.

Reference: Bassett GS: The osteochondrodysplasias, in Morrissey RT,


Weinstein SL 9eds): Lovell and Winter’s Pediatric Orthopaedics, ed 4.
Lippincott-Raven, 1996, pp 203-223.
Question 43 -

A 75-year-old man has a destructive lesion of the proximal femur with soft-
tissue extension. A technetium bone scan shows no other lesions, and a CT
scan of the chest is negative. A needle biopsy of the soft-tissue component
shows high-grade chondrosarcoma. According to the staging system of the
Musculoskeletal Tumor Society, the stage of the lesion is

1. IB
2. IIB
3. III
4. 2
5. 3
Answer: 2

Grade is an assessment of the biological aggressiveness of the lesion, which is


a blend of histology, radiographic assessment, and clinical reflection.
G0=benign, G1=low grade malignant, G2=high grade malignant
Chondrosarcoma is considered high grade malignant. The anatomic setting or
Site (T) has a direct relationship to the prognosis and the choice of surgical
procedure. These are determined primarily by clinical and radiographic
techniques. T1=intracapsular, T2= extracapsular, intracompartmental,
T3=extracapsular, extracompartmental. In most staging systems for
carcinomas metastatic involvement is stratified as either regional (N) or distant
(M). For sarcomas both have the same ominous prognosis and both are
designated (M). M0=no regional or distant mets, M1=regional or distant mets
present. The three factors of grade, site, and metastasis are combined to form
the criteria for the progressive stages of benign and malignant lesion. Benign
lesions are designated by the Arabic numerals 1,2,and 3 corresponding to
latent, active, or aggressive lesions. Stages of malignant lesions are
designated by the Roman numerals I,II, or III, and these stages are
synonymous with low-grade, high-grade, and metastatic. The three states of
sarcomas are further stratified into A or B depending on whether the lesion is
anatomically intracompartmental (A) or extracompartmental (B).
Question 48

Figure 8 shows the AP radiograph of the sacroiliac joints of a 29-year-old man


who has had increasing lower back pain and stiffness for the past 4 months.
Management consisting of exercise and nonsteroidal anti-inflammatory drugs
has provided only minimal relief. Examination reveals decreased lumbar
mobility in all directions, normal sensibility, and no weakness. The most useful
physical finding for confirming the diagnosis is

1. a positive straight leg raising test.


2. limitation of chest expansion to 1" or less.
3. the presence of urethritis.
4. bilateral tightness of the hamstrings.
5. unilateral absence of the Achilles reflex.
Answer: 2

The AP radiograph of the patient's sacroiliac joints shows sclerosis in the


bilateral SI joints indicative of sacroiliitis. This is a hallmark finding in ankylosing
spondylitis as are the rest of the symptoms described in the question. Patients
with AS often have other joint involvement such as pain from inflammation of
the costovertebral joints which inhibits chest expansion. Eventual ankylosis of
these joints may result in severe fixed limitations of chest expansion. A positive
straight leg-raising test is a tension sign that is suggestive of a herniated disc,
yet the patient has normal sensibility and no weakness in his lower extremities.
This would also exclude unilateral absence of the Achilles reflex. The presence
of urethritis would be suggestive of Reiter's. Bilateral tightness of the
hamstrings may be a finding but is not a useful finding for confirming the
diagnosis.

Reference: Frymoyer JW (ed): The Adult Spine: Principles and Practice.


Philadelphia, PA, WB Saunders, 1992, pp 699-705.
El-Khoury GY, Kathol MH, Brandser EA: Seronegative spondyloarthropathies.
Radiol Clin North Am 1996;34:343-357.
Question 63 -

Figure 13 shows the AP radiograph of both knees of a 55-year-old man who


reports an acute exacerbation of right knee pain for the past 36 hours.
Examination reveals a palpable effusion, moderate warmth, and pain on range
of motion. A synovial fluid aspiration will most likely show

1. A WBC greater than 100,000/mm3


2. Gram-negative intracellular diplococcus
3. Positive birefringent crystals
4. Negative birefringent crystals
5. Elevated complement levels
Answer: 3

Figure 13 shows chondrocalcinosis of bilateral knee menisci, which


may be caused by several disorders, including calcium pyrophosphate
deposition (CPPD), ochoronosis, hyperparathyroidisim, hypothyroidisim, and
hemechromatosis. CPPD (a.k.a. pseudogout) is a common disorder of
pyrophosphate metabolism that occurs in older patients and occasionally
causes acute attacks, usually in the lower extremities, especially the knee.
Short, blunt (rhomboid-shaped) rods that are weakly positively birefringent are
demonstrated following aspiration. Negative birefringent crystals are seen in
gouty knee aspirates. Answers 1 and 2 are seen in septic knee aspirates,
which with this patient’s symptoms, should be in the differential diagnosis.
However, the radiograph is more suggestive of pseudogout condition.
Question 74

Which of the following benign lesions may occasionally metastasize to the


lungs.

1. Nonossifying fibroma
2. Osteofibrous dysplasia
3. Chondromyxoid fibroma
4. Chondroblastoma
5. Periosteal chondroma
Answer: 4

Chondroblastoma is a rare benign neoplasm of fetal-type cartilage


differentiation. Over 50% of the lesions occur in patients between the ages of
15 and 25, with 70% occurring in the long bones. Radiologically,
Chondroblastoma is usually a well-circumscribed lytic lesion that, when in a
long bone, involves the epiphysis. Histologically, Chondroblastoma is
composed of sheets of stromal cells, scattered multinucleated giant cells, and
varying amounts of chondroid matrix. The stromal cells, resembling fetal
chondroblasts, are distinctive. This benign, slow growing neoplasm is best
treated with curettage and bone grafting. Rarely, these are locally
destructive. Another rare behavior is pulmonary metastasis, the so-called
“benign metastasizing Chondroblastoma.” Growth of these rare metastatic foci
is very slow, and they are usually treated successfully by pulmonary wedge
resections.

McCarthy EF, Frassixa FJ (eds): Pathology of Bone and Joint Disorders with
Clinical and Radiological Correlation. Philadelphia, PA, WB Saunders, 1998,
p221.
Question 95

In which of the following pediatric neoplasms can involvement of the synovium


result in arthralgias?

1. Ewing’ tumor
2. Osteosarcoma
3. Eosinophilic granuloma
4. Leukemia
5. Chondrosarcoma
Answer: 4

Bone and joint symptoms are common presenting complaints in children with
acute leukemia. These include bone tenderness, swelling, and arthralgias.
Arthralgias, which symmetrically involve multiple joints, are caused by leukemic
infiltration of the synovial membrane. Synovial involvement is minimal to
nonexistent in the other forms of cancer above.

(Pathology of Bone and Joint Disorders with Clinical and Radiologic


Correlation, 1998, 127-128; Annals of Internal Medicine, 1963)
Question 104

Which of the following devices provides the best prophylactic stabilization of an


impending pathologic fx caused by a lytic defect in the mid-diaphysis of the
femur?

1. plate and screws alone


2. plate and screws with cement
3. Rush rod with cement
4. Ender’s rods with cement
5. locked intramedullary nail
Answer: 5

Impending pathologic fx of the femoral shaft can usually be treated by


conventional closed IM rodding techniques. The use of reconstruction nails,
locked prox. And distally, has served to reduce the common complication of
progressive fem collapse with telescoping of the fx fragments and prox
migration of the rod. Prox placement of screws into the fem head offers more
secure fixation than conventional transverse or antegrade screw placement and
may protect the femoral head from subsequent fx.
Both impending lesions and complete pathologic fx of femoral head and
neck should be managed with replacement arthroplasty. Pathologic fx in
supracondylar and condylar regions of femur are unusual and difficult to tx. If
there is sufficient bone stock, use of conventional internal fixation devices
augmented with methylmethacrylate will usually achieve stability. Tx principles
are the same regardless of location. A construct should ideally provide enough
stability to allow immediate full weight bearing with
enough durability to last the patient’s expected lifetime.
Question 112 -

Figures 25a and 25b show the AP and lateral radiographs, and Figures 25c and
25d show the T1 and T2 weighted MRI scans of a 42 year old man who has a
painful mass of the ankle. Figure 25e shows a histopathologic specimen. What
is the most likely diagnosis?

1. Aneurysmal bone cyst


2. Giant cell tumor
3. Hemangioma
4. Osteomyelitis
5. Intraosseuos ganglion
Answer: 5

Inraosseous ganglions are also known as subchodral bone cysts. They typically occur in
the 2nd – 6th decades of life and are more common in males. On plain radiographs
inraosseous ganglia appear as well defined, oval or round osteolytic areas with a thin
rim of sclerotic bone. On MRI, there is a low signal on T1 and high on T2. Histological
features include stellate or fibroblast-like cells that produce a large amount of mucoid
ground substance.
Aneurysmal bone cysts are eccentric, lytic, expansile areas of bone destruction on x-ray.
The essential histologic features are cavernous blood filled spaces without an
endothelial lining. This is not seen in Fig. 25e.
Giant cell tumors reveal a purely lytic destructive lesion in the metaphysis. The basic
proliferating cell has a round or oval shaped nucleus. Numerous mitotic figures are also
seen. Hemangiomas are soft tissue tumors that can occur in a cutaneous,
subcutaneous, or intramuscular location. Plain radiographs may reveal small phleboliths.
On MRI, heterogeneous lesions with numerous small blood vessels are seen.
There multiple forms of osteomyelitis and most are easily recognized clinically,
radiographically and histologically. Most are located in the metaphyseal region and show
varying degrees of a lytic process with some degree of sclerosis.

Helwig U, Lang, Baczynski,: The intraosseous ganglion: A clinical pathologic report on


42 cases. Arch Orthop Trauma Surg 1994;114: 14-17.
M. Forest, Tomeno, Vanel, Orthopedic Surgical Pathology, 1998, pp547-552.
Question 118 -

Which of the following soft-tissue tumors has the following MRI scan signal
characteristics: high signal on T1-weighted images and intermediate to high
signal on T2-weighted images?

1. Synovial sarcoma
2. Extra-abdominal desmoid tumor
3. Malignant fibrous histiocytoma
4. Lipoma
5. High-grade pleomorphic liposarcoma
Answer: 4

Magnetic resonance imaging has become the most useful modality for
the definition of soft tissue masses. Although the MR image can detect soft
tissue masses with a very high sensitivity, it is not possible to accurately predict
the histology or whether a lesion is benign or malignant. The two exceptions to
this general rule are lipomas and hemangiomas. Lipomas often are very
homogeneous and have signal characteristics that exactly match those of the
surrounding fat, high signal on T1-weighted images and intermediate to high on
T2-weighted images, thus establishing the diagnosis.

Reference: Frassica FJ, Frassica DA: Soft tissue tumors: Diagnosis,


evaluation, and management. J AM Acad Orthop Surg 1994;2:202-211
Question 132

Figures 31a and 31b show the radiograph and biopsy specimen of a 57-year-
old man with no history of trauma who has had painful swelling of the distal
phalanx of the little finger for the past 2 months. What is the most likely
diagnosis?

1. Osteomyelitis
2. Epidermal inclusion cyst
3. Metastatic lung carcinoma
4. Metastatic kidney carcinoma
5. Metastatic prostate carcinoma
Answer: 3

“Acrometastases” is a term first used by German authors about 25


years ago to describe metastatic lesions occurring in the hands and feet. The
filtering mechanism of the lungs may partially explain why the lung tumor cells
bypass the lung-filtering activity by directly invading pulmonary vessels,
reaching the left heart and then spreading to remote regions. Therefore, lung
tumors theoretically are more likely to be associated with acrometastatic
tumors. Carcinomas of any origin can spread to bone, the most common are
carcinomas of lung, breast, prostate, kidney, or thyroid. Carcinomas of the
prostate, breast, and kidney secondary to “batson’s plexus” is a major factor in
the development of spinal metastasis. Pelvic bone metastasis of prostate
cancer is common secondary to proximity. The figures do not show the
characteristics of osteomyelitis of cortical destruction and formation of
periosteal new bone. Epidermal inclusion cysts do not pertain to figures shown.

Reference: Lombardi RM, Amadio PC: Acrometastases, in Sim FH (ed):


Diagnosis and Management of Metastatic Bone Disease: A multidisciplinary
Approach. New York, NY, Raven Press, 1988, pp. 237-243.
Question 145 -

A 73-vear-old woman has had aching pain in the right ankle region for the past
4 months. Results of serum and urine protein electrophoresis show normal
migration. Figures 34a through 34c show the plain lateral radiograph of the
talus, sagittal TI-weighted MRI scan, and biopsy specimen. What is the most
likely diagnosis?

1. Metastatic carcinoma
2. Lymphoma
3. Multiple Myeloma
4. Fibrosarcoma
5. Malignant fibrous histiocytoma
Answer: 2

The referenced films show an x-ray with a lytic lesion and periosteal elevation
of the talus, an MRI with decreased signal on T1, and a photomicrograph with
abundant mononuclear cells in various stages of division. Given the negative
SPEP/UPEP, the only possible correct response is Lymphoma. Bone tumors
caused by Lymphoma have the above characteristics and are mostly found in
the Femur, Pelvis, and Skull. Involvement of the talus is rare.

Reference: Huvos, Bone Tumors 2d Edition, 625-637


Question 157 -

Figures 38a through 38c show the plain radiograph and coronal T1- and T2 -
weighted MRI scans of a 30-year-old man who has a painful lump over the
distal deltoid on the lateral aspect of his arm. The mass is marginally excised,
and a histologic section at the periphery of the mass is shown in Figure 38d.
Management should include

1. observation.
2. serial follow-up staging studies.
3. radiation therapy.
4. wide reexcision, followed by radiation therapy.
5. chemotherapy.
Answer: 1

Osseous soft tissue tumors are either extraskeletal osteosarcomas or myositis


ossificans. If they specifically show you the periphery of the biopsy specimen,
you can bet it will be myositis ossificans because that is where the
characteristic features of trabecular osteoid, rimming osteoblasts and remnants
of the muscle tissue will be found. The osteoscarcoma would have the osteoid
in the center and be much more disorganized. So once you've figured out this is
MO, you know to just watch it. If you excise it, it will often recur, so don't bother.
There is nothing to stage, and no adjuvant therapy is recommended for this
benign process.

Enzinger FH, Weiss, SW: Osseous soft tissue tumors, in Enzinger, FM, Weiss
SW (eds): Soft Tissue Tumors, ed 3. St Louis, MO, Mosby, 1995, pp
1013-1037.

Huvos, AG: Miscellaneous tumors of soft tissue and bone, in Huvos AG (ed):
Bone Tumors. Diagnosis, Treatment and Prognosis, ed 2. Philadelphia, PA, WB
Saunders, 1991, pp 745-753
Question 161 -

A 42-year-old woman has had persistent aching pain in her right arm for the
past 9 months that now awakens her from sleep. Management consisting of
oral narcotics has failed to provide relief. Figures 39a and 39b show the plain
radiographs of the right humerus, and Figure 39c shows the biopsy specimen.
What is the most likely diagnosis?

1. Chondrosarcoma
2. Ewing’s sarcoma
3. Enchondroma
4. Osteosarcoma
5. Malignant fibrous histiocytoma
Answer: 1
The radiographs show a long bone diaphyseal intramedullary process, maybe
some lytic stuff going on inferiorly, calcification in the proximal portion, some
erosion into the endosteal surface on the close-up, no real periosteal reaction,
and no cortical expansion- The micrograph looks kind of like cartilage, with cells
and matrix, and the cells don't look too nasty- don't see giant cells or
multinucleated cells all over the place. The other key feature is the process is
painful. This is all typical of chondrosarcomas. Ewing's typically have periosteal
reaction, and the biopsy should be more cellular. Enchondromas are typically
not painful, they don't erode endosteal surfaces, and the biopsy would be less
cellular. The radiograph doesn't look anything like an osteosarcoma- not
aggressive enough, no periosteal reaction, etc. MFH typically won't have the
lytic areas, and the micro would be very different- much more cellular, lots of
atypia, giant cells.

Unni KK: Chondrosarcoma (primary, secondary, dedifferentiated, and clear


cell), in Unni KK (ed): Dahlin's Bone Tumors: General Aspects and Data on
11,087 cases, ed 5. Philadelphia, PA, Lippincott-Raven 1996, pp 71-108.

Wold LE, McLeod RA, Sim FH, Unni KK (eds): Atlas of Orthopedic Pathology.
Philadelphia, PA, WB Saunders 1990, Ch 3, 10, 15, 23, 35.
Question 166 -

Figures 41a through 41c show the axial T1- and T2-weighted images and the
biopsy specimen of a 53-year-old patient who has a painless mass of the left
proximal thigh. A radiograph and CT scan of the chest and a bone scan are
normal. Management should consist of

1. radiation therapy and chemotherapy


2. radiation therapy and wide excision
3. radiation therapy and debulking
4. wide excision alone
5. marginal excision
Answer: 2

Using the TNM staging system, tumors are staged as I (low-grade G1), stage II
(intermediate), stage III (high grade), and any tumor with spread (mets) is Stage
IV. Using the AJCC system: tumors < 5 cm are T1, those > 5 cm are T2; Any
patient with a soft tissue mass should receive a CXR to initially screen. Bone
scans or CT’s are used to detect metastases. Biopsy specimens are used to
determine the type of soft tissue mass. This biopsy revealed fibroblastic type of
nuclei, nuclei of various shapes and sizes, and occasional mitotic figures. The
screening exams were negative for metastasis. The MRI revealed the mass to
be fibrous (~ muscle on T1, bright on T2). Radiation therapy (followed by wide
excision) is necessary for large tumors to improve chances for local control.
Surgery is the mainstay for treating soft tissue sarcomas. Radiation therapy
can be done either pre-operatively or post-operatively, however, pre-operatively
it will shrink the tumor mass, and often cause the formation of a pseudocapsule
around the mass, helping to further isolate/localize the tumor and reduce the
amount of tissue resected (permitting limb-sparing surgery).

Chang, et al: Clinical evaluation and treatment of soft tissue tumors, in


Enzinger and Weiss: Soft-Tissue Tumors, ed 3., 1995, pp 17-38
Question 176 -

Figure 45 shows the AP radiograph of the pelvis of a 42-year-old woman who


reports the insidious onset of anterior pelvic pain over the past 2 years. History
reveals that she had a hysterectomy 7 years ago. Examination reveals
localized tenderness over the symphysis pubis, and laboratory studies show an
erythrocyte sedimentation rate of 2 mm/hr (normal up to 20 mm/hr).
Microscopic examination of the involved area would most likely reveal

1. osteomyelitis.
2. osteomalacia.
3. osteonecrosis.
4. neoplasia.
5. chronic inflammation.
Answer: 5

5 Ref. JBJS 1989;71A:358-364

Osteitis pubis is a painful, inflammatory, but non-infectious condition involving the


periosteum, bone, cartilage, and ligamentous structures in the region of the symphysis
pubis (2,7,28). Because of the variety of pathological changes with which osteitis pubis
may be associated, there has been considerable confusion in the literature as to the
precise cause of the disease. In 1924, Beer{3 } first described osteitis pubis, which
developed after urological procedures. Since that time, numerous cases have been
reported in association with pregnancy, rheumatological disorders, trauma, infection,
and athletic exertion, as well as after urological and gynecological operations.
In the referenced article on patients with osteitis pubis: all had localized tenderness to
palpation over the symphysis pubis, and anteroposterior radiographs, made with the
patient standing, showed the bilateral involvement of the symphysis pubis that is
characteristic of osteitis pubis. In the four patients who had been first seen less than six
months after the onset of symptoms, the radiographs showed rarefaction and erosion of
the adjacent pubic bones forming the symphysis and widening of the symphyseal joint
space . In the six patients who were first seen more than six months after the onset of
symptoms, the radiographs showed sclerosis of the adjacent pubic bones and narrowing
of the symphyseal joint space.
Question 177 -

A 72-year-old man reports a progressive increase in pain in his right hip, where
he has had very mild aching pain for several years. Figure 46a shows the plain
AP radiograph of the pelvis, and Figures 46b and 46c show the coronal T1- and
T2-weighed MRI scans. A biopsy specimen from the femoral neck region is
shown in Figure 46d. What is the most likely diagnosis?

1. Dedifferentiated chondrosarcoma
2. Telangiectatic osteosarcoma
3. Pagetoid osteosarcoma
4. Metastatic carcinoma
5. Bone infarct-associated osteosarcoma
Answer: 3
Explanation: AP radiograph and MRI of the pelvis show a lytic lesion involving the entire Right femoral head and
neck. Biopsy specimen shows a highly cellular bony specimen with irregular thickened bony trabeculae. The
bone marrow is replaced by vascular fibrous tissue with very irregular appearing cells. This specimen shows
some evidence of the classic mosaic pattern of Pagetoid bone.
Sarcomatous transformation in Paget’s disease occurs in 0.9% to 5.5% of cases, most commonly in Males (2 : 1),
with an average age of 67 (42-78). Most commonly involving the pelvis, femur, humerus, and skull with most
lesions occurring in the metaphyseal-diaphyseal region. These patients typically present with acute pain or a
progressive increase in the intensity of chronic pain and swelling or associated soft tissue mass. Pathologic
fracture is the initial presentation in 10-37% of the cases AP radiograph and MRI typically show a lytic lesion with
irregular margins and cortical penetration often with an associated soft tissue mass. Biopsy, which is required for
definitive diagnosis, shows bony trabeculae with the typical mosaic pattern and enlarged hyperchromic and
irregular nucleated cells. Multinucleated giant cells are also typically seen.
Telangiectatic osteosarcoma – Typically occurs in the 2nd decade of life. Radiographs show purely lytic lesion with
no sclerosis. Biopsy of the lesion shows minimal osteoid production with spaces separated by Septa as seen in
an aneurysmal bone cyst.
Bone infarct-associated osteosarcoma – Typically occurs in the 1st or 2nd decade, and presents with acute onset
of pain and swelling. The roentgenograms typically show a well-circumscribed lesion with mineralization at the
periphery and irregularly mineralized soft tissue masses. Biopsy shows minimal osteoid production.
Dedifferentiated chondrosarcoma – Radiographs show dense mineralization surrounded by poorly defined lytic
lesions. Biopsy shows hyaline cartilage surrounded by cellular neoplasm.
Metastatic carcinoma – Radiographs demonstrate irregular destructive lesions, mostly osteolytic with the biopsy
showing glandular arrangements of epithelial cells in the stroma. Patients may have a know primary cancer and
often present with systemic signs and symptoms of metastasis.

References: Hadjipavlou A, Lander P, Srolovitz, et al: Malignant transformation in Paget’s disease of Bone.
Cancer 1992;70:2802-2808.
Unni KK; Osteosarcoma, in Unni KK (ed): Dahlin’s Bone Tumors: General Aspects and Data on 11,087 cases, ed
5. Philadelphia, PA, Lippincott-Raven, 1996, pp 143-183.
Question 188

A 12-year-old girl sustained several puncture wounds to her palm from cat
bites. Twenty-four hours later the hand is swollen, erythematous, and painful.
What organism is most likely associated with this infection?

1. Eikenella corrodens
2. Proteus mirabilis
3. Streptococcus
4. Staphylococcus aureus
5. Pasteurella multocida
Answer: 5

The acute onset of cellulitis, lymphangitis, and serosanguineous or purulent


drainage from hand wounds 12-24 hours after cat or dog bites should suggest
Pasteurella multocida. P. multocida has been isolated in oral cavities of up to
70% of domestic cats and 66% of domestic dogs.
The oral drug of choice for animal bites is Augmentin.

Reference: Aron MS, Fernando L: Pasteurella multocida: The major cause of


hand infections following domestic animal bites.
Lucas GL, Bartlett DH: Pasteurella multocida infection of the hand. Plat
Reconstr Surg 1981; 67:49-53.
Question 189 -

Which of the following is considered the most common presenting symptom in a


patient with a high-grade primary malignant bone tumor?

1. Weight loss
2. Fatigue
3. Fever
4. Pain
5. Malaise
Answer: 4

The most common presenting symptom in people with high grade


primary malignant bone tumor is pain. Weight loss, fatigue, fever and malaise
are symptoms seen with malignancy. However they are usually not the
presenting symptom encountered in primary malignant bone tumor.

Source: McCarthy EF, Frassica FJ (eds): Pathology of Bone and Joint Disorder
with Clinical and Radiological Correlation. Philadelphia, PA, WB Saunders,
1998, p197.
Question 193 -

Which of the following organisms can produce gas gangrene and may
necessitate open amputation above the level of infection?

1. Pseudomonas aeruginosa
2. Staphylococcus aureus methicillin-resistant
3. Staphylococcus aureus coagulase-negative
4. Clostridium perfringens
5. Group D enterococcus
Answer: 4
DeHaven KE, Evarts CM: The continuing problem of gas gangrene: A review
and report of illustrative cases. J Trauma 1971;11:983-991.
MacLennan has defined gas gangrene as an invasive, anaerobic infection of
muscle, characterized by extensive local edema, massive tissue necrosis, variable
degrees of gas production, and profound toxemia.
The bacteriology of his condition has been well studied. There are six species
of clostridia capable of producing gas gangrene (table 1). The most important of which is
C. perfringens. These organisms are Gram-positive rods that seldom produce spores in
the tissues or in culture media. They are obligate anaerobes, and cannot multiply in the
relatively high oxidation-reduction potentials found in healthy tissues. Clostridia are
widely distributed in nature, being found in soil and in feces of most animals, including
man.

Bacteriology
A. Organisms B. Toxins

Clostridium perfringens (C. welchii) Lecthinase (toxin )


C. novyi Collagenase
C. septicum Hyaluronidas
C. histolyticum Leukocidin
C. bifermentans Deoxyribonuclease
C. fallax Protease
Lipase
Laggaard SW, McElfresh EC,Premer RF: Gangrene of the upper extremity in
diabetic patients. J Bone Joint Surg 1989;71A:257-264.
Twenty-two patients who had diabetes mellitus and needed an amputation for gas
gangrene in an upper extremity at an age of fifty-one years were identified and
followed…
Oe patient was not included in the study because the gangrene had been caused
by an aggressive infection with Clostridium perfringens…
Gangrene was thought to have developed spontaneously in seven upper
extremities, as the patients were unable to recall an injury or infection that preceded
the gangrene. Seven patients associated the development of gangrene with an
episode of minor trauma: a cracked nail in two patients and a sliver, blister, fall,
bump, and blow from a hammer in one patient each. The gangrene was preceded
by spontaneous infection in eight patients, although the infecting organisms, which
were always multiple, were identified in preoperative or intra-operative cultures in
only four of these eight patients. The organisms that were identified as an initial
infection were Staphylococcus (3 infections), Streptococcus (2 infections, Proteus
(2 infections), Enterobacter (1 infection) and Veillonella (1 infection). In patients
who did not have an initial infection, the cultures of secondary infections grew
Staphylococcus twice, Proteus once, and Corynebacteria once.
The results of amputation of the upper extremity were very unsatisfactory. In only
two of the twenty-two patients did the surgical wound heal at the level that was
initially selected, without additional surgical intervention…
Question 200 -

Which of the following soft-tissue sarcomas most commonly shows prominent


mineralization within the lesion on plain radiographs?

1. Rhabdomyosarcoma
2. Liposarcoma
3. Epithelioid sarcoma
4. Synovial sarcoma
5. Angiosarcoma
Answer: 4

“Amorphous calcifications are frequently present (up to 30% of


patients) in synovial sarcomas and thus this entity should always be considered
in the differential diagnosis of a soft tissue mass that contains amorphous
calcification.” Calcification can be observed in benign soft tissue lesions
including nodular (pseudosarcomatous) fasciitis and myositis ossificans.
Question 207 -

Figures 55a through 55c show the AP radiograph and T1- and T2-weighted axial
MRI scans of an 8-year-old boy who has a painless mass in the right
antecubital space. The mass increases in size with activity and then decreases
after activity. Gross blood is obtained during a needle biopsy. Management
should include

1. wide resection.
2. observation.
3. open incisional biopsy.
4. CT scan-guided core biopsy.
5. radiation therapy.
Answer: 2

A hemangioma may be present at birth or may exist initially as a small red spot.
Within several months these lesions undergo rapid growth and proliferation and
usually reach maximal size by two to three years of age. Lesions within the
hand and wrist are usually circumscribed and have no predominant location. By
age five, spontaneous involution is underway. This process is heralded by gray
areas of lighter pigmentation called "herald spots." Most of these lesions have
the appearance of a "strawberry" if dermis is involved, but many deeper lesions
will show few skin changes. CT scan will demonstrate hemangiomas to be
homogeneous lesions with well-defined borders. MRI demonstrates depth of
structures as well as distinguishing these lesions from sarcomas and capillary
malformations. Treatment is conservative. Compression garments for large
lesions and reassurance of parents remains the treatment of choice. Earlier
surgery is reserved for serious obstructions of the aerodigestive tract,
obstruction of vision, gastrointestinal bleeding, and other life-threatening
conditions. Early surgery is rarely indicated in the upper extremity.

OKU: Hand Surgery Update, Chapter 36: Pediatric Hand Tumors


Question 212

Which of the following antibiotics is contraindicated in patients who require oral


anticoagulants?

1.Trimethoprim
2.Clindamycin
3.Ciprofloxin
4.Aminoglycoside
5.Erythromycin
Answer: 5

Erythromycin interacts with oral anticoagulants to increase prothrombin


time. Sulfamethoxazole, Fluconazole, Metronidazole and Rifampin also
increase prothrombin time. Trimethoprim combined with methotrexate is
associated with megaloblastic pancytopenia. Clindamycin enhances effects of
neuromuscular blocking agents. Ciprofloxacin increases theophyline levels
while its absorption is decreased by antacids, iron and zinc salts, and Carafate.
Aminoglycoside’s nephro- and ototoxicity is enhanced when combined with
diuretics. Aminoglycides enhance respiratory suppression of neuromuscular
blocking agents.

Reference: Kasser JR (ed): Orthopaedic Knowledge Update 5. Rosemont, IL,


American Academy of Orthopaedic Surgeons, 1996, ppp149-161
Question 235 -

Figures 59a through 59c show the radiograph and biopsy specimens of an
11-year-old child who has distal thigh pain. What is the most likely diagnosis?

1- Nonossifying fibroma
2- Aneurysmal bone cyst
3- Intraosseous ganglion
4- Periosteal chondroma
5- Telangiectatic osteosarcoma
Answer: 2

An aneurysmal bone cyst is a solitary, expansile lesion of unknown etiology


which is generally eccentric in location. These lesions are most commonly seen
in individuals under 20 years of age, where swelling, pain, and/or tenderness
may be the presenting complaint. The periphery of the lesion is often indistinct,
and the tumor itself often has a trabeculated appearance. On histologic
examination, the lesion is found to contain cystic spaces of different sizes which
are filled with blood but are not lined with a vascular endothelium. Between the
blood filled spaces are fibrous septa containing giant cells and foci of immature
bone or osteoid.

Bullough, PG: Orthopaedic Pathology, ed 3. Chicago, IL, Mosby-Wolfe, 1997,


pp. 402-404.
Question 243

A 12-year-old boy has pain at rest and at night over the proximal tibia. Plain
radiographs show a 6mm lucent area in the cortex surrounded by sclerotic
bone. What is the most likely diagnosis?

1. High-grade osteosarcoma
2. Periosteal osteosarcoma
3. Osteoblastoma
4. Osteoid osteoma
5. osteofibrous dysplasia
Answer: 4

Osteoid osteoma is a benign tumor of bone that affects 5-20-year-olds.


It has a male to female ratio of 3/1. Patients complain of night pain, relieved
with ASA, NSAIDS or prostaglandin inhibitors. X-ray findings are consistent
with the test question, that is a well circumscribed radiolucent nidus that is less
than 2 cm in diameter. The lesion is typically surrounded by dense reactive
bone.
The other choices are incorrect, not consistent with the history.
Osteoblastoma would have a similar picture on X-ray except the
lucency tends to be greater than 2 cm. It also lacks night pain.
Periosteal osteosarcoma and high-grade osteosarcoma typically
present with a painful, palpable mass, both with distinct X-ray findings.
Osteofibrous dysplasia usually affects girls less than 10-years-old.
Pain is usually absent unless a pathological fracture occurred. May see an
enlarged tibia with anterolateral bowing. On X-ray one will see a radiolucent,
expansible and often bubbly appearance with well-defined margins in the
metadiaphyseal anterior cortex of the tibia.
Question 251 -

Figures 64a and 64b show the plain frog lateral radiograph and axial MRI scan
of a 7-year-old child who has had left groin pain and a limp for the past 3
months. A biopsy reveals an aneurysmal bone cyst. Management should
include

1. observation with serial radiographs.


2. hip spica casting.
3. curettage and bone grafting.
4. curettage and cementation.
5. en bloc wide excision.
Answer: 3

The patient’s plain x-rays show an aneurysmal bone cyst of the left
proximal Femur. In light of the patient’s 3 month history of groin pain and limp
for 3 months, surgical curettage and bone grafting is indicated.

Ref: Cancer, 1992; 2921-2931.


Question 257 -

Figures 65a and 65b show the hematoxin eosin stains of the biopsy specimen,
and figures 65c and 65d show the T1 and T2 weighted axial MRI scans of a 36
year old woman who has a painless mass in the posterior thigh. Management
should include:

1. marginal excision
2. wide excision
3. wide excision and radiation therapy
4. radiation therapy alone
5. amputation
Answer: 1

Lesion in figures are most likely representative of neurilemmomas


which are most commonly found in extremities. These lesions are usually
treated by excision with preservation of nerve and major trunk.
Question 267

Figures 71a and 71b show the plain AP radiograph and coronal T1-weighted
MRI scan of a 75 year old patient who has had progressively worsening pain in
the left hip for the past 6 months. Figure 71c shows the incisional biopsy
specimen. Staging studies show localized disease. Surgical treatment should
include:

1. prophylactic fixation
2. curettage and cementation
3. curettage and bone grafting
4. en bloc wide excision
5. hemipelvectomy
Answer: 4

The plain films, MRI, and biopsy clearly show a chondrosarcoma.


“Low grade peripheral lesions (chondrosarcoma) have almost no risk of
metastasis and an extremely low recurrence rate when a wide margin is
achieved”

Reference: Musculoskeletal Tumor Surgery. 1983. p 963-4. Enneking, WF,


ed. Churchill Livingstone, New York, NY
OITE 2000

14. An otherwise healthy 10-year-old boy has an erythematous, painful epitrochlear


lymph node after visiting a relative with cats 3 days ago. Which of the following
organisms is the most likely cause of this problem?

1- Bartonella henselae
2- Mycobacterium marinum
3- Eikenella corrodens
4- Blastomycosis dermatitidis
5- Pasteurella
Answer: 1

Bartonella henselae is one of four Bartonella species that have been shown to be pathogenic for humans. B. henselae is
the cause of cat-scratch fever, a localized lymphadenopathy in a person who reports being in contact with or being
scratched by a cat. The pathologic response is generally a granulomatous, suppurative, extracellular and intracellular one,
self-limited, mild and usually unresponsive to antibiotic treatment. The diagnosis is made by biopsy of the involved
lymph node and a positive skin test with sterile pus from the biopsy.
The orthopaedic reference (Laskin and Potenza) presents two cases of cat-scratch
fever that presented as a soft tissue mass in an otherwise healthy child. A tentative diagnosis of an atypical soft tissue
mass was entertained, but open biopsy confirmed an infected, granulomatous process. Thus, an infectious process must
always be entertained in the differential diagnosis of a soft tissue mass work-up.
Mycobacterium marinum causes “swimming pool granuloma,” ulcerating, granulomatous lesions that occur in the skin at
the site of abrasions incurred at swimming pools. The natural habitat is both fresh and salt water, with the treatment
being tetracycline.
Eikenella corrodens is a gram-negative rod found in the human mouth flora. Although not the most common organism
(Alpha-hemolytic strept. and S. aureus are the most common) isolated from fight bite injuries (7-29%), it must be
covered in the antibiotic regimen (i.e. Augmentin).
Blastomycosis dermatitidis is a mold in the soil, endemic in North and Central America causing respiratory tract
infections, but is usually asymptomatic and rarely recognized. Disseminated disease may result in ulcerative granulomas
of skin, bone or other sites, with Ketoconazole the drug of choice. Pasturella multocida is a gram-negative rod found in
the mouth flora of dogs and cats. About 26% of animal bites become infected with the organism (alpha-hemolytic strept
most common 46%). Penicillin or augmentin remains the antibiotic of choice.

References:
Bass JW, Vincent JM, and Person DA. The expanding spectrum of Bartonella infections: II. Cat-scratch disease. Pediatr
Infect. Dis J. 1997: Feb;16(2):pp 163-79.

Laskin RS and Potenza AD. Cat Scratch Fever—a confusing diagnosis for the orthopaedic surgeon. JBJS 1971; Vol 53-
A, no.6, Sept. pp.1211-1214.
33. Improvement in hip range of motion following cemented total hip arthroplasty in
patients with ankylosing spondylitis has been found to be limited by

1- infection.
2- neurologic involvement.
3- heterotopic ossification.
4- soft-tissue contractures.
5- ankylosis of the lumbar spine.
Answer: 3

Arthritis of the hip joint occurs in approximately 30% of patients with ankylosing
spondylitis and, when present, it is usually bilateral. Cemented total hip arthroplasty has
been shown to provide excellent relief of pain and improved ambulatory capacity.
However, the improvement in the total range of motion of the hip was limited in several
series due to the high incidence of heterotopic ossification. In one series of 29 hips, severe
heterotopic ossification occurred in 23% of the hips. Prophylaxis against heterotopic
ossification by means of either low-dose radiation or indomethacin therapy is generally
recommended.

References:
OKU: Hip and Knee Reconstruction, 1996, pp 79-86.
46. Which of the following processes is related to osteofibrous dysplasia
(Campanacci's disease)?

1- Nonossifying fibroma
2- Osteoid osteoma
3- Adamantinoma
4- Fibrosarcoma
5- Ollier's disease (multiple enchondromatosis)
Answer: 3

Osteofibrous dysplasia and adamantinoma are uncommon lesions that usually occur in the
tibia. They were considered unrelated until the authors of recent publications suggested
that osteofibrous dysplasia is either the benign counterpart of a neoplastic process that
produces adamantinoma or that it is the residual of a spontaneously regressing
adamantinoma. Alternate possibilities include osteofibrous dysplasia is a precursor to and
may become adamantinoma, this, based on a few isolated cases of osteofibrous dysplasia
apparently becoming adamantinoma. The importance of understanding their relationship
is significant because observation is the recommended treatment for a patient with
osteofibrous dysplasia.

References:
Springfield DS, Rosenberg AE, Mankin HJ, Mindell ER,: Relationship between
osteofibrous dysplasia and adamantinoma. Clin Orthop 1994; 309:234-244.

Bridge JA, Dembinoski A, DeBoer J, Travis J, Neff JR: Clonal chromosomal abnormalities
in osteofibrous dysplasia: Implications for histopathogenesis and its relationship with
adamantinoma. Cancer 1994;73:1746-1752.
54. Which of the following soft-tissue lesions is best described as a Musculoskeletal
Tumor Society stage 3 lesion (aggressive)?

1- Nodular fasciitis
2- Lipoma
3- Malignant fibrous histiocytoma
4- Fibromatosis (extra-abdominal desmoid)
5- Giant cell tumor of the tendon sheath
Answer: 4

MSTS stage 3 lesion is a benign aggressive lesion. Therefore one can exclude malignant
fibrous histiocytoma. A giant cell tumor is an aggressive lesion but not a giant cell tumor
of the tendon sheath. An extra-abdominal desmoid is the most locally invasive of all
benign soft tissue tumors. The lesion infiltrates adjacent tissues. Therefore, it is the
answer to this question.
79. A 28-year-old woman has had intermittent aching pain in the left ankle for the
past year that is exacerbated by activity. Figures 17a through 17e show the plain
radiograph, the coronal T1-weighted MRI scan, the axial T2-weighted MRI scan,
and low- and highpower photomicrographs. What is the most likely diagnosis?

1- Giant cell tumor


2- Osteochondroma
3- Enchondroma
4- Chondromyxoid fibroma
5- Chondroblastoma
Answer: 4

Chondromyxoid fibroma usually presents as an eccentrically placed lytic lesion with well
defined margins in the metaphysis of the lower extremity. The lesion usually has a sclerotic
margin of bone. Usually presents in the 2-3 decade. These are all clues to the diagnosis,
but as Dr. Mott says the proof is in the pudding, and the histology gives the diagnosis.
Histologically, nodules of cartilage are found between nests of fibromyxoid areas. The
chondrocytes are often spindle shaped.

References:
Scarborough MT, Moreau G: Benign cartilage tumors. Orthop Clin North Am
1996;27:583-589.
82. A 10-year-old girl has had knee pain for the past 3 months. History reveals that
an incidental knee radiograph obtained 2 years ago showed no skeletal
abnormalities. Current plain radiographs and a biopsy specimen are shown in
Figures 18a through 18c. The patient's current condition is most likely associated
with

1- familial infantile retinoblastoma.


2- multiple hereditary osteochondromatosis.
3- multiple enchondromatosis.
4- polyostotic fibrous dysplasia.
5- Gaucher's disease.
Answer: 1

Retinoblastoma gene mutation is associated with development of osteosarcoma. The


radiographs are consistent with osteosarcoma. The plain films show an aggressive lesion
in the distal femur. As Dr. Mott says all lesions around the knee in the 2nd decade of life is
osteosarcoma until proven otherwise. Histology confirms the diagnosis ass it shows
tumor cells producing osteoid. A two step answer as 1st recognize the osteosarcoma
diagnosis, then make the connection to retinoblastoma. RB gene is a tumor suppressor
gene and when both alleles mutate, thus decreases tumor suppression and makes the
subject more susceptible to oncologic sequelae.

References:
Hansen MF: Molecular genetic consideration in osteosarcoma. Clin Orthop
1991;270:237-246.
87. A 62-year-old man has had an enlarging painless mass on his thigh for the past 4
months. A biopsy specimen and MRI scans are shown in Figures 19a through
19c. This type of tumor will most often metastasize to which of the following
structures?

1- Bone
2- Lungs
3- Liver
4- Lymph nodes
5- Kidney
Answer: 2

Soft tissue tumors metastasize to the lungs. You don’t really even need to look at the MRI and
photomicrograph. You don’t need to waste any precious brain cells figuring out which type of
tumor it is, except to realize it is big (more than 5cm), and since it is rapidly growing it will be
malignant. I couldn’t find the soft tissue tumor book they referenced, but another tumor book by
Enzinger and Weiss tells us that “by far the most frequent site of metastasis is the lungs”. (page 22)
There are two series in the journal Cancer that have 26/36 (72%) and 69/77 (90%) of MFH mets
show up in the lungs. (Bertoni, 1985 and Pezzi, 1992) The lymph node answer is the one to trip you
up, but most of the tumors we will be faced with don’t do nodal mets as much as pulmonary.
(Epithelioid sarcomas, embryonal rhabdomyosarcomas, angiosarcs and melanoma do nodes- you
know enough from the pictures that the tumor isn’t one of those. Just remember lungs for all
ortho tumor mets and you’ll be safe.

References:
Chang AE, Sondak VK: Clinical Evaluation and Treatment of Soft Tissue Tumors (ch 2) and
Malignant Fibrous Histiocytoma (ch 15), in Enzinger FM and Weiss SW (eds): Soft Tissue Tumors,
3rd ed. St. Louis, Mosby, 1995, pp17-23 and pp351-69.

Bertoni, F, Capanna R, Biagini R, Bacchini P, et al, Malignant Fibrous Histiocytoma of Soft Tissue.
Cancer 1985; 56:356-67.

Pezzi CM, Rawlings MS, Esgro JJ, Pollock RE, Romsdahl MM, Prognostic Factors in 227 Patients
with Malignant Fibrous Histiocytoma. Cancer 1992; 69:2098-2103.
92. A 52-year-old woman with no history of malignancy has had mild aching pain in
the left tibia for the past 2 years. Examination reveals a firm mass in the
subcutaneous border of the tibia; a bone scan shows this to be an isolated finding.
Figures 20a through 20d show the plain radiograph, the sagittal Tl-weighted MRI
scan, the axial T2-weighted MRI scan, and the biopsy specimen. Management
should now include

1- serial radiographic observation.


2- curettage and bone grafting.
3- wide en bloc excision.
4- chemotherapy and wide en bloc excision.
5- prophylactic stabilization and radiation therapy.
Answer: 3

Whenever I hear about adamantinoma I think of Adam and the Ants, the punk rock group
from the1980’s. Anyway. Must differentiate OFD / fibrous dysplasia / adamantinoma.

Adamantinoma is treated with wide en bloc excision. Mets can occur in up to 30 percent
of cases which can lead to death (tends to occur in cases where incomplete resections were
performed). Does not respond to chemotherapy or radiation therapy.
OFD treatment is close observation. After the age of ten, the lesion should be biopsied
and extensive histological study should be done to exclude the presence of epitheloid
components, even if the radiological findings are highly suggestive for OFD. Large
specimens need to be taken at biopsy to avoid missing tumour cells (no needle biopsy).
93. Normal cortical bone has which of the following MR signal characteristics?

1- Low on T1-weighted images and low on T2-weighted images


2- Low on T1-weighted images and high on T2-weighted images
3- Moderate on T1-weighted images and low on T2-weighted images
4- High on T1-weighted images and low on T2-weighted images
5- High on Tl-weighted images and high on T2-weighted images
Answer: 1

The following is the chart that needs to be memorized!

Tissue T1-weighted T2-weighted


Fat high intermediate to high
Articular cartilage intermediate high
Fluid (joint, CSF) low high
Cortical bone low low
Tendon/ligament low low
/fibrocartilage
Hemosiderin low very low

References:
Beaty JH (ed): Orthopedic Knowledge Update 5. Rosemont, IL, American Academy of
Orthopedic Surgeons, 1999, pp 81-87
106. Figure 28 shows the AP radiograph of a 65-year-old woman who has mild
shoulder pain and anemia. History reveals no significant illnesses or injuries. What
is the next step in evaluation?

1- MRI scan
2- Serum protein electrophoresis
3- Serum alkaline phosphatase studies
4- Open biopsy
5- Needle biopsy
Answer: 2

This patient has radiographs consistent with a diagnosis of multiple myeloma. It is a


neoplasm composed of plasma cells showing various degrees of differentiation. The
process is usually multi-centric and diffusely involves the bone marrow. Increasing pain is
the most common patient complaint, and is most often centered in the thoraco-lumbar
spine. Most patients will also complain of weakness and weight loss. Pathologic fracture
is common, again most commonly in the vertebral column. Diagnosis is confirmed by
serum electrophoresis revealing a single monoclonal protein spike. The presence of
Bence-Jones proteins in the urine and moderate to severe anemia are other common
findings. Radiographically, these patients demonstrate multiple osteolytic, “punched out”
lesions throughout the axial and appendicular skeletons. Bone scan is not helpful as these
lesions appear cold secondary to a predominance of osteoclastic activity. An MRI would
be superfluous; this is a plain film diagnosis. Biopsy is not indicated. Serum alkaline
phosphatase elevation may be present in multiple neoplastic conditions, but not
specifically in multiple myeloma.

References:
Unni KK: Dahlin’s Bone Tumors, ed 5, Philadelphia, PA, Lippincott-Raven, 1996, pp
225-236.
111. Which of the following bone tumors is typically multifocal and involves bones in
the same extremity?

1- Osteoblastoma
2- Osteosarcoma
3- Chondrosarcoma
4- Chondroblastoma
5- Hemangioendothelioma
Answer: 5

Hemangioendothelioma (Hemangiosarcoma) is a rare malignant vascular tumor of bone. Up to 1/3


of these patients will have multifocal lesions, which are predominantly lytic. This lesion can occur in
any age group and usually presents with pain. Osteoblastoma is a benign bone-producing tumor,
typically appearing in the spine, hip and proximal humerus. Radiographically, the bone destruction
appears moth-eaten. Osteosarcoma is a malignant spindle cell neoplasm that produces osteoid.
There are several variants of osteosarcoma including: intramedullary (classic), parosteal, periosteal,
telangiectatic, occurring with Paget’s, and post irradiation. The most common is high-grade
intramedullary osteosarcoma, which presents with pain about the knee of a child or young adult. Of
these, 90% present with a stage IIB lesion (i.e. high-grade extra-compartmental, no metastases) and
10-20% have pulmonary metastases. Chondrosarcoma is malignant neoplasm of cartilage occurring
in adults and older age groups. They usually present with pain or a mass in the knee, spine, shoulder
and pelvis girdles. Chondroblastoma is a benign cartilage tumor centered in the epiphysis, triradiate
cartilage of the pelvis and apophysis of young people (i.e. open physes). These patients also present
with pain.

References:
Dorfman HD, Czerfniak B: Bone Tumors. St. Louis, MO, Mosby, 1998, pp 369-370.

McCarthy EF, Frassica FJ: Pathology of Bone and Joint Disorders. Philadelphia, PA, WB Saunders,
1998, p 267.
120. Figure 33 shows the AP radiograph of the femur of an asymptomatic 14-year-old
patient. What is the most likely diagnosis?

1- Ollier's disease
2- Paget's disease
3- Fracture malunion
4- Residual proximal femoral focal deficiency
5- Fibrous dysplasia
Answer: 5

Figure 33: AP radiograph of proximal femur with large ground glass lesion extending distal to
femoral neck. It has a well defined rim of sclerotic bone. There appears to be a slight varus bowing
to the proximal femur possibly developing into a "Shepard's Crook Deformity."
This is a radiographic diagnosis, in this case Fibrous Dysplasia. Fibrous Dysplasia is a developmental
abnormality of bone which is either mono or polyostotic. When associated with skin (yellow or
brown patches) and endocrine abnormalities (precocious puberty) in addition to multiple bone
lesions the diagnosis of McCune-Albright syndrome can be made. Patients are usually asymptomatic
the lesions being found incidentally on Xray. By far the most common location is the proximal
femure. Radiographically Fibrous Dysplasia usually shows well defined zones of rarefaction (highly
lytic or ground glass appearing), often surrounded by narrow rim of relatively sclerotic bone. These
lesions may expand and thin the cortex especially in thinner bones. These lesions may contain large
amounts of cartilidge which may show ring-like or dot-like calcification especially around femoral
neck. These lesions can appear aggressive and suggesting diagnois of sarcoma.
As for the other answers Ollier's Disease is associated with multiple enchondromas which have a
different radiographic appearance, Paget's disease as well is entirely differnet having characteristic
coarse trabeculae, remodeled cortices, and a blastic appearance. Identifying that a lesion is present
would rule out fracture malunion and residual PFFD as answers.

Reference:
Unni KK: Dahlin's Bone Tumors, Ed 5 1996 pp 355-433
135. Which of the following is considered a risk factor for osteoporosis?

1- Obesity
2- Mediterranean heredity
3- Fair skin and hair
4- A history of manual labor
5- Late-onset menopause
Answer: 3.

“Genetic predisposition (individuals who are fair-skinned and small, have hypermobile
joints, are of Northern European ancestry, or have scoliosis), cigarette smoking, and
excessive alcohol intake are other risk factors.”

References:
Beatty JH (ed): Orthopaedic Knowledge Update 6. Rosemont, IL, American Academy of
Orthopaedic Surgeons, 1999, p. 153
143. A 76-year-old woman has had generalized muscle weakness and arthralgias for
the past 2 years. Radiographs show generalized osteopenia and a pseudofracture
(Looser's zone) in the inferomedial aspect of the femoral neck. Laboratory studies
show normal serum calcium and hemoglobin levels and a mildly elevated alkaline
phosphatase level. What is the most likely diagnosis?

1- Osteomalacia
2- Osteoporosis
3- Multiple myeloma
4- Leukemia
5- Paget's disease of bone
Answer: 1

Osteomalacia is a metabolic disorder in which there is inadequate mineralization of newly formed osteoid. It can
result from vitamin D deficiency, vitamin D resistance, intestinal malabsorption, acquired or hereditary renal
disorders, intoxication with heavy metal such as aluminum or iron, and other assorted etiologies. The childhood
form of osteomalacia is termed rickets. The diagnosis of osteomalacia often is difficult because patients usually
have nonspecific complaints such as muscle weakness of diffuse aches and pains. Radiographic evidence of
osteomalacia often mimics other disorders including osteoporosis. However, the presence of pseudofractures or
Looser’s transformation zones is good evidence that some degree of osteomalacia is present. Usual lab finds –
elevated alkaline phosphatase, low calcium or low phosphorus levels. Serum assays for vitamin D metabolites help
clarify the abnormality.
Pagets disease of bone is characterized by bone resorption and formation, showing areas of wide lamellae and
irregular cement lines, which produce the characteristic mosaic pattern appearance of pagetic bone. Two markers
are used to follow the course of disease- serum alkaline phosphatase and urinary piridinium cross links.
Osteoporosis is a silent disease, without specific laboratory findings. The only radiographic finding can be a
fracture (most common wedging of vertebrae), or bone loss, which is detected on radiographs only after 30% to
50% of the mineral is lost.
Multiple Myeloma will present as lytic lesions in bone, and not as Looser’s lines.
Leukemia is more common in children, with skeletal involvement in 50% of cases. Diffuse osteopenia is the most
frequent manifestation. Sometimes lucencies and periostitis may mimic osteomyelitis.

References:
Beaty JH (ed): Orthopedic Knowledge Update 5. Rosemont, IL, American Academy of Orthopedic Surgeons,
1999, pp427-439.
150. What type of chondrosarcoma has the lowest 5-year disease-free survival rate?

1- Chondrosarcoma developing in an osteochondroma


2- Chondrosarcoma developing in Ollier's disease
3- Intermediate grade (grade 11)
4- Mesenchymal
5- Clear cell
Answer: 4

Just Memorize.
159. A 21-year-old marathon runner who is 5' 2" tall and weighs 95 lb reports the
onset of left leg pain when she increased her training program 2 months prior to
an event. History reveals that her last menstrual period was 18 months ago. Bone
density of the spine tested by DEXA is 1.8 standard deviations below the mean
for age. A bone scan is normal. Management should include

1- cessation of running and 10 mg of alendronate per day.


2- nutritional counseling and a stretching program.
3- an ultrasound of the heel and 500 mg of calcium per day.
4- calcitonin, a short leg cast, and multivitamins.
5- evaluation of the amenorrhea, achievement of nutritional balance, and cross
training.
Answer: 5

Amenorrhea in the female athlete may be related to a low body fat percentage and/or
stress. The incidence approaches 50 % in elite runners and is related to stress fx
(osteopenia) and eating disorders. Dietary management and birth control pills are helpful
for treating the problem. A period of cross training would help to alleviate
supraphysiologic stresses and maintain cardiovascular fitness (5 correct). A bone scan fails
to show evidence of tibial stress fx so casting is not indicated (4 incorrect).
Bisphosphonates are not indicated for this process which should be amendable to dietary
management (1 incorrect).

References:
OKU: Sports Medicine 2 pp 43-47.
171. Normal mineralization of bone is seen in which of the following conditions?

1- Rickets
2- Seizure disorder treated with phenytoin
3- Renal osteodystrophy
4- Fanconi syndrome type II.
5- Osteoporosis
Answer: 5

Rickets = Osteomalacia in Adults = failure of meneralization due to many causes. Labs are
low normal Ca, low Pa, increased PTH, low Vit D. Many physical deformities result (Miller
p31). Tx with Vit D (5000 IU daily) Phenytoin administration is one of the many causes of
rickets. Renal Osteodystrophy is another cause of rickets which is a failure of
meneralization
173. Which of the following bone lesions arises exclusively in the epiphysis or
apophysis of the long bones?

1- Giant cell tumor


2- Aneurysmal bone cyst
3- Unicameral bone cyst
4- Fibrous dysplasia
5- Chondroblastoma
Answer: 5

Chondroblastomas (and infection) is the only lesion that arises from the epiphysis. The
epiphyseal location is a major clue for making the diagnosis but 52% in the series quoted also
involved the metaphysis. However none of the tumors arising in long bones involved only the
metaphysis. The radiographic appearance of chondroblastomas of long bones is relatively
typical when the tumor involves the epiphysis or an apophysis, but radiographic features are
less typical for flat bone lesions. The most frequent feature of chondroblastomas in general
was an eccentric oval or round lytic lesion that was well defined without a sclerotic rim and
showed no sclerosis of the surrounding bone and that eroded or destroyed the adjacent cortex
without periosteal reaction. A sclerotic rim and matrix calcification are helpful in making the
diagnosis , but these features generally are not present. Chondroblastoma can be treated
effectively with curettage and bone grafting. Giant cell tumors arise in both the epiphyseal and
metaphyseal region of long bones. Aneurysmal bone cysts are located eccentrically in the
metaphysis of a long bone and are usually well delineated. The unicameral bone cyst is a benign
tumor that develops in metaphyseal bone adjacent to the physis. Fibrous dysplasia frequently
affects more that one bone on the same side and is usually metaphyseal.

References:
Dorfman HD: Bone tumors. St.Louis, MO, Mosby, 1998, pp296-297

Turcotter RE: Chondroblastoma. Hum Pathol 1993;23:944-949


183. An 11-year-old boy has an enlarging, slightly tender mass over the proximal
phalanx of the third digit. The plain radiograph and a biopsy specimen are shown
in Figures 39a and 39b. What is the most likely diagnosis?

1- Giant cell tumor


2- Chondroblastoma
3- Periosteal chondroma
4- Aneurysmal bone cyst
5- Nonossifying fibroma
Answer: 3

Figure 39a shows a saucer-like erosion with a well-defined rim of reactive bone underlying the lesion which is
characteristic of periosteal chondroma. Lesions may measure up to 4cm and calcifications may be present. Figure 39b
shows lobules of hypercellular, immature cartilage; double nuclei; faint staining matrix and little, if any calcification, and
mild cellular atypia which is consistent with this lesion.
Periosteal chondromas are benign cartilaginous tumors forming beneath the periosteum and external to the cortex of
bone. This extremely rare lesion, with few cases reported in the literature can cause saucerization of the underlying
cortex and is mainly discovered in the third to fourth decades of life (case reports involve children as young as 4) and
arises from subperiosteal cartilage formation. Male to female ration of 2:1. The classic location for this tumor is
metaphyseal cortex of proximal humerus. Surgical treatment consists of resection of symptomatic larger lesions with a
wide or marginal margin, recurrence with intralesional resection is common. Pain only occurs if the lesion is in proximity
with a joint. Regression with maturity does not occur. Asymptomatic patients with latent lesions may be observed with
serial radiographs.
Chondroblastoma appear more aggressive radiologically, begin in the epiphyseal portion of long bones, generally are
painful and patients present with associated joint swelling. Histologically this tumor shows immature chondroblasts,
giant cells, less cartilaginous matrix and a greater degree of cellular atypia and hypercellular content.
ABC’s histologically are composed of spindle cells, some bone formation and large bone cysts that usually are blood-
filled and pain is the most common symptom.
Giant cell tumors invariably present with pain and are usually located in the proximal tibia and histologically are
composed of multinucleated giant cells and not cartilage matrix.
NOF would not histologically show this immature cartilage matrix.

References:
Weiner SD, Iorio CD: Painless deformity of a long finger phalanx of a 4-year old girl. CORR 1999;369:357-65.

Scarborough MT, Moreau G: Benign cartilage tumors. Orthop Clin North Am 1996;27:583-89.
184. A 30-year-old woman has chronic ankle pain and swelling without any history of
trauma. Examination reveals diffuse swelling and soft-tissue fullness along the
anterior aspect of the ankle joint. Plain radiographs are normal. An MRI scan
shows an effusion of the ankle and a soft-tissue mass arising from the ankle joint
that is dark on both Tl- and T2weighted images. A needle biopsy specimen is
shown in Figure 40. Management should consist of

1- below-knee amputation and chemotherapy.


2- external beam radiation.
3- extra-articular resection and tibiotalar arthrodesis.
4- open synovectomy.
5- triple antibiotic therapy.
Answer: 4

Pigmented Villonodular synovitis is probably a proliferative reaction to some type of


inflammatory agent. This reaction, characterized by giant cells, villous and nodular masses
that fuse together in the synovial membrane to form a single mass. Arthroscopy and
biopsy are valuable in making the diagnosis. Figure 40 shows nodules of glandular tissue
w/ lipid filles histiocytes and giant cells. PVNS is relatively rare lesion, occurs in adults
and the knee is the usual joint affected. The lesion produces a bulky mass in the synovial
membrane and may even erode bone(this patient c/o mass at anterior ankle and has h/o
swelling/inflammation/pain). The lesion in synovial joints responds well to surgical
excision of the involved area of synovial membrane. For diffuse and widespread intra-
articular disease, extensive synovectomy (either open or artroscopic is required.

Antibiotics will not help, fusion is not necessary, radiation is not helpful and BKA and
chemotherapy is too aggressive for this benign lesion.

References:
Mizel MS, Miller RA, Scioli MW (eds): Orthopaedic Knowledge Update: Foot and Ankle
2. Rosemont IL, American Academy of Orthopaedic Surgeons, 1998, pp11-26.
195. An otherwise healthy 37-year-old man has had bilateral posterior heel pain for
the past year. Examination reveals fullness, warmth, and tenderness over the
posterior aspect of the heels. Radiographs are normal. Laboratory studies show a
normal CBC and an erythrocyte sedimentation rate of 50 mm/h (normal up to 20
mm/h), and an HLA-B27 is positive. What is the most likely diagnosis?

1- Rheumatoid arthritis
2- Ankylosing spondylitis
3- Lupus erythematosus
4- Reiter syndrome
5- Lyme disease
Answer: 4

Given the clinical presentation (37yo man, bilateral heel pain, elevated ESR, and HLA B-27 +) Reiter
syndrome is the logical choice. Reiter’s can consist of the triad of conjunctivitis, urethritis, and arthritis.
Unknown cause, 96% + for HLA B-27, lower extremities usually involves knees, ankles, feet. Heel pain
can be presenting symptom.
Ankylosing Spondylitis - Mainly effects axial skeleton, adolescent males, 90% + for HLA B-27, + hip and
BACK pain, XR show bilateral symmetric sacroiliac erosions, followed by joint space narrowing, then
ankylosis and late “bamboo spine”. Limitation of chest wall expansion is specific for AS.
Rheumatoid arthritis - Systemic disease affecting synovial tissues usually symmetrically, 3:1 female to male
ratio, 90% have FOREFOOT involvement (hallux valgus, dorsal subluxation of plantar fat pad (walking
on marbles).
Lyme disease - spirochete (Borrelia burgdorferi) transmitted by deer tic (Ixodes dammini). Target shaped
rash, fever, and systemic disease. Late musculoskeletal overuse syndrome. Endemic regions northeast
U.S., Minnesota, Michigan, Oregon and California. Get Lyme titers. High index of suspicion needed. Tx
with Zithromax.
Lupus erythrematosis - multisystem autoimmune disease, 9:1 female to male ratio, usually 2-3rd decade of
life, skin rashes, joints- nonerosive synovitis with little deformity, renal disorders-glomerular nephritis,
hematologic disorders-anemias, neurologic- seizures. Titers of anti-DS DNA and anti-Sm antibodies are
virtually diagnostic of lupus.

References:
Thomas FM, Mann RA: Arthritides, in Mann RA (ed); Surgery of the Foot and Ankle 1994 pp 618-619.
206. Which of the following diseases is characterized by a defect in type I collagen
metabolism? .

1- Diastrophic dwarfism
2- Osteogenesis imperfecta
3- Mucopolysaccharidosis
4- Pseudoachondroplasia
5- Multiple epiphyseal dysplasia
Answer: 2

1-Diastrophic dwarfism: caused by abnormal sulfate transporter leading to undersulfated


proteoglycans…proteoglycan swelling is part of the mechanism that produces growth
2- Osteogenesis imperfecta: the defect is in the gene for the 1 and 2 chains of type I
collagen
3- Mucopolysaccharidosis: lysosomal enzyme defect that alters the ability to mobilize
glycosaminoglycans (ex. Hunter’s and
Hurler’s disease)
4- Pseudoachondroplasia: mutation in fibroblast growth factor receptor 3
5- Multiple epiphyseal dysplasia: autosomal dominant defect in the pericentromeric region
of chromosome 19 and 1

Reference:
Simon SR (ed): Orthopaedic Basic Science. Rosemont, IL, American Academy of
Orthopaedic
Surgeons, 1994, pp 187-217.
247. Figure 60 shows the bone mineral density (BMD) report for a 57-year-old
Caucasian woman. According to the National Osteoporosis Foundation
guidelines, management should consist of

1- observation and serial BMD testing.


2- vitamin D and calcium dietary supplements only.
3- pharmacologic treatment if additional risk factors for fracture are positive.
4- pharmacologic treatment in the absence of additional risk factors.
5- pharmacologic treatment only if the patient has a history of a osteoporotic
fracture
Answer: 4

Physicians Guide to Prevention and Treatment of Osteoporosis: www.nof.org/physguide.


Hip BMD is the best predictor of hip fractures and predicts fracture at other sites as well as other
measurements. BMC is expressed as a relationship between two norms:
Z-score: expected BMD for the patient’s age and sex
T-score: BMD for a normal, young adult of same sex.
The difference between the patients score an Z-score is expressed in the form of standard deviations
above or below the mean. (one SD = 10-12% difference in bone density). The T-score is the
parameter correlated with risk of osteoporotic fracture. Interpretation of T-score is as follows:
Above (-1)SD Normal
Between (-1)SD and (-2.5)SD Osteopenic
Below (-2.5)SD Osteoporotic
Treatment: 1) Everyone gets Calcium(1200 mg), Vitamin D(400-800 IU), regular weight bearing
and muscle strengthening exercises, and advise to avoid tobacco and to keep alcohol intake to a
minimum. 2) Pharmacologic therapy (Hormone replacement, Alendronate, Calcitonin, Raloxifene)
is initiated when BMD T-score is <-2 and there are no risk factors or when T-score is <-1.5 and
there are risk factors present.

References:
National Osteoporosis Foundation Development Committee: Osteoporosis: Physician’s Guide to
Prevention and Treatment of Osteoporosis. Belle Mead, NJ, Excerpta Medica, 1998, pp 18-22.
254. What is the rate of bone loss per year at menopause?

1- 0.3% to 0.5% for 3 to 5 years


2- 0.3 % to 0.5 % until age 65 years
3- 2% to 3% for 2 to 3 years
4- 2% to 3% for 6 to 10 years
5- 4% to 5% for 2 to 3 years
Answer: 4

In OKU 6 and in two supporting papers from the New England Journal of Medicine the
rate of bone loss per year at menopause is 0.3% to 0.5% for 3 to 5 years.

You might also like